Một số chuyên đề Số học bồi dưỡng học sinh giỏi THCS

357 25 0
Một số chuyên đề Số học bồi dưỡng học sinh giỏi THCS

Đang tải... (xem toàn văn)

Tài liệu hạn chế xem trước, để xem đầy đủ mời bạn chọn Tải xuống

Thông tin tài liệu

Câu 65.. Chứng minh rằng nếu lấy 4 p chia cho 3 và loại bỏ phần dư thì nhận được một s ố là bình phương của một số nguyên lẻ.. Cho n là số tự nhiên lẻ. Viết một cách tùy ý 100 số đó nố[r]

(1)



Trịnh Bình sưu tầm

MỘT SỐ CHUYÊN ĐỀ SỐ HỌC

BỒI DƯỠNG HỌC SINH GIỎI THCS

(2)

MỘT SỐ CHUYÊN ĐỀ SỐ HỌC

LI NÓI ĐẦU

Nhằm đáp ứng nhu cầu giáo viên toán THCS học sinh về các chun đề tốn

HỌC Chúng tơi kham khảo qua nhiều tài liệu để viết chuyên đề vềnày nhằm đáp ứng nhu cầu về tài liệu hay cập nhật dạng toán số học thường kì thi gần

phương, phương trình nguyên, tổ hợp suy luận…

Các vị phụhuynh thầy dạy tốn có thể dùng có thểdùng chuyên đề này đểgiúp con em học tập Hy vọng chuyên đề số học sẽcó thể giúp ích nhiều cho học sinh phát huy

nội lực giải tốn nói riêng học tốn nói chung.

Mặc dù có sựđầu tư lớn vềthời gian, trí tuệsong khơng thể tránh khỏi hạn chế,

sai sót Mong sựgóp ý thầy, cô giáo em học!

Chúc thầy, cô giáo em học sinh thu kết quảcao từchuyên đềnày! THCS, website thuvientoan.net giới thiệu đến thầy cô em MỘT SỐ CHUYÊN ĐỀSỐ

(3)

Mục Lục

Trang

Lời nói đầu

Chủ đề Quan hệ chia hết tập hợp số Chủ đề Các tốn số phương Chủ đề Các toán số nguyên tố, hợp số

Chủ đề Các toán phương trình nghiệm ngun Chủ đề Các tốn tổ hợp, suy luận

Chủ đề Các toán phân nguyên, phần lẻ

Hướng dẫn giải – đáp số

Chủ đề Quan hệ chia hết tập hợp số Chủ đề Các tốn số phương Chủ đề Các toán số nguyên tố, hợp số

Chủ đề Các tốn phương trình nghiệm ngun Chủ đề Các toán tổ hợp, suy luận

(4)

các chuyên đề bồi dưỡng

Chương I

QUAN HỆ CHIA HẾT TRONG TẬP HỢP SỐ

A. KiÕn thøc cÇn nhí

1 Định nghĩa phép chia

Cho hai số nguyên a b b ≠0 ta ln tìm hai số ngun q r cho a bq r= + , với r b≤ ≤ Trong đóa số bị chia, b số chia, q thương, r số dư

Khi a chia cho b số dư r∈{0;1; 2; 3; ; b}

• Nếu r 0= a bq= , ta nói a chia hết cho b hay b chia hết a Ký hiệu: a b hay

b a

Vậy a chia hết cho b tồn số nguyên q cho a bq=

• Nếu r 0≠ , ta nói a chia b có số dư r 2 Một số tính chất cần nhớ

• Tính chất Mọi số nguyên khác chia hết cho

• Tính chất Số ngun a chia hết cho số nguyên b số nguyên b chia hết cho số nguyên c số nguyên a chia hết cho số ngun c

• Tính chất Số nguyên a chia hết cho số nguyên b ngược lại a= ±b

• Tính chất Nếu a.b m (b,m)=1thì a m

• Tính chất Nếu hai số ngun a b chia hết cho m (a b m± )

• Tính chất Nếu a chia hết cho m n, (m,n)=1 a mn

• Tính chất Nếu số ngun a chia hết cho số nguyên b số nguyên c chia hết cho số ngun d tích ac chia hết cho tích bd

• Tính chất Trong n số nguyên liên tiếp tồn số ngun chia hết cho n

• Tính chất Nếu a b 0− ≠ với a, b số tự nhiên an−b n Nn( ∈ ) chia hết cho a b−

• Tính chất 10 Nếu a b 0+ ≠ với a, b số tự nhiên n số tự nhiên lẻ an+bn chia hết cho a b+

(5)

3 Một số dấu hiệu chia hết

Đặt A a a a a a= n n 1− 0, với a ;a ; ;a ;a ;an n 1− chữ số Khi ta có dấu hiệu chia hết sau

• Dấu hiệu chia hết cho 2: Số tự nhiên A chia hết cho a0∈{0; 2; 4;6;8}

• Dấu hiệu chia hết cho 5: Số tự nhiên A chia hết cho a0∈{ }0;

Từ suy A chia hết cho 10 a0 =0

• Dấu hiệu chia hết cho 25: Số tự nhiên A chia hết cho 4(hoặc 25) a a 1 0 chia hết cho (hoặc 25)

• Dấu hiệu chia hết cho 125: Số tự nhiên A chia hết cho 8(hoặc 125)

a a a chia hết cho (hoặc 125)

• Dấu hiệu chia hết cho 9: Số tự nhiên A chia hết cho 3(hoặc 9) tổng chữ số số A chia hết cho 3(hoặc 9)

• Dấu hiệu chia hết cho 11: Số tự nhiên A chia hết cho 11 hiệu tổng chữ số hàng lẻ tổng chữ số hàng chẵn số chia hết cho 11

B CÁC DẠNG TOÁN THƯỜNG GẶP

Dạng 1: Sử dụng tính chất n số nguyên liên tiếp có số chia hết

cho n (n ≥ 1)

* Cơ sở phương pháp: Sửdụng tính chất như: tích hai sốnguyên liên tiếp chia hết cho 2, tích ba sốnguyên liên tiếp chia hết cho chia hết cho Chúng ta vận dụng linh hoạt tính chất nhiều tốn vềchia hết

Bài toán Chứng minh rằng:

a) Tích sốnguyên liên tiếp chia hết cho b) Tích sốchẵn liên tiếp chia hết cho c) Tích sốnguyên liên tiếp chia hết cho 120

Hướng dẫn giải

a) Trong số nguyên liên tiếp có số chia hết cho số chia hết tích số nguyên liên tiếp chia hết cho (do (2, 3) = 1)

b) Hai số chẵn liên tiếp có dạng 2n (2n + 2) với n Z∈

Do tích hai số ngun liên tiếp có dạng 4n(n + 1) Do n n + hai số nguyên liên tiếp nên n n 2( + ) Vì 4n n 8( + )

(6)

c) Ta có 120 = 3.5.8

Do sốnguyên liên tiếp có sốliên tiếp nên theo ý a) ta có tích sốnguyên liên tiếp chia hết cho

5 sốnguyên liên tiếp có sốchẵn liên tiếp nên theo ý b) ta có tích số ngun liên tiếp chia hết cho

Mặt khác sốngun liên tiếp ln có sốchia hết tích chúng chia hết cho

Vậy tích sốngun liên tiếp ln chia hết cho 120

Chú ý: Tổng qt ta có tích n sốtựnhiên liên tiếp chia hết cho n! Bài toán Chứng minh tích số chẵn liên tiếp chia hết cho 48

Hướng dẫn giải

Ba số chẵn liên tiếp có dạng 2n, (2n + 2) (2n + 4) với n Z∈

Do tích hai số ngun liên tiếp có dạng 8n(n + 1)(n + 2)

Do n, (n + 1) (n + 2) số nguyên liên tiếp nên n n n 6( + )( + ) Vì n n n 2( + )( + )=6m m Z( ∈ )

Do tích số chẵn liên tiếp 8n n n 2( + )( + )=48m 48

Vậy toán chứng minh

Bài toán Chứng minh với số nguyên n n n3− chia hết cho 6

Hướng dẫn giải

Ta có:

( ) ( ) ( )

3− = − =1 −1 +1

n n n n n n n

Biểu thức tích sốnguyên liên tiếp nên sốchia hết cho 2, sốchia hết cho mà (2, 3) = nên (n n3− )6

Bài toán Chứng minh với số nguyên lẻ n thì

1

− − +

n n n chia hết cho 128

Hướng dẫn giải

Ta có:

( ) ( ) ( )( ) ( ) (2 )

6− 4− 2+ =1 − −1 − =1 2−1 4− =1 2−1 2+1

n n n n n n n n n n

nlà sốlẻnên đặt n = 2k + 1 (k N∈ ) Ta có:

(7)

( 2 )2 ( )2 ( 2 )2 ( )

1  1 4 4 

− = + −  = + = + 

 

n k k k k k

Ta có k(k + 1) chia hết nên 4k k( +1)264 Mặt khác: n2 + =1 2( k+1)2 + =1 4k2+4k+ =2 2( k2+2k+1 2) Do n n n6− 4− 2+ =1 (n2−1) (2 n2+1 128) (đpcm)

Chú ý: Bình phương sốlẻlà sốlẻ

Dạng 2: Phân tích thành nhân tử

* Cở sở phương pháp: Để chứng minh A(x) chia hết cho pta phân thích A(x) = D(x).p, cịn khơng thểđưa phân tích ta cóthểviết p = k.q

Nếu (k, q) = ta chứng minh A(x) chia hết cho k q.

Nếu ( )k q, ≠1 ta viết A(x) = B(x).C(x) rồi chứng minh B(x) chia hết cho k C(x) chia hết choq.

* Ví dụ minh họa:

Bài tốn Cho a, b, c sốnguyên khác thỏa mãn điều kiện:

 

+ + = + +

 

 

2

2 2

1 1 1 .

a b c a b c

Chứng minh rằng: a3+b c3+ chia hết cho 3.

(Đềthi HSG lớp TP Thanh Hóa 2016-2017)

Hướng dẫn giải

Từ giả thiết  + +  = + + ⇔  + + =

   

2

2 2

1 1 1 2 1 0 a b c a b c ab bc ca

Vì a, b, c ≠0 nên a + b + c =

( ) ( )

⇒ + = − ⇒ + = −

⇒ + + + = − ⇒ + + =

3

3 3

3 3

a b c

a b c

a b 3ab(a b) c a b c 3abc

Vậy a3+b c 33+ 3 với a, b, c ∈Z

Bài toán Cho A=1.2.3 29, B=30.31.32 58 Chứng minh A + B chia hết cho 59

(8)

Hướngdẫn giải

Ta có:

(59 29 59)( 28 59)( 27 59 1) ( ) 59 1.2.3 29 59 ( ) 59 59

= − − − − = − = − ∈ ⇒ + = 

B k k A k Z A B k

Vậy A + B chia hết cho 59

Bài toán Cho 3 sốnguyên dương x, y, z Chứng minh rằng:

( ) (5 ) (5 )5

− + − + −

x y y z z x chia hết cho 5(x y y z z x− )( − )( − ) Hướng dẫn giải

Đặt a x y b y z= − , = − ⇒ − = − +z x (a b)

Do ta cần chứng minh: a b5+ 5− +(a b)5chia hết cho −5ab a b( + ) Ta có: a b5+ − +5 (a b)5 = −(5a b4 +10a b3 2+10a b2 3+5ab4)

= −5ab a b( 3+ 3+2a b2 +2ab2)

( )( ) ( )

( )( )

2

2

5

5

 

= −  + − + + + 

= − + + +

ab a b a ab b ab a b

ab a b a ab b

Do tốn chứng minh

Bài toán Chứng minh với ba sốtựnhiên a,b,c có sốlẻvà hai số chẵn ta ln có ( ) (3 ) (3 ) (3 )3

c b a a c b c b a c b

a+ + − + − − + − − − + Chia hết cho 96

(Trích đềthi HSG lớp tỉnh Phú Thọ2015)

Hướng dẫn giải

Đặt a + b - c = z; b + c - a = x; a + c - b = y x + y + z = a + b + c

Ta có (x y z+ + )3−x y z3− 3− =3(x y)(y z)(x z) 3.2c.2a.2 b 24abc+ + + = =

Do sốa, b, c có sốchẵn nên abc chia hết cho 24abc chia hết cho 24.4 = 96 Vậy toán chứng minh

Dạng 3: Sử dụng phương pháp tách tổng

* Cở sở phương pháp: Đểchứng minh A(x) chia hết cho pta biết đổi A(x) thành tổng hạng tửrồi chứng minh hạng tửchia hết cho p.

* Ví dụ minh họa:

(9)

Bài toán Chứng minh m, n số nguyên ta có:

a) n n( +11 6) b mn m n) ( 2− 2)6 c n n) ( +1 2)( n+1 6)

Hướng dẫn giải

a) Ta có: n n( 2+11)=n3+11n n n= 3− +12n=(n−1) (n n+ +1 12) n Dễ chứng minh: (n−1) (n n+1 6, 12 6) n (n Z∈ )

Do đó: n n( 2+11 6)

b) Ta có: ( 2− 2)= ( 2− −1) ( 2−1)= ( 2− −1) ( 2−1)

 

mn m n mn m n mn m mn n

Do: mn m( 2− =1) n m( −1) (m m+1 6,) mn n( 2− =1) m n( −1) (n n+1 6) Do đó: ( )

2− 6

mn m n

c) Ta có: n n( +1 2)( n+ =1) (n n+1)(n+ + − =2 n 1) (n n+1)(n+ +2) (n−1) (n n+1) Do: n n( +1)(n+2 6,) (n−1) (n n+1 6)

Do đó: n n( +1 2)( n+1 6)

Chú ý: Tách tổng phương pháp chứng minh chia hết mà lời giải dễ hiểu, ngắn gọn đẹp mắt nên thường trình bày tốn giải nhiều phương pháp, nhiên để áp dụng em cần linh hoạt việc tách

Ví dụ: câu a) ta thấy 12n chia hết ta tách riêng phần cịn lại phân đưa dạng tích, dựa vào tính chất chia hết tích số tự nhiên dễ dàng chứng chia

Câu b) nghĩ việc thêm bớt để tạo tổng hai tích số tự nhiên liên tiếp Tương tự câu c) dễ dàng tách 2n + = (n – 1) + (n + 2) để đưa tổng hai tích số tự nhiên tiếp

Bài toán Chứng minh rằng:n n5có chữ sốtận giống với n sốtựnhiên.

Hướng dẫn giải

Đểchứng minh n n5có chữ sốtận giống ta chứng minh (n n5− )10 Thật vậy: ( ) ( )( ) ( ) ( )

1 1  5

− = − = − + = −  − + 

n n n n n n n n n n

( )( ) ( ) ( )( ) ( )( ) ( ) ( )

1 1 1

− − + − = − − + + + − +

n n n n n n n n n n n n n

(10)

Nhận xét: (n−2)(n−1) (n n+1)(n+2)là tích năm sốtựnhiên liên tiếp nên chia hết cho chia hết cho 10

Mặt khác (n−1) (n n+1)là tíchcủa sốtựnhiên liên tiếp nên chia hết

( ) ( )

5 n−1 n n+1 chia hết cho 10

Do (n n5− )10 vậy tốn chứng minh. Bài toán a) Chứng minh

5 + 15+

n n n là sốnguyên với mọi n Z

b) Chứng minh 12 24+ +

n n n là sốnguyên với n sốnguyên chẵn

Hướng dẫn giải

a) Ta có:

15 = −15 = − −5

n n n n n n

Do đó: 5

5 15 5

− −

+ + = + + − − = + +

n n n n n n n n n n n n n

Từcác thí dụtrên ta dễdàng chứng minh được: (n n5− ) (5, n n3− )3 do tốn chứng minh

b) Do n số nguyên chẵn nên n = 2m (với m Z∈ )

Do đó: 3 3 ( 2)( 1)

12 24 6

+ +

+ +

+ + = + + = = m m m

n n n m m m m m m

Theo ý c) thí dụ6 ta có n n( +1 2)( n+1 6) tốn chứng minh Bài tốn Chứng minh ax bx c Z x Z2+ + ∈ ∀ ∈, khi chỉ 2 ,a a b+ ,c Z∈

Hướng dẫn giải

Ta có: 2 ( ) 2 ( 1) ( ) .

2

+ + = − + + + = x x + + +

ax bx c ax ax a b x c a a b x c

Dễthấy: ( 1)

− ∈ x x

Z x (x – 1) là hai sốnguyên liên tiếp Do đó: ax bx c Z x Z2 + + ∈ ∀ ∈, khi chỉ 2 ,a a b+ ,c Z∈

Bài toán Cho số nguyên a ;a ; ;a1 2 n Đặt A a a a= 1+ 2+ + n = 3+ 3+ +

1 n

B a a a Chứng minh A chia hết cho B chia hết cho

Hướng dẫn giải

Trước hết ta chứng minh bổ đề: Với số ngun a ta ln có a a 63− 

(11)

Thật vậy, ta có a a3− =(a a a 1− ) ( + )

Ta thấy ba số tự nhiên liên tiếp có số chia hết cho có số chia hết cho 3, lại có nguyên tố nên ta suy a a3− =(a a a 6− ) ( + ) Xét hiệu sau

( ) ( ) ( ) ( ) ( )

− = 3+ 3+ + − + + + = 3− + 3− + + −

1 n n 1 2 n n

B A a a a a a a a a a a a a Áp dụng bổ để ta ( 3− ) ( 3− ) ( − )

1 2 n n

a a 6; a a 6; ; a a

Do ta B A Suy A chia hết cho B chia hết cho 6.− 

Dạng 4: Sử dụng đẳng thức

Cở sở phương pháp: Nếu a, blà sốnguyên thì:

a bnn chia hết cho a – bvới n sốtựnhiên a b

a bnn chia hết choa + bvới n sốtựnhiên chẵn a≠ −b a bn+ n chia hết cho a + bvới n sốtựnhiên lẻvà a≠ −b ( + ) = +

n n

a b ka b với klà sốnguyên, nlà sốtựnhiên

(a+1)n=ac+1 (a−1)n =ac+ −( )1 n , nlà sốtựnhiên * Ví dụ minh họa:

Bài tốn Với n sốtựnhiên chẵn. Chứng minh rằng: a) 22 55

22 +55 b) 20n+16n −3 323.n−  Hướng dẫn giải

a) Ta có: 22 55 ( )22 ( )55 ( )22 ( )55

22 55 21 56 7

= + = + + − = + + −

P BS BS

= BS + + BS – = BS nên 22 55

22 +55 chia dư

b) Ta có: 323 17.19= Ta biến đổi 20n+16n −3 1n− =(20n− +1) (16n−3n) Ta có: (20n−1 : 20 1) ( − ⇒) (20n−1 19)

Mặt khác n sốchẵn nên (16n−3n)(16 3+ )⇒(16n −3 19n) Do (20 1n − +) (16n−3 19n) ⇒(20n+16n−3 19 1n − ) ( ) Ta biến đổi 20n +16n−3 1n− =(20n−3n) (+ 16 1n − n)

Ta có: (20n−3 : 20 3n) ( − )⇒(20 17n− )

Mặt khác n sốchẵn nên (16 1n− n)(16 1+ ⇒) (16n−3 17 2n) ( )

(12)

Do (17, 19) =1 nên từ (1) (2) suy ra: 20n+16n−3 323.n −  Bài toán Chứng minh với sốtựnhiên n ta có:

n 2n n n 2n 2n n

a) 11 + +12 + 133 b) + +26.5 +8 + 59 c) 7.5 +12.6 19

  

Hướng dẫn giải

a) Ta có: 11n 2+ +122n 1+ =11 11 12.122 n+ 2n =121.11 12.144n+ n

=(133 12 11 12.144− ) n + n =133.11 12 144n+ ( n−11n) Do 133.11 133n và 12 144( n−11n)(144 11 hay 12 144− ) ( n−11 133n) Nên 133.11 12 144n + ( n−11n)⇒11n 2+ +122n 1+ 133 (đpcm)

b) Ta có: 5n 2+ +26.5n+82n 1+ =25.5n+26.5n+8.82n =51.5n+8.64n

(59 8.64) n n 59.5 64n ( n 5n)

= − + = + −

Vì (64n−5n)(64 5− ⇒) (64n −5 59n)

Nên 59.5n+8 64( n−5 59n) ⇒5n 2+ +26.5n+82n 1+ 59(đpcm) c) Ta có: 7.52n +12.6n =7.25n+(19 6− ) n =19.6 25 6n+ ( n− n)

Vì (25 6n− n)(25 6− ⇒) 7 25 19( n− n)

Nên 19.6n+7 25( n−6 19n) ⇒57.52n+12.6 19n (đpcm) Bài toán Chứng minh A 1993= 1997 +1997199330

Hướng dẫn giải

Sửdụng tính chất (a b+ )n =ka b+ nvới klà sốnguyên, nlà sốtựnhiên. Ta có:

( ) ( )

( )

( )

1997 1993

1997 1993

1997 1993

1993

1993

A 1993 1997 1980 13 2010 13 1980c 13 2010d 13

1980c 2010d 13 13 30 66c 67d 952.13 30

= + = + + −

= + + −

= + +

= + + 

Bài toán Chứng minh C 5= n( n+ −1 3) (n n+2 91 n N n) ( ∈ )

(Chuyên sư phạm Hà Nội 1997 – 1998)

Hướng dẫn giải

(13)

Sửdụng tính chất (a b+ )n=ka b+ n, (a+1)n=ac+1, (a−1)n =ac+ −( )1 nvới k

sốnguyên, nlà sốtựnhiên Ta có:

( ) ( ) ( )

( )

25 18 12

21 14 21 14 7

= + − −

= + + − + − +

= + + − − − −

= − − 

n n n n

n n n n

n n n n

C

c d e

c d e Mặt khác:

( ) ( ) ( )

( ) ( )

( )

26 13 13 26 13 13 13 13

= − + − + − −

= + − + − − − − −

= − − 

n n n n

n n n n

C

f g h

f g h

Vì (13, 7) = nên C 7.13 91  =

Bài toán Chứng minh rằng: 3 3

1 100

= + + + +

A chia hết choB = + + + +1 100

Hướng dẫn giải

Ta có B = (1 + 100) + (2 + 99) + …+ (50 + 51) = 101.50

Để chứng minh A chia hết cho B ta chứng minh A chia hết cho 50 101 Ta có:

( ) ( ) ( )

( )( ) ( )( ) ( )( )

( ) ( )

3 3 3

2 2 2

2 2 2

1 100 99 50 51

1 100 100 100 99 2.99 99 50 51 50 50.51 51

101 100 100 2.99 99 50 50.51 51 101

= + + + + + +

= + + + + + + + + + + + +

= + + + + + + + + + 

A

Ta lại có: ( 3) ( 3) ( 3)

1 99 98 50 100

= + + + + + +

A

Mỗi số hạng chia hết cho 50 nên A chia hết cho 50 (2)

Từ(1) (2) suy A chia hết cho 101 50 nên A chia hết cho B

Bài toán Chứng minh với sốnguyên n ta có: A 2= 5+ 5+3 n5+ + 5chia hết choB= + + + +1 n

(Chuyên sư phạm Hà Nội 2001) Hướng dẫn giải

Ta có cơng thức quen thuộc ( 1)

+ = + + + + =n n

B n

(14)

Lại có: 2A=(n 15+ +) (n 1− )5+25 + (n 2− )5+35+ + + n( 5)

   

   

Nhận thấy số hạng chia hết cho (n +1) nên 2A n( +1) ( )1 Lại có 2A 2n− =(n 1− )5+15 + (n 2− )5+25+

   

    chia hết cho n

Do 2n n5 nên 2A n ( )2 Do 2n n5 nên 2A n ( )2

Từ(1) (2) suy 2A chia hết cho n(n + 1) do 2A 2B ⇒A B (đpcm) Chú ý: Ta có cơng thức tổng qt: với n sốngun dương k sốtựnhiên lẻthì:

( )

( ) ( )

)1

)1 2

+ + + + + +

+ + + +

 

k k k

k

k k

a n n

b k n n

Đây tập, bạn đọc có thểtựchứng minh đểcủng cố kiến thức

Dạng 5: Sử dụng phương pháp xét số dư

* Cơ sở phương pháp: Đểchứng minh A(n)chia hết cho p ta xét sốncó dạng n = kp + r

với r∈{0;1; 2; ;p−1 } * Ví dụ minh họa:

Bài toán Chứng minh n 2n 7( 2+ ) chia hết cho với số nguyên dương n.

Hướng dẫn giải

Xét trường hợp:

- Trường hợp 1: n = 3k n 2n 7( 2+ )=3k 3k ( )2+7=3k 18k 3( 2+ )

- Trường hợp 2: n = 3k +

( ) ( ) ( )

( ) ( )( )

 

+ = +  + + 

 

 

= +  + + + = + + +

2

2

n 2n 3k 3k

3k 18k 12k 3k 6k 4k 3 - Trường hợp 3: n = 3k +

( ) ( ) ( )

( ) ( )( )

 

+ = +  + + 

 

 

= +  + + + = + + +

2

2

n 2n 3k 2 3k

3k 18k 24k 3k 6k 8k 3 Từ3 trường hợp suy n 2n 7( 2+ ) chia hết cho 3.

Bài toán Chứng minh với sốtựnhiên n ta có: n 2n 7n 1( + )( + )chia hết cho

(15)

Hướng dẫn giải

Trong sốn (7n + 1) phải có sốchẵn nên n 2n 7n 2( + )( + )

Mà (3, 2) = nên ta cần chứng minh n 2n 7n 3( + )( + )

Xét trường hợp:

- Trường hợp 1: n = 3k n 2n 7n 3k 6k 21k 3( + )( + =) ( + )( + )

- Trường hợp 2: n = 3k + 2n 7+ =(6k 3+ ) ⇒n 2n 7n 3( + )( + )

- Trường hợp 3: n = 3k + 7n 1+ =(21k 15 3+ ) ⇒n 2n 7n 3( + )( + )

Từ3 trường hợp suy n(2n + 7)(7n + 1) chia hết cho

Bài toán Cho a, b, c số nguyên Chứng minh (a3+b c 93+ 3) thì ba số a, b, c chia hết cho

Hướng dẫn giải

Với a, b, c số nguyên ta có a 3q r ; b 3q= 1+ 1 = 2+r ;c 3q2 = 3 +r3 với q ;q ;q1 2 3 số nguyên số dư r ;r ;r1 2 3∈ −{ 1;0;1}

Dễ thấy = = =

1

r r ;r r ;r r Từ ta

( ) ( ) ( )

= + = + = + = + = + = +

3 3

1 1 2 3 3

a 3q r 9k r ; b 3q r 9k r ;c 3q r 9k r

Khi ta 3+ 3+ = ( + + ) (+ + + )

1 3

a b c k k k r r r

Mà theo giả thiết ta có (a3+b c 93+ 3) Do nên ta suy ( + + )

r r r Dễ thấy r r r1+ +2 3 ≤3, đósuy r r r 01+ + =2 3

Do r ;r ;r1 2 3∈ −{ 1;0;1} nên từ r r r 01+ + =2 3 suy r ;r ;r1 2 3 có số Điều có nghĩa ba số a, b, c có số chia hết cho

Bài toán Cho x, y, z số nguyên thỏa mãn (x y y z z x− )( − )( − )= + +x y z ( )* Chứng minh (x y z+ + )chia hết cho 27

Hướng dẫn giải

(16)

Ta xét trường hợp sau:

- Nếu số x, y, z chia cho có số dư khác (x – y), (y – z), (z – x) không chia hết cho (x – y)(y – z)(z – x) sẽkhơng chia hết cho Nhưng tổng số(x + y + z) chia hết cho điều trái với điều kiện (*) tốn, thếtrường hợp không thểxảy

- Nếu sốx, y, z có số chia cho có sốdư (x – y), (y – z), (z – x) sẽcó hiệu chia hết cho (x – y)(y – z)(z – x) chia hết cho Nhưng tổng số (x + y + z) không chia hết cho điều trái với điều kiện (*) bày tốn, thếtrường hợp khơng thểxảy

Vậy sốx, y, z chia cho phải số dư, (x – y), (y – z), (z – x) sẽđều chia hết tích (x – y)(y – z)(z – x) sẽchia hết cho 27 Mặt khác theo giảthiết (*) ta có (x – y)(y – z)(z – x) = x + y + z nên (x + y + z) chia hết cho 27

Vậy toán chứng minh

Dạng 6: Sử dụng phương pháp phản chứng

* Cơ sở phương pháp: Đểchứng minh A(x)không chia hết cho nta giả sửA(x)chia hết cho n sau dùng lập luận đểchỉra mâu thuẩn đểchỉra điều giả sửlà sai

* Ví dụ minh họa:

Bài tốn Chứng minh n2+ −n 16 khơng chia hết cho 25 với mọi sốtựnhiên n

Hướng dẫn giải

Giả sử n2 + −n 16 chia hết cho 25.

Do n2+ −n 16 chia hết cho 25 nên chia hết cho 5. Ta có: n2+ −n 16=(n n 10+ )( − )−

Do n2+ −n 16 và 10 chia hết (n + 3)(n – 2) chia hết cho (1)

Mặt khác (n + 3) (n – 2) có hiệu nên chúng chia hết cho không chia hết cho 5, lại (1) nên (n + 3) (n – 2) chia hết cho suy ta có (n + 3)(n – 2) chia hết hết cho 25

Tức n2+ −n 16 chia cho 25 dư 15 mâu thuẫn với giả sử, toán chứng minh.

Bài toán Chứng minh với số tự nhiên n,n3 chia hết cho n cũng chia hết cho

Hướng dẫn giải

(17)

Giả sửn không chia hết cho Khi n có dạng n = 3k +1 n = 3k + (với k số tự nhiên)

Nếu n = 3k + n3 =(3k 1+ )3 =27k3+27k2 +9k 1+ khơng chia hết cho 3 Nếu n = 3k + n3 =(3k 2+ )3 =27k3+54k2+36k 4+ không chia hết cho 3

Cảhai trường hợp mâu thuẫn suy n phải hết cho toán chứng minh

Bài toán Chứng minh số dương có tổng bình phương chia hết cho số phải chia hết cho

Hướng dẫn giải

Giả sử2 sốnguyên dương a, b có sốkhông chia hết cho 3, chẳng hạn sốđó a Khi a = 3k + a = 3k + với k sốtự nhiên, ta cóa2 = +3l 1 nếu sốb chia hết cho khơng chia hết cho a2+b2 ln có dạng 3m + 3m +2, nghĩa không chia hết cho 3, mâu thuẫn

Vậy toán chứng minh

Dạng 7: Sử dụng phương pháp quy nạp

* Cơ sở phương pháp: Để kiểm tra mệnh đề với số tự nhiên n p≥ ta làm sau:

1) Kiểm tra mệnh đềđúng với n = p.

2) Giả sửmệnh đềđúng n = k (Giải thiết quy nạp) 3) Chứng minh mệnh đềđúng với n = k + 1.

Nhận xét: Trong việc chứng minh phương pháp quy nạp bạn cần khai thác triệt để giả thiết quy nạp (là mệnh đề chia hết n = k), tức q trình giải tốn bước chứng n = k + bạn phải biến đổi xuất giả thiết quy nạp

* Ví dụ minh họa:

Bài toán Chứng minh n 2n 7( 2+ ) chia hết cho với mọi sốnguyêndương n.

Hướng dẫn giải

Với n = ta có: n 2n( +7)=1 7( + )=9 3 , tốn với n = 1 Giải sử toán đến n = k với k≥1,k N∈ tức là:

( ) ( ) ( *)

k 2k hay k 2k 7+  + =3x x N∈ , Ta sẽcần chứng minh toán với n = k + 1 Thật vậy:

(18)

( ) ( ) ( )( )

( ) ( )

( )

2 2

3 2

3

2

k k n 2n 4n

2n 4n 9n 2n 4n 2n 7n 6n 6n 3x 2n 2n 3y

 

+  + + = + + +

 

= + + + + +

= + + + +

= + + + = 

Do n 2n 7( + ) chia hết cho với n = k + 1 Vậy toán chứng minh

Bài toán Chứng minh 4n +15n−1 chia hết cho với n N∈ *

Hướng dẫn giải

Với n = ta có: A 18= chia hết cho 9, tốn với n = Giải sử toán đến n = k với k≥1,k N∈ tức là:

( )

2 *

4k 15k hay 4k 15k 9x x N+ −  + − = ∈ ⇔4k =9x 15k 1− + , ta sẽcần chứng minh toán với n = k + 1 Thật vậy:

( )

( )

k k

4 15 k 1 4.4 15k 14

4 9x 15k 15k 14 36x 45k 18

+ + + − = + +

= − + + +

= − + 

Do A 4n 15n 1= 2+ − chia hết cho với n = k + 1 Vậy toán chứng minh

Bài toán Chứng minh 52n+7 chia hết cho với số nguyên dương n.

Hướng dẫn giải

• Với n 1, ta có = 5 32 82 + =  (đúng)

• Giả sử mệnh đề với , tức ta có 52n+7 8

• Ta cần chứng minh mệnh đề với n Thật vậy, ta có +

( +) + = + = +( + )

2 n 2n 2n 2n

5 25.5 24.5

Để ý 52n+7 8 và 24.5 82n Do ta ( )+ +  n

5

Vậy theo nguyên lý quy nạp ta 52n+7 chia hết cho với số nguyên dương n.

Bài toán Cho n số nguyên dương, Chứng minh rằng: 2 ( )

7.2 − −

= n + n

C

(19)

Hướng dẫn giải

Xét với n = ta có: C =10 5 Vậy (1) với n =

Giả sử (1) với n = k (k≥1,k N∈ ), tức là: 2 ( )

7.2 − −

= k + k

k

C

Ta chứng minh (1) với n = k + 1, tức phải chứng minh: ( ) ( )

2 2 1

1 7.2

+ − + −

+ = k + k

k

C

Ta có: 2( 1) 2( 1) 2 2 2 2

1 7.2 7.2 3 4.7.2 9.3

+ − + − + − − − −

+ = k + k = k + k = k + k

k

C

( 2 1) 2

4 7.2 − − 5.3 − 5.3 −

= k + k + k = + k

k

C

Vậy theo nguyên lý quy nạp ta 2

7.2 − −

= n + n

C chia hết cho với số nguyên

dương n

Bài toán Chứng minh số tạo 3n chữ số giống chia hết cho3n với *

n N

(Đề thi học sinh giỏi lớp toàn quốc năm 1978) Hướng dẫn giải

Với n = 1, ta có: aaa=111.a3, Vậy toán với n =

Giả sử toán đến n = k (k≥1,k N∈ ), tức là: 

3 k

k

aa a

Ta chứng minh mệnh đề đến n = k +

Thật vậy:       

1 1

1

3 3 3 3

100 0100 01

+ − −

+

= = × 

k k k k k k k

n

aa a aa a aa a aa a aa a

Vậy toán chứng minh

Dạng 8: Sử dụng nguyên lý Dirichlet

* Cơ sở phương pháp: Đầu tiên ta phải nắm nguyên lý Dirichle: “Nhốt m = kn + 1

con thỏvào k (k < n) chuồng tồn chuồng có n + con thỏ”

Áp dụng nguyên lý Dirichle vào toán chia hết sau: “Trong m = kn + sốcó

n + sốchia hết cho k có sốdư” * Ví dụ minh họa:

Bài toán Chứng minh số ngun tìm ba số có tổng chia hết cho

Hướng dẫn giải

Một sốkhi chia cho tồn loại sốdư là: 1,

(20)

Trường hợp 1: Nếu tồn cả3 loại sốdư chia cho thì:

( )

1

2 2 3

3

3

3 3

3

= +

 = + ⇒ + + = + + +

 

= +



a k

a k a a a k k k

a k

Trường hợp 2: Chỉtồn hai loại sốdư, theo nguyên lý Dirichlet trong số ngun ln tồn số dư chia cho suy tổng số chia hết cho

Trường hợp 3: Chỉtồn tài du loại sốdư chia hết cho suy sốtùy sốđó chia hết cho

Bài toán Cho số nguyên phân biệt a, b, c, d Chứng minh rằng:

( )( )( )( )( )( ) 12

= − − − − − − 

A a b a c a d b c b d c d

Hướng dẫn giải

Theo nguyên lý Dirichlet trong sốnguyên tùy ý ln tồn hai sốngun tùy ý có sốdư chia hết cho suy A3

Trường hợp 1: cả4 sốđều sốchẵn nên tồn hiệu chia hết cho suy A4

Trường hợp 2: cả4 sốđều sốlẻnên tồn hiệu chia hết cho suy A4

Trường hợp 3: sốchẵn hai sốlẻ nên tồn hiệu chia hết cho suy A4

Trường hợp 4: sốchẵn sốlẻ, từ3 sốchẵn cho ta hiệu chia hết cho suy A4

Trường hợp 5: sốlẻvà sốlẻ, từ3 sốlẻđó cho ta hiệu chia hết cho suy A4

Do A chia hết cho mà (3, 4) = nên A chia hết cho 12

Bài toán Chứng minh 101 số ngun tìm hai số có chữ số tận giống

Hướng dẫn giải

Lấy 101 sốnguyên chia cho 100 theo ngun lý Dirichle có có sốcó sốdư chia cho 100 Suy 101 sốđã cho tồn sốcó chữ sốtận giống

Bài tốn Cho 2014 số tự nhiên x ,x ,x , ,x1 2 3 2014 Chứng minh tồn số chia hết cho 2014 tổng số sốchia hết cho 2014

Hướng dẫn giải

Xét 2014 số: S1 =x ;S1 2 =x x ; ;S1+ 2 2014 =x x x1+ 2+ + 2014

(21)

Nếu tồn Si với i = 1, 2, 3, …., 2014 chia hết cho 2014 tốn chứng minh Nếu khơng tồn Si với i = 1, 2, 3, …., 2014 chia hết cho 2014 Đem 2014 sốnày chia cho 2014 nhận 2014 số dư Giá trị số dư nhận thuộc vào tập hợp

{1,2,3, ,2013} Vì 2014 số dư mà chỉcó 2013 giá trịnên theo nguyên lý Dirichlet có số dư

Kí hiệu hai sốđó S ,Sm n có sốdư chia cho 2014 {m,n N,1 n m 2014∈ ≤ < ≤ }

Thì hiệu: Sm−Sn =xn 1+ +xn 2+ + + xmchia hết cho 2014

Nhận xét:Ta tổng qt hóa tốn sau: Cho n số tự nhiên x ; x ; ; x1 2 n Chứng minh rằng n số có số chia hết cho n số số có tổng chia hết cho n.

Dạng 9: Xét đồng dư Tóm tắt lý thuyết đồng dư:

Định nghĩa: Cho a, blà sốnguyên (nlà sốnguyên dương) Ta nói ađồng dư với b

theo modunnvà kí hiệu a b≡ (mod n)nếu abcó sốdư chia cho n.

Như vậy: a b≡ (mod n) (⇔ −a b n) .Ví dụ: 2019 mod 5≡ ( )

Một số tính chất bản:

1) Với sốnguyên a ta có: a a≡ (mod n)

2) a b≡ (mod n)⇔ ≡b a(mod n)

3) a b≡ (mod n) b c≡ (mod n)⇒ ≡a c(mod n)

4) a b≡ (mod n) c d≡ (mod n) (⇒ ± ≡ ±a c) (b d)(mod n)

Hệ tính chất 4)

( ) ( ) ( )

( ) ( )

1 2

1 2

mod , mod , , mod

a mod

n n

n n

a b n a b n a b n

a a b b b n

≡ ≡ ≡

⇒ + + + ≡ + + +

5) a b≡ (mod n) c d≡ (mod n)⇒a c b d ≡ mod( n)

Hệquảcủa tính chất 5)

( ) ( ) ( )

( ) ( )

1 2

1 2

mod , mod , , mod

a mod

n n

n n

a b n a b n a b n

a a b b b n

≡ ≡ ≡

⇒ ≡

6) a b≡ (mod m)⇔anbn(mod m)∀ ∈n N

(22)

7) Nếu a b≡ (mod m) d là ước chung absao cho (d, m) = 1thì

(mod )

a b m

d d

8) Nếu a b≡ (mod m) d là ước chung a, b, mthìa b modm

d d d

 

≡  

 

9) Nếu a r≡ (mod m) 0≤ <r m,thì rchính sốdư phép chia a cho m.

* Cơ sở phương pháp: Sửdụng định nghĩa tính chất đồng dư thức đểgiải tốn chia hết

* Ví dụ minh họa:

Bài toán Chứng minh rằng: A = 7.52n +12.6nchia hết cho 19

Hướng dẫn giải

Ta có: A=7.52n+12.6n =7.25 12.6n+ n

( ) ( ) ( )

≡ ⇒ ≡ ⇒ ≡

25 mod 19 25n mod 19n 7.25n 7.6 mod 19n

Do

( )

⇒7.25 12.6n+ n ≡7.6 12.6 mod 19n+ n

Mà 7.6 12.6 19.6 19n+ n= n ⇒7.25 12.6 19n+ n ⇒ =A 7.52n+12.6 19n Vậy toán chứng minh

Bài toán Chứng hai số: A=61000−1 và B=61001+1 Chứng minh A B bội sốcủa

Hướng dẫn giải

Ta có: 6≡ −1 mod 7( )⇒61000 ≡ −( ) (1 1000 mod 7)⇒61000 ≡1 mod 7( )⇒61000−1 7 Vậy A bội

Từ 61000 ≡1 mod 7( )⇒61001 ≡6 mod 7( )

Mà 6≡ −1 mod 7( )⇒61001≡ −1 mod 7( )⇒61001+1 7

Vậy B bội

Bài toán a) A = 22225555+ 55552222chia hết cho b) 1962 1964 1966

1961 1963 1965

B= + + + chia hết cho

(23)

Hướng dẫn giải

a) Xét số dư 22225555 khi chia cho 7.

Ta có: 2222 ≡3 (mod 7) (1) => 22224≡34(mod 7)

=> 22224≡81 (mod 7) Mà 81 ≡4 (mod 7)

=> 22224≡4 (mod 7) (2)

Nhân vế với vế (1) (2) ta 22225≡3.4 (mod 7) => 22225≡5 (mod 7) =>22225555≡51111(mod 7) (3)

+ Tương tự: 55552222≡21111(mod 7) (4) Cộng vế với vế (3) (4) ta có: A ≡21111+ 51111(mod 7) (5) Mặt khác: 21111+ 51111≡(2 + 5) (mod 7)

≡0 (mod 7) (6)

Từ (5) (6) ta được: A ≡0 (mod 7) Vậy: A = 22225555+ 55552222chia hết cho 7 b) Ta có:

Ta có: 1961≡1 (mod 7) => 19611962 ≡1 (mod 7)

Tương tự: 1964 1964( ) ( )3 654( ) 654( ) ( )

1963 ≡3 mod ≡9 mod ≡9.27 mod ≡2 mod

1966 ( ) (1966 ) ( )3 655( ) 655( ) ( )

1965 ≡ −2 mod ≡2 mod ≡2.8 mod ≡2 mod

( ) ( )

1 2 mod mod

B

⇒ ≡ + + + ≡

Vậy: 1962 1964 1966

1961 1963 1965

B= + + + 

Bài tốn Tìm số dư phép chia: 1532 15− cho 9.

Hướng dẫn giải

Ta có: 1532 mod9≡ ( )⇒15325 ≡2 mod 95( )

Mà 25 ≡5 mod9( )⇒15325 ≡5 mod 9( )⇒1532 mod 95− ≡ ( )

Vậy số dư phép chia 1532 15− cho 4.

(24)

Dạng 10: Tìm điều kiện biến để chia hết Bài tốn

a) Tìm n ngun để

2

= + − +

A n n n chia hết cho B=n2−n

b) Tìm anguyên để a 2a 7a 73− 2+ − chia hết cho a 32+

Hướng dẫn giải

a) Chia A cho B ta có: ( )( )

2 3

+ − + = + − +

n n n n n n

Để A chia hết cho B phải chia hết cho ( )

1

− = −

n n n n chia hết cho n, ta có:

n -1 -2

n - -2 -3

n(n – 1) 2

loại loại

Vậy đểgiá trịbiểu thức

2

= + − +

A n n n chia hết cho giá trịbiểu thức B=n2−nthì n = -1

hoặc n =

b) Thực phép chia a 2a 7a 73− 2+ − cho a 32+ được kết quả:

( )( ) ( )

3 2

a 2a 7a 7− + − = a +3 a 2− + 4a 1−

Đểphép chia hết 4a 1− phải chia hết cho a 32+

( ) ( )

( )( ) ( )

( ) ( )

− +

⇒ − + + ∈ ⇒ + ∈

⇒ − +

  

2

2

4a a

4a 4a a (a 4a ) 16a a

( )

( )

⇒ +

 + =  =

⇒ + = ⇒ = −

 

 2

49 a

a a a 49 loai a

Thửlại ta a = a = - thỏa mãn

Bài toán Tìm sốtựnhiên n để n2+(n 1+ ) (2 + n 2+ ) (2+ n 3+ )3chia hết cho 10.

Hướng dẫn giải

Ta có: A n= 2+(n 1+ ) (2+ n 2+ ) (2+ n 3+ )3 =2 2n 6n 7( 2+ + )

( )

⇔ + + ⇔ + + ⇔ + +

⇔ + +

   

2 2

2

A 10 2n 6n 2n 6n n 3n n 3n

(25)

Do n(n + 3) có tận hay n có tận Vậy n có tận thỏa mãn yêu cầu toán

Bài toán Tìm sốnguyên dương n để(n + 3)(n + 4) chia hết cho 3n

Hướng dẫn giải

Ta có: (n n 3n+ )( + ) ⇔n 7n 12 3n2+ +  ⇔n2 + +n 12 3n

( ) ( )

 + + 

⇒ 

+ + 

 

2

n n 12 n n n 12

Từ(1) suy ra: 12 n ⇒ ∈n 1,2,3,4,6,12{ } ( )3

Từ(2) suy ra: ( + ) ⇒ = += ( ∈ ) 

 n 3k

n n k N

n 3k Loại sốcó dạng n = 3k + ở(1) ta được:

n 12

3n 18 36

n2+ n + 12 18 24 54 168

Chỉcó n = n = n n 12 ch2+ + ia hết cho 3n đó: (n n 3n+ )( + )

Vậy n = n =

Bài tốn Tìm sốngun dương x y lớn cho x + chia hết cho y y + chia hết cho x

Hướng dẫn giải

Giải sử2 ≤ x ≤ y

a) Xét y = x = 2, không thỏa mãnx + chia hết cho y

b) Xét y ≥ Đặt x + = ky (kN) (1) ky = x + ≤ y + ≤ y + y = 2y nên k ≤ 2.

Với k = 1, từ (1) có x + = y Thay vào: y+3x x+6xnên lại có x > nên

{2;3; }

x

x

y

Với k = 2, từ (1) có x + = 2y Thay vào: y+3x 2y+6x⇒ +x 9x⇒9x

do x > nên x∈{ }3;9

(26)

Khi x = y = 3, thửlại

Khi x = y = 6, loại trái với x≤ y

Các cặp số(x, y) phải tìm (2; 5), (5; 2), (3; 6), (6; 3), (6; 9), (9; 6), (3; 3)

TỔNG KẾT CÁC PHƯƠNG PHÁP THƯỜNG ÁP DỤNG

Để làm giải tốt toán vềchia hết, cần sửdụng linh hoạt phương pháp nêu trên, ởnhiều tốn chia hết giải nhiều phương pháp, có tốn nhìn tương tựnhư có phương pháp giải Để mơ điều tơi trích viết tác giả Nguyễn Đức Tấn tạp chí Tốn học tuổi trẻ:

Bài toán: Chứng minh với sốngun n thì n n2+ +1 khơng chia hết cho 9. Cách 1: (Sửdụng phương pháp xét sốdư)

Ta xét trường hợp sau:

- Trường hợp 1: n = 3k(k Z∈ ) n2 + + =n 3k k 1( + +) - Trường hợp 2: n = 3k + (k Z∈ )thì n2+ + =n 9k k 3( + +) - Trường hợp 3: n = 3k + (k Z∈ )thì n2+ + =n 3k( 2+5k 1+ )+

Từ3 trường hợp suy n n2+ +1 không chia hết cho với mọi sốnguyên n Cách 2: (Sửdụng phương pháp tách tổng)

Ta có: n n2+ + =1 (n−1)(n+ +2 3)

Do (n + 2) – (n – 1) = nên (n + 2) (n – 1) đồng thời không đồng thời chia hết cho Nếu (n+2 3;) ( n−1 3) ⇒(n−1)(n+2 9) nên (n−1)(n+2 3)+ sẽkhông chia hết cho

Nếu (n + 2) (n – 1) đềkhơng chia hết cho (n−1)(n+2 3)+ sẽkhông chia hết cho Vậy n n2 + +1 không chia hết cho với mọi sốnguyên n

Cách 3: (Sửdụng phương pháp phản chứng)

Giả sử (n n2+ +1 9.) Đặt n n2+ + =1 9m m Z( ∈ )⇒n n2 + + −1 9m=0 *( ) Phương trình (*) có ∆ =36m− =3 12( m−1)

Ta thấy ∆ sốchính phương chỉchia hết cho mà khơng chia hết (*) khơng có nghiệm Vô lý!

Vậy n n2 + +1 không chia hết cho với mọi sốnguyên n

(27)

Cách 4: Ta có: 4(n n2+ + =1) (2n+1)2+3

Nếu (2n+1 3) ⇒(2n+1 9)2 nên (2n+1)2+3sẽkhông chia hết cho

Nếu (2n + 1) không chia hết cho (2n+1)2 khơng chia hết (2n+1)2 +3sẽ khơng chia hết cho sẽkhông chia hết cho

Vậy 4(n n2 + +1) không chia hết n n2+ +1sẽkhông chia hết cho với mọi số nguyên n.

Các bạn rèn luyện khảnăng sửdụng phương pháp chứng minh toán chia hết thơng qua tốn tương tựsau:

1) Chứng minh: n2+11n+39 không chia hết cho 49. 2) Chứng minh: n2+3n+5 không chia hết cho 49. 3) Chứng minh: n2+5n+16 không chia hết cho 169. Tuy nhiên với tốn:

Chứng minh: 9n3+9n2+3n−16 khơng chia hết cho 343 với mọi sốnguyên n.

Ta dễthấy với cách 1, 2, có lẽchúng taphải bó tay, khai thác giải ý 343 7=

ta có lời giải thật “dễthương” sau:

( )3

3

9n +9n +3n−16= 3n+1 −49

Nếu (3n+1 7) ⇒(3n+1 7)3 =343 nên (3n+1)3−49sẽkhông chia hết cho 343.

Nếu (3n + 1) khơng chia hết cho (3n+1)3−49 khơng chia hết (3n+1)3−49

không chia hết cho 343 7= 3

Vậy 9n3+9n2 +3n−16sẽkhông chia hết cho 343 với mọi sốnguyên n.

Do để giỏi toán cần linh hoạt nắm vững phương pháp giải đểcó thể vận dụng tốt ở các toán khác nhau!

C BÀI TẬP ÁP DỤNG

Câu Chứng minh a a 30 a5−  ( ∈)

Câu a) Đặt A n 3n 5n 3.= 3+ 2+ + Chứng minh Achia hết cho với giátrị nguyên dương n

b) Nếu achia 13 dư 2và b chia 13 dư a b2+ 2chia hết cho 13 Câu Chứng minh rằng: A=n n 73( 2− )2−36n 7

 

  với ∀ ∈n 

Câu Chứng minh n 28n3− chia hết cho 48 với mọi nlà sốnguyên chẵn Câu Cho nlà sốtựnhiên lẻ Chứng minh n n3− chia hết cho 24

(28)

Câu Chứng minh n 17n3+ chia hết cho 6 với mọi n∈

Câu Chứng minh rằng: Q n= 3+(n 1+ ) (3+ n 9+ )3 với mọi n∈* Câu Chứng minh : 20092008+20112010chia hết cho 2010

Câu Chứng minh

a) 8 25+ 11chia hết cho 17 b) 1919 +6919chia hết cho 44

Câu 10 Chứng minh A n n 2= 2+ + không chia hết cho 15 với mọi sốnguyên n.

(Đềthi HSG lớp huyện ThủyNguyên 2018-2019)

Câu 11.Chứng minh với n N∈ thì: n 6n 11n4+ 3+ 2+30n 24− chia hết cho 24.

(Đềthi HSG lớp huyện Thanh Hà 2016-2017)

Câu 12 Cho a, b sốnguyên thỏa mãn: 2a 3ab 2b2+ + 2 chia hết cho Chứng minh 2

a b− chia hết cho 7.

(Đềthi HSG lớp huyện Kinh Môn 2013-2013)

Câu 13 Cho n sốnguyên không chia hết cho Chứng minh P=32n+ +3n chia hết cho

13

(Đềthi HSG lớp huyện Vũ Quang 2018-2019) Câu 14 Cho biểu thức P a a a a= 1+ 2+ 3+ + 2019 với a ;a ;a ; ;a1 2 3 2019 sốnguyên

dương P chia hết cho 30 Chứng minh 5 5

1 2019

Q a= +a +a + + a chia hết cho 30

(Đềthi HSG Thành PhốHải Phòng 2018-2019) Câu 15. Cho x sốtựnhiên chẵn Chứng tỏrằng biểu thức M=x3 +x2 + x

24 12 có giá trịlà sốnguyên

(Đềthi Chọn HSG lớp THCS Hiệp An 2018-2019)

Câu 16.Chứng minh với sốtựnhiên n ta có: A = 7.52n+ 12.6n chia hếtcho 19

(Đềthi HSG lớp huyện Phù Ninh 2013-2014) Câu 17 Chứng minh với sốtựnhiên nthì : A 5= n 2+ +26.5 8n+ 2n 1+ 59

Câu 18 Cho a ,a , ,a1 2 2016là sốtựnhiên có tổng chia hết cho Chứng minh rằng: 3

1 2016

A a= +a + a+ chia hết cho

Câu 19 a) Chứng minh tổng hai sốnguyên chia hết cho tổng lập phương chúng chia hết cho

b) Tìm sốnguyên n để n 15+ chia hết cho n 13+

Câu 20 Tìm tất cảcác cặp sốnguyên (x ; y) với x > 1, y > cho

(29)

(3x + 1)  y đồng thời (3y + 1)  x

(Đềthi học sinh giỏi lớp huyện Hoằng Hóa năm 2014-2015)

Câu 21 Tìm sốnguyên dương n bé đểF = n3+ 4n2 – 20n – 48 chia hết cho 125

(Đềthi HSG lớp huyện Hoằng Hóa 2015-2016)

Câu 22.Tìm tất cảcác cặp sốnguyên dương a,b cho: a + b2chia hết cho a2b – 1.

(đềthi học sinh giỏi lớp huyện Thanh Oai 2012-2013)

Câu 23 Cho sốnguyên dương x, y, z thỏa mãn x y2+ =z2 Chứng minh A = xy chia hết cho 12

(Đềthi HSG lớp huyện Vĩnh Lộc 2016-2018)

Câu 24 Chứng minh sốtựnhiên A 1.2.3 2017.2018 1 1 2017 2018

 

=  + + + + + 

 

chia hết cho 2019

(Đềthi HSG lớp huyện Hoài Nhon 2018-2019) Câu 25 Tìm sốdư phép chia đa thức (x x x x 2010+ )( + )( + )( + +) cho đa thức x 10x 212+ +

Câu 26 Tìm a,b cho f(x) ax= 3+bx 10x 42+ − chia hết cho đa thứcg(x) x= + −x 2 Câu 27 Cho đa thức Với giá trịnguyên giá trịcủa đa thức chia hết cho giá trịcủa đa thức

Câu 28 Giả sửf(x) đa thức bậc với hệ sốnguyên

Chứng minh rằng: Nếuf(x) 7với ∀ ∈ Ζx hệ sốcủa f(x) cũng7

(Đềthi học sinh giỏi lớp trường Trần Mai Ninh năm 2012-2013)

Câu 29 Tìm sốdư phép chia (x x x x 2033+ )( + )( + )( + +) cho x 12x 302+ + Câu 30 Tìm đa thức f(x) biết : f(x) chia cho x 2+ dư 10, f x( )chia cho x 2− dư 26,

( )

f x chia cho x 42− được thương −5xvà dư

Câu 31 Cho đa thức P(x) = ax3+ bx2+ cx + d với a, b, c, d hệ sốnguyên Chứng minh P(x) chia hết cho với giá trịnguyên x hệ sốa, b, c, d chia hết cho

(đềthi học sinh giỏi lớp huyện Thạch Hà 2016-2017)

Câu 32 Cho p sốnguyên tốlớn Chứng minh p20−1 chia hết cho 100

(Đềthi HSG lớp huyện Lục Nam 2018-2019) Câu 33. Cho a, b, c sốnguyên khác thỏa mãn điều kiện:

 

+ + = + +

 

 

2

2 2

1 1 1 .

a b c a b c

3

f(x) x 3x= − +3x 4.− x

f(x) x2+2

(30)

Chứng minh rằng: a3+b c3+ chia hết cho 3.

(Đềthi HSG lớp TP Thanh Hóa 2016-2017)

Câu 34 Cho N = k4+ k3 – 16 k2 – 2k +15, k sốnguyên Tìm điều kiện k để sốN chia hết cho 16

(Đềthi HSG huyện Lê Ninh 2018-2019)

Câu 35 Cho hai sốnguyên, sốthứnhất chia cho dư 1, sốthứhai chia cho dư Hỏi tổng bình phương chúng có chia hết cho không ?

Câu 36 Chứng minh với số nguyên dương thì:

Câu 37 Chứng minh A = + + 32+ 33 + + 311 chia hết cho 40.

Câu 38 Tìm đa thức biết: chia cho dư 5; chia cho dư 7; chia cho thương đa thức dư bậc với

Câu 39 Cho sốtựnhiên Chứng minh tích

chia hết cho

Câu 40. Cho a b, sốnguyên dương thỏa mãn p=a2+b2 sốnguyên tốvà p−5

chia hết cho Giả sửcác sốnguyên x y, thỏa mãn ax2−by2 chia hết cho p Chứng minh

rằng hai số x y, chia hết cho p

(Đềthi HSG lớp TP Hải Phòng 2017-2018)

Câu 41. Cho ba sốnguyên dương a b c, , thỏa mãn a3+b3+c3 chia hết cho 14 Chứng

minh abc chia hết cho 14

(Trích đềChun tốn Sư Phạm Hà Nội 2019-2020)

Câu 42.

a) Tìm tất cảnhững sốtựnhiên n cho 2n+1 chia hết cho

b) Cho n sốtự nhiên n>3 Chứng minh 2n+1 không chia hết cho 2m −1 với sốtựnhiên m cho 2< ≤m n

(Trích đềPhổThơng khiếu HồChí Minh 2019-2020) Câu 43. Chứng minh với sốnguyên dương n, số 4

9.3 8.2 2019

= nn+

M chia

hết cho 20

(Trích đềChuyên Quảng Nam 2019-2020) Câu 44. Có sốtựnhiên n không vượt 2019 thỏa mãn n3+2019 chia hết cho

n 5 5n( n+ −1 3) (n n+2 91n)

( )

f x f x( ) x−2 f x( ) x−3 f x( )

(x−2)(x−3) x2 −1 x

3.

n> 2n =10a+b a b( , ∈,0< <b 10)

ab

(31)

(Trích đềChuyên Nam Định 2018-2019)

Câu 45. Cho x y, sốnguyên cho 2

2 ;

xxyy xyyx chia hết cho

Chứng minh 2

2x +y +2x+ycũng chia hết cho

(Trích đềChuyên KHTN Hà Nội 2018-2019)

Câu 46.Tìm tất cảcác sốngun khơng âm a b c, , thỏa mãn

( ) (2 ) (2 )2

6

ab + bc + ca = abc a3 +b3+ +c3 1 chia hết cho a+ + +b c 1.

(Trích đềChuyên Nam Định 2016-2017) Câu 47. Cho n sô tựnhiên chẵn, chứng minh số n n n

20 −3 +16 −1chia hết cho số323

(Trích đềChun Bình Định 2018-2019)

Câu 48. Cho n sốnguyên dương tùy ý, với sốnguyên kta đặt Sk = +1k 2k + +nk Chứng minh S2019S1

(Trích đềChuyên Lam Sơn 2018-2019)

Câu 49. Cho phương trình 3

2 9!(1)

x + y + z = với x y z; ; ẩn 9! Là tích sốnguyên

dương liên tiếp từ1 đến

a) Chứng minh có sốnguyên x y z; ; thỏa mãn (1) x y z, , chia hết cho

b) Chứng minh không tồn sốnguyên x y z, , thỏa mãn (1)

(Trích đềChuyên Vĩnh Phúc 2018-2019) Câu 50. Cho plà sốnguyên tốlớn Chứng minh p2−1chia hết cho 24

(Trích đềChuyên Bến Tre 2018-2019)

Câu 51. Cho sốtựnhiên n≥2và sốnguyên tố pthỏa mãn p−1chia hết cho nđồng thời

3

1

n − chia hết cho p Chứng minh n+ plà sốchính phương

(Trích đềChuyên Phan Bội Châu 2018-2019)

Câu 52. Với n sốtựnhiên chẵn, chứng minh rằng: (20n+16n− −3n 323)

(Trích đềChuyên Lâm Đồng 2018-2019)

Câu 53. Đặt N = +a1 a2+ +a2018,

5 5

1 2018

M =a +a + +a (a a1; 2; a2018∈+) Chứng

mỉnh N chia hết cho 30 M chia hết cho 30

(Trích đềChuyên Hải Dương 2018-2019)

Câu 54. Cho a, b,c sốnguyên Chứng minh 2016 2017 2018

a +b +c chia hết cho

2018 2019 2020

a +b +c chia hết cho 6.

(32)

(Trích đềChuyên Tuyên Quang 2018-2019)

Câu 55. Tìm dạng tổng quát sốnguyên dương nbiết: M= n.4n+ 3nchia hết cho

(Trích đềChuyên Hải Dương 2016-2017)

Câu 56. Chứng minh với sốtựnhiên n thì

9 27

− +

n n không chia hết cho 81

(Trích đềChuyên Quảng Ngãi 2018-2019)

Câu 57. Cho m n, sốnguyên thỏa mãn 4(m+n)2−mnchia hết cho 225 Chứng minh

rằng: mncũng chia hết cho 225

(Trích đềChuyên Lào Cai 2018-2019)

Câu 58 Cho n số nguyên dương tùy ý, với số nguyên dương k đặt

1k 2k k k

S = + + +n Chứng minh S2019S1

(Chun tốn Thanh Hóa 2018-2019)

Câu 59. Chứng minh p (p + 2) hai sốnguyên tốlớn tổng chúng chia hết cho 12

(Trích đềChun Hịa Bình 2015-2016)

Câu 60. Chứng minh sốnguyên n lớn thoảmãnn2+4 n2+16là số nguyên tốthì n chia hết cho 5.

(Trích đềChuyên Phú Thọ2015-2016) Câu 61. Chứng minh biểu thức 3( ) (2 )( )

2

S =n n+ + n+ nn+ − n− chia hết cho 120,

với n số ngun

(Trích đềChun Bình Phước 2017-2018)

Câu 62. Cho ( 2015 2015 2015)

2

A= + + +n với n sốnguyên dương Chứng minh A

chia hết cho n(n + 1)

(Trích đềChuyên Quảng Nam 2015-2016)

Câu 63. Cho biểu thức

2 16 15

Q=a + aaa+ Tìm tất giá trị nguyên ađể Q

chia hết cho 16

(Trích đềChuyên Quảng Nam 2016-2017)

Câu 64 Chứng minh ba số phương tùy ý tồn hai số mà hiệu chúng chia hết cho

Câu 65. Cho a, b, c ba số nguyên khác thỏa 1 1= +

a b c Chứng minh rằng: abc chia hết

cho

(33)

(Trích đềthi HSG lớp tỉnh Đồng Nai 2019)

Câu 66. Chứng minh

2 n 4n 16

A= + + chia hết cho với sốnguyên dương n.

(Trích đềthi HSG lớp tỉnh NghệAn Bảng A 2019)

Câu 67.Chứng minh A=4n+17 chia hết cho với sốnguyên dương n.

(Trích đềthi HSG lớp tỉnh NghệAn Bảng B 2019) Câu 68. Cho *

nN Chứng minh 2n+ 3n+ sốchính phương n

chia hết cho 40

(Trích đềthi HSG lớp tỉnh Thanh Hóa 2019)

Câu 69.Chứng minh với sốnguyên n chẵn thì: n3 +20n 96+ chia hết cho 48

(Trích đềthi HSG lớp tỉnh Bình Phước 2019) Câu 70. Cho plà sốnguyên tốthỏa mãn p a b= 3− với

a,b hai sốnguyên dương

phân biệt Chứng minh lấy 4p chia cho loại bỏphần dư nhận sốlà bình phương sốngun lẻ

(Trích đềthi HSG lớp tỉnh Khánh Hòa 2018)

Câu 71.Cho a b c, , ba sốnguyên khác thỏa 1

a = +b c Chứng minh rằng: abc chia hết

cho

(Trích đềthi HSG lớp tỉnh Đồng Nai 2019) Câu 72.1 Cho p số nguyên tố lớn Chứng minh p2016 – chia hết cho 60.

2 Cho x y z, , số dương khác đôi x3+y3 +z3chia hết cho x y z2 2

Tìm thương phép chiax3 +y3 +z : x y z3 2

(Trích đềthi HSG lớp tỉnh Thanh Hóa 2017) Câu 73. Cho hai sốnguyên a b thỏa 24a b 2+ = Chứng minh có số a

hoặc b chia hết cho 5.

(Trích đềthi HSG lớp tỉnh Quảng Nam 2017)

Câu 74.Cho p q sốnguyên tốlớn thỏa mãn p q 2= + Tìm sốdư chia p q+ cho 12

(Trích đềthi HSG lớp tỉnh Vĩnh Long 2016) Câu 75.Cho nguyên a, b, c, d thỏa mãn điều kiện a3+b3 =2 c 8d( 2− 3)

(34)

Chứng minh a b c d+ + + chia hết cho

(Trích đềthi HSG lớp Thành PhốHà Nội 2016) Câu 76 Chứng minh với số nguyên n, số

3 15

Ann chia hết cho 18

(Trích đềthi HSG lớp tỉnh Gia Lai 2019) Câu 77 Biết a b; sốnguyên dương thỏa mãn a2−ab b+ chia hết cho 9, chứng minh

rằng a b chia hết cho

(Trích đềthi HSG lớp Thành PhốHà Nội 2019)

Câu 78 Chứng minh 3 3

1 n

a +a +a + +a chia hết cho 3, biết a a a1, 2, 3, ,an chữ

sốcủa 2018

2019

(Trích đềthi HSG lớp tỉnh Hải Dương 2019) Câu 79 Cho nlà sốtựnhiên lẻ Chứng minh: 46n +296.13n chia hết cho 1947

(Trích đềthi HSG lớp tỉnh Bà Rịa Vũng Tàu 2019) Câu 80 Chứng minh

2n +3n +n chia hết cho 6với sốnguyên n

(Trích đềthi HSG lớp tỉnh Lâm Đồng 2019) Câu 81 Cho a b c, , số nguyên thỏa mãn

2018

a b+ = −c c Chứng minh

3 3

A=a + +b c chia hết cho

(Trích đềthi HSG lớp tỉnh Quảng Ngãi 2019) Câu 82 Chứng minh sốcó dạng 20142014 2014 có sốchia hết cho 2013

(Trích đềvào 10 Chun Lạng Sơn năm 2013-2014) Câu 83 Cho a b, hai số nguyên dương thỏa mãn a+20 b+13 chia hết cho 21

Tìm số dư phép chia A=4a +9b+ +a b cho 21

(Trích đềvào 10 Chuyên Hải Phòng năm 2013-2014) Câu 84 Cho biểu thức: ( 2020 2020 2020) ( 2016 2016 2016)

A= a +b +ca +b +c với a,b,c số

nguyên dương Chứng minh A chia hết cho 30

(Trích đềvào 10 Chuyên Tin Lam Sơn năm 2019-2020)

Câu 85 Cho hai số nguyên dương x y, với x>1 thỏa mãn điều kiện: 2x2− =1 y15

Chứng minh x chia hết cho 15

(Trích đềvào 10 Chuyên Toán Lam Sơn năm 2019-2020) Câu 86 Cho số1;2; 3; ; 100 Viết cách tùy ý100số nối hàng ngang ta số tự nhiên Hỏi số tự nhiên có chia hết cho2016hay khơng?

(Trích đềvào 10 Chun Tốn Lam Sơn năm 2015-2016)

Câu 87 Tìm k để tồn số tự nhiên n cho (n2−k 4) với k∈{0;1; 2; 3}

(35)

Câu 88.Cho n số dương Chứng minh rằng: (n n 2n+ )( + ) ( ) chia hết cho 2n

Chương II

CÁC BÀI TỐN VỀ SỐ CHÍNH PHƯƠNG

B. KiÕn thøc cÇn nhí

1 Định nghĩa số phương.

Số phương số bình phương số nguyên (tức n số phương thì: = ( ∈ )

n k k Z )

2 Một số tính chất cần nhớ

1- Số phương có chữ số tận 0, 1, 4, 5, 6, 9; khơng thể có chữ tận 2, 3, 7,

2- Khi phân tích thừa số nguyên tố, số phương chứa thừa số nguyên tố với số mũ chẵn

3- Số phương có hai dạng 4n 4n + Khơng có số phương có dạng 4n + 4n + (n ∈ N).

4- Số phương có hai dạng 3n 3n + Khơng có số phương có dạng 3n + ( n ∈N ).

5- Số phương tận 1, chữ số hàng chục chữ số chẵn Số phương tận chữ số hàng chục

Số phương tận chữ số hàng chục chữ số lẻ 6- Số phương chia hết cho chia hết cho

Số phương chia hết cho chia hết cho Số phương chia hết cho chia hết cho 25 Số phương chia hết cho chia hết cho 16 Mọi số phương chia cho 5, cho dư 1, 0,

8 Giữa hai số phương liên tiếp khơng có số phương

9 Nếu hai số ngun liên tiếp có tích số phương hai số số 10 Số ước số phương số lẻ Ngược lại, số có số ước số lẻ số số phương

11 Nếun2< k < (n+1)2( n ∈ Z) k khơng số phương

(36)

12 Nếu hai số tự nhiên a b nguyên tố có tích số phương số a, b số phương

13 Nếu a sốchính phương, a chia hết cho sốnguyên tố p a chia hết cho p2

14 Nếu tích hai số a b số phương số a b có dạng

2

;

 

a mp b mq

B CÁC DẠNG TOÁN THƯỜNG GẶP

Dạng 1: Chứng minh số số phương, tổng nhiều số phương

* Cơ sở phương pháp:

Để chứng minh số n số số phương ta thường dựa vào định nghĩa, tứclà chứng minh: = ( ∈ )

n k k Z

* Ví dụ minh họa:

Bài toán Cho n sốtựnhiên Chứng minh rằng: An n 1n2n 3 số

chính phương

Hướng dẫn giải

Ta có:

 2  2   2 2  2   2 2

3 3 3

            

A n n n n n n n n n n

n nên n23n 1  Vậy A sốchính phương

Bài tốn 2.Cho: B1.2.32.3.4  k k 1k2với k sốtựnhiên Chứng minh

rằng 4B + sốchính phương

Hướng dẫn giải

Ta thấy biểu thức B tổng biểu thức nghĩ đến việc phải thu gọn biểu thức B trước

Ta có:

 1 2  1 2  3  1  1 2 3  1  1 2

4

n nn  n nn  n  n  n nnn  n n nn 

Áp dụng:

 

1

1.2.3 1.2.3.4 0.1.2.3

 

(37)

 

 

           

1

2.3.4 2.3.4.5 1.2.3.4

1

3.4.5 3.4.5.6 2.3.4.5

1 2 1

4

k k k k k k k k k k k

 

 

 

           

Cộng theo vếcác đẳng thức ta được:

      

   

1

1.2.3 2.3.4 2

4

4 1

B k k k k k k k

B k k k k

         

      

Theo ví dụ ta có:  2 2

4B 1 k 3k1

k nên k2 3k 1  Vậy 4B1là sốchính phương

Bài toán 3. Chứng minh rằng:  

2

11 44 n n

C   với n sốtựnhiên Chứng minh Clà sốchính phương

Hướng dẫn giải

Ta có: 11 100 0   11 44

n n

n n

C   

Đặt 11 1

n

a= 99 9

n

a= Do 99 10 n

n a + = = +      2

.10

9

33 n

n

C a a a a a a

C a a a

C

       

     

 

Vậy Clà số phương

Nhận xét:

Khi biến đổi số có nhiều chữ số giống thành số phương ta nên đặt 11 1

n a

= 99 10 n

n

a

+ = = + .

Bài toán Cho

2016

11

a= , 

2015

10

b= Chứng minh ab+1 số tự nhiên

Hướng dẫn giải Cách 1:

(38)

Ta có:   

2015 2016 2016

10 10 9

b= = − + = + = a+ .

⇒ ab + = a(9a + 6) + = 9a2+ 6a + = (3a + 1)2

ab+1= (3a+1)2 =3a+1∈N

Vậy ab+1 số tự nhiên

Cách 2:

Ta có:  2016 2016 2016

10

11 , 10

9

a= = − b= + .

( ) ( 2016)2 2016

2016

2016 10 4.10

10

1 10

9

ab − + − +

⇒ + = + + = 2016 10  +  =     .

( 2016 )

10

1

3

ab +

⇒ + =

Mà ( 2016 )

10 +2 3 Do đó, ab+1 số tự nhiên

Vậy ab+1 số tự nhiên

Bài toán Cho số tự nhiên agồm 60 chữ số 1, số tự nhiên bgồm 30 chữ số Chứng minh

a - blà số phương

Hướng dẫn giải Cách 1:

Ta có:  60

60

10

11

a= = − , 

30 30

10

22 2

b= = −

60 30 60 30

10 2(10 1) 10 2.10

9 9

a b − − − +

⇒ − = − =  2 30 30 10 33 3  −    =  =      . Cách 2:   30 30 22 2.11

b= = ,    

60 30 30 30

11 11 1.00 11

a= = +  30 

30 30

11 1.10 11

= +

Đặt 

30

11

c=  30

30

9c 99 10

⇒ + = + =

Khi đó: ( )

9

a=c c+ + =c c + c b=2c

( )2 

2

30

9 2 33

a b c c c c  

⇒ − = + − = =  

 

(39)

Bài toán tổng quát: Cho k số tự nhiên khác 0, số tự nhiên agồm 2kchữ số số tự nhiên

bgồm kchữ số Chứng minhrằng a b− số phương

Bài toán Cho n∈ cho

2

1

n

là tích hai sốtựnhiên liên tiếp Chứng minh

n tổng hai sốchính phương liên tiếp

Hướng dẫn giải

Giả sửta có:

3

n

=a a( +1)

Từđó có 2

3

n = a + a+ ⇒ 4n2− =1 12a2+12a+3

⇒ ( )( ) ( )2

2n−1 2n+ =1 2a+1

Vì 2n+1; 2n−1là hai sốlẻliên tiếp nên ta có trường hợp:

Trường hợp 1: 32

2

n p

n q

 − = 

+ = 

Khi 2

3

q = p + ( Vơ lí ) Vậy trường hợp không xảy

Trường hợp 2: 22

2

n p

n q

 − = 

+ = 

Từđó p sốlẻnên p=2k+1 Từđó ( )2

2n= 2k+1 +1 ⇒ n=k2+(k+1)2 (đpcm)

Bài toán Cho k sốnguyên dương a3k23k1

a) Chứng minh 2a a2 tổng ba sốchính phương

b) Chứng minh a ước sốnguyên duong b b tổng gồm

ba sốchính phương n

b tổng bà sốchính phương

Hướng dẫn giải

a) Ta có 2  2  2 2

2a6k 6k 2 2k1  k1 k

và 2 4 3 2  2  2 2  2 2 2 2 2 2

1

9 18 15

              

a k k k k k k k k k k a a a

b) Vì b a nên đặt bca

b tổng ba sốchính phương nên đặt bb12 b22 b32

(40)

Khi 2 2 2 2

1

   

b c a c a a a

Để kết thúc việc chứng minh, ta tiến hành sau: cho n2p1 ta được:

  2 

2 2

1

   

p p

b b b b b cho n2p2 ta   2 2 2

1

  

n p

b b b a a a

Dạng 2: Chứng minh số không số phương * Cơ sở phương pháp:

Để chứng minh n khơng số phương, tùy vào tốn ta sử dụng cách sau:

1) Chứng minh n viết dạng bình phương số nguyên 2) Chứng minh k2< n < (k + 1)2với k số nguyên

3) Chứng minh n có tận 2; 3; 7; 4) Chứng minh n có dạng 4k + 2; 4k + 5) Chứng minh n có dạng 3k +

6) Chứng minh n chia hết cho số nguyên tố p mà khơng chia hết cho p2 * Ví dụ minh họa:

Bài tốn 1.Một sốtựnhiên có tổng chữ sốbằng 2018 có thểlà sốchính phương khơng ? sao?

Hướng dẫn giải

Gọi số tự nhiên có tổng chữ số 2018 n

Ta có : 2018 = 3m + nên số tự nhiên n chia dư 2, số n có dạng 3k + với k số tự nhiên Mặt khác số phương trình khơng có dạng 3k + suy số tự nhiên n khơng số phương

Bài toán 2.Chứng minh số

2 2

Annnn n ∈ N n >1

khơng phải số phương

Hướng dẫn giải

Ta có:

   

     

 

4 2

2 2

2

2

2 2 2

1

1

A n n n n n n n n n

n n n n n n

A n n n

          

       

     Mặt khác:

(41)

 

 

 

2

2 2

4 2

2

1 2

2 2 1

1

n n n n n n n

n n n n n A n A n

A n n

       

          

   

Do  2 2  2 2

1

nn  A n  n

Ta có (n2+ n) (n2+ n + 1) hai số tự nhiên liên tiếp nên A khơng thể số phương

Bài tốn 3. Cho 33

1 2

A= + + + + + Hỏi A có số phương khơng? Vì sao?

Hướng dẫn giải

Ta có ( 5) ( 30 31 32 33)

1 2 2 2 2

A= + + + + + + + + + +

( ) ( )

2 30

3 2 2

= + + + + + + + + +

( )

29 29

3 2.30 30 3.10

= + + + = + + +

Ta thấy A có chữ số tận

Mà số phương khơng có chữ số tận Do đó, A khơng số phương Vậy A khơng số phương

Bài tốn 4.Chứng minh 4 4

2012 n 2013 n 2014 n 2015 n

A= + + + số

phương với số nguyên dương n

(Đề thi vào lớp 10 chuyên trường ĐHSP TP Hồ Chí Minh 2015 - 2016)

Hướng dẫn giải

Ta có:

4

2012 n4; 2014 n4, ∀ ∈n N*

( )

4 4

2013 n =2013 n− + =1 2013 n− +1 chia cho dư

( )4

4

2015 n =2015 n− −1 n +1 chia cho dư

Do đó, 4 4

2012 n 2013n 2014 n 2015 n

A= + + + chia cho dư

Ta có: A2, A không chia hết cho 22, mà số nguyên tố Suy A khơng

là số phương

Vậy A khơng số phương

(42)

Bài toán 5. Cho 2 n , Chứng minh An6 n4 2n32n2 khơng thểlà sốchính

phương

Hướng dẫn giải

Ta có 2 

2 2

       

A n n n n n n n n

   

2 2

1

 

n n n   n 

    

2

1

 

n n nn  n 

 2 

2

1 2

n nnn

Với 2 n , ta có n22n 2 n22n  1 n 12

Và 2  

2 2

     

n n n n n Do  2 2

1 2

    

n n n n

Như

2

 

n n sốchính phương nên A khơng phải sốchính

phương

Bài toán 6. Chứng minh tổng bình phương hai sốlẻbất kì khơng phải số phương

Hướng dẫn giải

Giả sử: a2m1, b2n1, với m n, 

Ta có: 2 2   2 2  2 2 

2 4

           

a b m n m m n n k với k

Khơng có sốchính phương có dạng 4k2 a2b2 khơng phải sốchính

phương

Dạng 3: Điều kiện để số số phương

* Cơ sở phương pháp: Chúng ta thường sử dụng phương pháp sau: - Phương pháp 1: Sửdụng định nghĩa

- Phương pháp 2: Sửdụng tính chẵn, lẻ

- Phương pháp 3: Sửdụng tính chất chia hết chia có dư - Phương pháp 4: Sửdụng tính chất

* Ví dụ minh họa:

(43)

Bài toán Tìm số nguyên n cho n(n + 3) số phương

Hướng dẫn giải

Để A = n(n + 3) số phương n(n + 3) = k2với k số tự nhiên, đó:

       2 2 2 2

4 12

4 12 9

2

2 2

n n k

n n k

n n k

n k

n k n k

 

  

    

   

     

Ta có 2n2k 3 2n2k3

Và = 9.1 = 3.3 = (-1).(-9) = (-3).(-3)

Trường hợp 1: 2

2 1

n k n k n

A

n k n k k

                                   

Trường hợp 2: 2 3 0

2 3 0

n k n k n

A

n k n k k

                                  

Trường hợp 3: 2 4

2

n k n k n

A

n k n k k

                                       

Trường hợp 4: 2 3 3

2 3

n k n k n

A

n k n k k

                                       

Vậy n = -4; -3; 0; ta có A sốchính phương

Bài tốn Tìm sốngun n cho n+1955 n+2014 sốchính phương

Hướng dẫn giải

Giả sử

1955

n+ =a ; n+2014=b2 với a, b∈ a<b

Khi 2 ( )( ) 29

59 59

59 30

b a a

b a b a b a

b a b

− = =

 

− = ⇔ − + = ⇔ ⇔

+ = =

 

Dễdàng suy n= −1114

Bài tốn Tìm số ngun dương n để biểu thức sau số phương:

2

) )

a An  n b Bn  n

Hướng dẫn giải

(44)

a) Với n = A = n2 – n + = khơng số phương Với n = A = n2 – n + = số phương

Với n > A = n2 – n + khơng số phương

 2    

1 2

n  n n n   n n

Vậy n = A số phương b) Ta có:    

1

n  n nn n

Với n = 5k n chia hết cho

Với n5k1thì n21chia hết cho

Với n5k2thì n2 1chia hết cho

Do

nn ln chia hết cho

Nên

2

n  n chia cho dư nên n5 n có chữ sốtận nên

5

2

Bn  n khơng sốchính phương

Vậy khơng có giá trịnào n thỏa đểB sốchính phương

Bài tốn Tìm sốngun dương n nhỏnhất cho số n+1, 2n+1, 5n+1

sốchính phương

Hướng dẫn giải

Nếu n=3k+1 (k∈) n+ =1 3k+2, khơng sốchính phương

Nếu n=3k+2 2n+ =1 6k+5, cho cho dư nên khơng sốchính phương Vậy n3 2n+1 sốchính phương lẻ nên chia cho dư Suy 2n8⇒n4⇒ +n lẻ Do n+1

sốchính phương lẻnên n+1 chia cho dư 1, suy n8

n chia hết cho số nguyên tốcùng nên n24 Với n=24

1 25

n+ = = ,

2

2n+ =1 49=7 , 5n+ =1 121 11=

Giá trịnhỏnhất n phải tìm 24

Bài tốn Tìm số tự nhiên n ≥ cho tổng 1! + 2! + 3! + … + n! số phương

(Đề thi HSG lớp - Phịng giáo dục đào tạo Phúc Yên - Vĩnh Phúc)

Hướng dẫn giải

Với n = thì1! = = 12là số phương

(45)

Với n = 1! + 2! = khơng số phương

Với n = 1! + 2! + 3! = + 1.2 + 1.2.3 = = 32là số phương

Với n ≥ ta có 1! + 2! + 3! + 4! = + 1.2 + 1.2.3 + 1.2.3.4 = 33 5!; 6!; …; n!đều tận 1! + 2! + 3! + … n! có tận chữ số nên khơng phải số phương Vậy có số tự nhiên n thoả mãn đề n = 1; n =

Bài tốn Tìm số nguyên dương n cho ( )( )

3 14

A= n+ n + n+ số

phương

(Đề thi chọn HSG Tốn tỉnh Thái Bình) Hướng dẫn giải

Ta có: ( )( )

4n +14n+ =7 n+3 4n+2 +1 n số nguyên dương nên n+3 4n2+14n+7

nguyên tố Vì vậy, để A số phương

4n +14n+7 n + phải số

chính phương

Do nZ+ nên ta có (2n+3)2 ≤4n2+14n+ <7 (2n+4)2

( )2

4n 14n 2n

⇒ + + = + ⇒ =n Khi n + = số phương

Thử lại, với n=1, ta có A=102

Vậy số nguyêndương cần tìm n=1

Bài tốn Tìm 3≤ ∈a  cho a a( −1 ) (a a− =1) (a−2) (aa a−1 )

Hướng dẫn giải

Ta có ( ) ( ) ( ) ( ) ( ) (2 ) ( )

1 1

a aa a− = aaa a− ⇔a a− = aaa a− (*)

Vì VT(*) sốchính phương nên VP(*) sốchính phương

Vì sốchính phương chỉcó chữ sốtận thuộc tập hợp {0;1; 4;5; 6;9}

nên a có chữ sốtận thuộc tập hợp {1; 2;5; 6; 7; 0}

Do a chữ sốnên a≤9 Kết hợp với 3≤ ∈a  nên a∈{5; 6; }

Thửlần lượt giá trịta thu a=7 thỏa mãn 762 =5776

Bài tốn Tìm sốtựnhiên n cho 2n+9 sốchính phương

Hướng dẫn giải

(46)

Giả sử

2n+ =9 m , m∈ ⇔ (m−3)(m+ =3) n

m− < +3 m nên ,

3

a b

m m

 − = 

+ =

 với a, b∈ a<b

Ta có 2b−2a = ⇔6 2a(2b a− − =1)

Vì 2a(2b a− −1 2) mà 2a(2b a− −1)4 nên a=1 Điều dẫn đến m=5 n=4

Dạng 4: Tìm số phương

* Cơ sở phương pháp: Dựa vào định nghĩa số phương A = k2, với k số nguyên yêu cầu tốn để tìm số phương thỏa tốn

* Ví dụ minh họa:

Bài tốn Tìm số phương abcd biết ab cd− =1

Hướng dẫn giải

Giả sử ( )

100 100

n =abcd = ab+cd = +cd +cd =101cd+100, nZ

( )( )

2

101.cd n 100 n 10 n 10

⇒ = − = − +

n<100 101 số nguyên tố nên n+10 101= 91

n

⇒ =

Thử lại:

91 8281

abcd = = có 82 81 1− =

Vậy abcd =8281

Bài tốn Cho A số phương gồm chữ số Nếu ta thêm vào chữ số A đơn vị ta số phương B Hãy tìm số A B

Hướng dẫn giải

Gọi

A=abcd =k

Theo đề ta có:

Ta có:

2 1111

A abcd k

B abcd m

 = = 

= + =



(với *

,

k mN 31< <k m<100, a b c d, , , =1, 9) 2 1111

m k

⇒ − = ⇔ (m - k)(m + k) = 1111 (*)

(47)

Nhận xét thấy tích (m – k)(m + k) > nên m – k m + k số nguyên dương Và m – k < m + k < 200 nên (*) viết (m – k) (m + k) = 11.101

Do đó: 11 56 2025

101 45 3136

− =  = =

 

⇔ ⇔

 + =  =  =

  

m k m A

m k n B

Vậy A = 2025, B = 3136

Bài tốn Tìm số phương gồm chữ số cho chữ số cuối số nguyên tố, bậc hai số có tổng chữ số số phương

Hướng dẫn giải

Gọi số phải tìm abcd với a; b; c; d số tự nhiên ≤ a ≤ 9; ≤ b, c, d ≤ Ta có abcd phương ⇒ d ∈{0,1,4,5,6,9}

Vì d số nguyên tố ⇒ d =

Đặt abcd = k2< 10000 ⇒ 32 ≤ k < 100, kN

Do k số có hai chữ số mà k2có tận ⇒ k tận

Tổng chữ số k số phương ⇒ k = 45 (vì k tận có chữ số)

abcd = 2025

Vậy số phải tìm là: 2025

Bài tốn Tìm số tự nhiên có hai chữ số biết hiệu bình phương số số viết hai chữ số số theo thứ tự ngược lại số phương

Hướng dẫn giải

Gọi số phải tìm ab với a, b ∈ N, ≤ a ≤ 9; ≤ b ≤ Theo giả thiết ta có: ab2 = (a + b)3 ( ) (2 )

ab a b a b

⇔ = + + Suy a+b số phương Khi ab lập phương a + b số phương

Vì 10 ≤ ab ≤ 99 ⇒ ab = 27 ab = 64 Nếu ab = 27 ⇒ a + b = số phương

(48)

Nếu ab = 64 ⇒ a + b = 10 khơng số phương ⇒ loại Vậy số cần tìm 27

C BÀI TẬP ÁP DỤNG

Bài 1: Cho số nguyên thỏa mãn điều kiện Chứng minh số phương

Bài 2: Tìm số nguyên dương n cho (2 1)

26

n n

là số phương

(Đề TS lớp 10 THPT Chuyên Lam Sơn- Thanh Hóa 2012-2013)

Bài 3: Tìm tất số nguyên n cho

A=n +n +n có giá trị số phương

(Đề TS lớp 10 THPT Chuyên Phan Bội Châu-Nghệ An 2010-2011 )

Bài 4: Chứng minh số nguyên x, y thìbiểu thức A=(x + y)(x + 2y)(x + 3y)(x + 4y) +

y có giá trị số phương

Bài 5: Chứng minh sốsau sốchính phương:

a)  

2

224 99 9100 

n n

A b)  

1

11 155 

n n

B

Bài 6: Chứng minh tổng bình phương sốliên tiếp khơng thểlà sốchính phương

Bài 7: Cho dãy số 49; 4489; 444889; 44448889;

Dãy sốtrên xây dựng cách thêm số 48 vào sốđứng trước Chứng minh

rằng tất cảcác sốcủa dãy sốchính phương

Bài 8: Chứng minh p tích n sốnguyên tốđầu tiên p−1 p+1 khơng thểlà sốchính phương

Bài 9:Có hay khơng số tự nhiên n để 2010 + n2là số phương.

Bài 10: Chứng minh tổng bình phương sốtựnhiên liên tiếp khơng thểlà

một sốchính phương

Bài 11: Chứng minh n số tự nhiên cho n + 2n + số phương n bội số 24

Bài 12: Tìm số phương có chữ số biết chữ số đầu giống nhau, chữ số cuối giống

Bài 13 : Tìm số lẻ liên tiếp mà tổng bình phương số có chữ số giống

; ;

a b c ab+bc+ca=1

2 2

(a +1)(b +1)(c +1)

(49)

Bài 14: Cho số nguyên dương n số A =

2

444

n

 (A gồm 2n chữ số 4); B = 888

n

 (B gồm n chữ số 8) Chứng minh A + 2B + số phương

(Đề vào chun tốn Hà Nam năm 2013-2014)

Bài 15: Giả sử N =1.3.5.7 2007

Chứng minh sốnguyên liên tiếp 2N−1, ,N 2N+1khơng có sốnào số phương

Bài 16: Với số nguyên dương n, ký hiệu Sn tổng n số nguyên tố

(S1 =2,S2 = +2 3,S3 = + +2 5, ) Chứng minh dãy số S S S1, 2, 3, không tồn hai số hạng liên tiếp số phương

(Đề vào chuyên toán sư phạm Hà Nội năm 2013-2014)

Bài 17: Cho p số nguyên tố Tìm p để tổng ước nguyên dương

p số

chính phương

(Đề vào chuyên Hưng Yên năm 2013-2014)

Bài 18: Tìm tất sốtựnhiên n cho n2 – 14n – 256 sốchính phương

(Đề thi HSG lớp Thanh Oai năm 2012-2013)

Bài 19: Cho sốnguyên a,b, c ≠ thoảmãn: 1 1

a b c abc+ + =

Chứng minh rằng: (1 a b c+ 2)( + 2)( + 2) là sốchính phương

(Đề thi HSG lớp trường Trần Mai Ninh năm 2012-2013)

Bài 20: Tìm sốtựnhiên n cho A= n2+n+6 sốchính phương

(Đề thi HSG lớp huyện Vĩnh Lộc năm 2018-2019)

Bài 21: Tìm sốtựnhiên gồm bốn chữ số abcd biết sốchính phương, chia

hết cho d sốnguyên tố

(Đề thi HSG lớp quận Ngô Quyền năm 2018-2019)

Bài 22: (Đề thi HSG lớp huyện Cẩm Giang năm 2018-2019)

Cho S = + 22 + 23 + + 298 Chứng tỏS khơng phải là sốchính phương. Bài 23: Tìm x ngun dương để 4x 14x 9x 63+ + − là sốchính phương

(Đề thi HSG lớp TP Bắc Giang năm 2017-2018)

Bài 24: Tìm sốtựnhiên n cho n 172+ là sốchính phương?

(Đề thi HSG lớp huyện Kim Thành năm 2012-2013)

Bài 25: Tìm sốnguyên dương n cho 2n +3n +4n sốchính phương

(Đề thi HSG lớp huyện Vũ Quang năm 2018-2019)

Bài 26: Tìm tất cảcác sốnguyên n cho n 20142 + là sốchính phương

(Đề thi HSG lớp Trường Thanh Văn năm 2017-2018)

Bài 27: Tìm sốnguyên x cho x 3x x 23− 2+ + sốchính phương

(Đề thi HSG lớp huyện Lục Nam năm 2018-2019)

Bài 28: Tìm sốtựnhiên Abiết ba mệnh đềsau có hai mệnh đềđúng mệnh đềsai:

a) A 51+ sốchính phương

b) Chữ sốtận bên phải Alà số1

(50)

c) A 38− sốchính phương

(Đề thi HSG lớp huyện Đan Phượng năm 2018-2019)

Bài 29: Tìm số hữu tỉ n thỏa mãn tổng sau sốchính phương: n2+ +n 503

Giả sửtồn số hữu tỉ n sốnguyên dương m để n2+ +n 503=m2

(Đề thi HSG lớp huyện Vũ Quang năm 2018-2019)

Bài 30: Tìm sốtựnhiên n cho n 50− n 50+ sốchính phương

(Đề thi HSG lớp huyện Thăng Bình năm 2018-2019)

Bài 31: Tìm sốtựnhiên n cho: n + 24 n – 65 hai sốchính phương

(Đề thi HSG lớp huyện Phù Ninh năm 2018-2019)

Bài 32: Chứng minh rằng: B = 4x(x + y)(x + y + z)(x + z) + y2z2là sốchính phương với x, y, z sốnguyên

(Đề thi HSG lớp huyện Tiền Hải năm 2017-2018)

Bài 33: Tìm *

n∈ cho: n4+n3+1 sốchính phương

(Đề thi HSG lớp huyện Thanh Oai năm 2012-2013)

Bài 34: Tìm tất cảcác cặp sốtựnhiên cho

đều sốchính phương

(Đề vào 10 Chuyên Nam Định năm 2019-2020)

Bài 35: Chứng minh số chia hết cho sốchính phương khác với số nguyên dương

(Đề vào 10 Chuyên Bình Thuận năm 2019-2020)

Bài 36: Cho là số nguyên dương thỏa mãn số nguyên Chứng minh số phương

(Đề vào 10 Chuyên Bắc Ninh năm 2019-2020)

Bài 37: Cho a, b, c số nguyên dương nguyên nguyên tố thỏa mãn

1 1

a  b c Chứng minh ab số phương

(Đề vào 10 Chuyên Thái Nguyên năm 2016-2017)

Bài 38: Chứng minh a b số tự nhiên lẻ a2 b2 khơng phải số

phương

(Đề vào 10 Chun Hịa Bình năm 2016-2017)

Bài 39: Tìm tất số nguyên dương n cho n2 3n là số phương.

(Đề vào 10 Chuyên Quốc Học Huế năm 2017-2018)

Bài 40: Chứng minh sốtựnhiên abc sốnguyên tốthì

4

bac khơng số

chính phương

(Đề vào 10 Chuyên Bình Định năm 2017-2018)

Bài 41: Tìm số nguyên m cho m2 +12 số phương

(Đề vào 10 Chuyên Phú Thọ năm 2017-2018)

Bài 42: Tìm tất cảcác cặp (x; y) nguyên dương cho

8

+

x y y2+8x sốchính

phương

(Đề vào 10 Chuyên Toán Hải Dương năm 2017-2018)

( )x y; x( +y2 −3x 2y 1+ )−

( 2 )

5 x +y +4x 2y 3+ +

( )4 4

M= n 1+ +n +1

n

n 12n2 1

2 12n  1 2

(51)

Bài 43:Cho biểu thức ( )2

A = m n+ +3m n+ với m, n số nguyên dương Chứng minh

rằng A số phương n3+1 chia hết cho m

(Đề vào 10 Chuyên TP Hồ Chí Minh năm 2017-2018)

Bài 44: Cho p số nguyên tố Tìm tất số nguyên n để A =n4 +4np 1− là số phương

(Đề vào 10 Chuyên Bà Rịa Vũng Tàu năm 2017-2018)

Bài 45: Cho hai sốnguyên dương m, n thỏa mãn m n+ +1là ước nguyên tốcủa

( 2)

2 m +n −1 Chứng minh m n sốchính phương

(Đề vào 10 Chuyên Nghệ An năm 2018-2019)

Bài 46: Tìm giá trịnguyên xđể M =x4+(x+1)3−2x2−2xlà sốchính phương

(Đề vào 10 Chuyên Hưng Yên năm 2018-2019)

Bài 47: Cho sốtựnhiên n≥2và sốnguyên tố pthỏa mãn p−1chia hết cho nđồng thời

3

1

n − chia hết cho p Chứng minh n+ plà sốchính phương

(Đề vào 10 Chuyên Đại học Vinh Nghệ An năm 2018-2019)

Bài 48: Tìm hai số nguyên tố p q, biết p+q p+4q số phương

(Đề vào 10 Chuyên Quảng Nam năm 2018-2019)

Bài 49: Chứng minh hiệu lập phương sốnguyên liên tiếp bình phương sốtựnhiên n n tổng sốchính phương liên tiếp

(Đề vào 10 Chuyên Bắc Ninh năm 2018-2019)

Bài 50: Chứng minh không tồn số tự nhiên n để 2018+n2 số phương (Đề vào 10 Chuyên Bắc Giang năm 2018-2019)

Bài 51: Cho 2

4

A=m nmn với m n, sốnguyên dương Khi n=2 tìm m đểA sốchính phương Khi n≥5chứng minh Akhơng thểlà sốchính phương

(Đề vào 10 Chuyên Bà Rịa Vũng Tàu năm 2018-2019)

Bài 52: Chứng minh số nguyên thỏa mãn hệ thức 2a+2b+1 số phương

Bài 53:Tìm số tự nhiên x để biểu thức

2 20

x + x+ có giá trị số phương

Bài 54 Tìm sốnguyên x cho Ax x( 1)(x7)(x8) sốchính phương

Bài 55 Cho  

2

11 88

n n

A= − + Chứng minh A số phương

Bài 56 Tìm tất số tự nhiên x,y để 2x + 5ylà số phương

Bài 57 Tìm nN để 28+211+2n số phương

Bài 58 Tìm sốtựnhiên n có chữ sốbiết 2n+1 1n+ sốchính phương

;

a b 2a2+ =a 3b2+b ab

(52)

Bài 59 Cho số:

2

1

11 11 11 11 ;

66 66

m

m

m A

B C

+

 =    =    = 

  

Chứng minh rằng: A B C+ + +8 sốchính phương

Bài 60 Tìm tất cảcác sốnguyên n cho n42n32n2 n sốchính phương

(Đềthi vào lớp 10 chuyên, trường ĐHKHTN – ĐHQG Hà Nội năm 1992)

Bài 61 Tìm tất cảcác sốngun khơng âm n cho có sốnguyên a, b thỏa mãn

2

n = +a b n3=a2+b2

(Romanian MO 2004)

Bài 62 Hãy tìm hai sốchính phương phần biệt a a a a1 b b b b1 biết

1 2 3 4

a − =b a − =b a − =b ab

Bài 63 Có tồn hay khơng 2013 sốnguyên dương a1, a2, , a2013 cho số

2

1 2,

a +a a12+a22+a32, a12+a22+ + a20132 sốchính phương?

Bài 64 Thay dấu * chữ số cho số sau số tựnhiên

6 4 ****

A=

Bài 65 Với n∈ , đặt ( )( )

10n 10n 10 10n

n

A = + − + + + + + + Chứng minh An

là sốchính phương

Bài 66 Giả sửrằng 2n+1 3n+1 sốchính phương Chứng minh 5n+3

hợp số

Bài 67 Có hay khơng số x y, phân biệt thuộc khoảng (988;1994) choxy+x xy+y sốchính phương ?

( Thi học sinh giỏi tốn lớp 9, TP.HCM năm 1994)

Bài 68 Có tồn hay không số tự nhiên n cho số k = n+ +1 n−1 số hữu tỉ

Bài 69 Cho dãy số , a2 =144, a3 =1444,

4

1444 44

n

n chu so

a = 

Tìm tất số tự nhiên n cho an số phương

Bài 70 Chứng minh có vơ số ba số tự nhiên (a b c, , )sao cho a b c, , nguyên tố

cùng số 2 2 2

n=a b +b c +c a số phương

Bài 71 Tìm số nguyên m n đa thức

( ) 29 4,

p x =x +mx + x +nx+ x∈

số phương Bài 72

(53)

1 Tìm số tự nhiên a nhỏ nhất, a≠0sao cho a chia hết cho 1000a số phương

2 Tìm sốtự nhiên b nhỏ cho số (b−1) không chia hết cho 9, b chia hết cho tích bốn số nguyên tố liên tiếp 2002.b số phương

Bài 73 Cho a b số tự nhiên, a2b2 số phương khơng?

Bài 74 Tìm sốtựnhiên k =ab có hai chữ số cho k+ab=(a b+ )2

Bài 75 Tìm tất cảcác sốnguyên n để 2 2

2017

An  n n sốchính phương

(Tạp chí Tốn & học tuổi trẻ số468)

Bài 76 Tìm sốnguyên dương n để 37

43

n n

 bình phương số hữu tỷdương tùy ý

(HSG Nam Định 2015 -2016)

Bài 77 Tìm sốtựnhiên có dạng abc thỏa mãn: abc=n2−1 cba=(n−2)2 với n∈, n >2

(HSG Sóc Trăng 2015 - 2016)

Bài 78 Tìm sốtựnhiên n cho n+12 n−11 sốchính phương

(HSG Sóc Trăng 2016 - 2017)

Bài 79 Tìm tất cảcác sốtựnhiên n cho

14 256

− −

n n sốchính phương

(HSG Quảng Nam 2014 - 2015)

Bài 80 Cho n sốtựnhiên có chữ số Tìm n biết n + 2n sốchính phương

(HSG Trà Vinh 2016 - 2017)

Bài 81 Cho n số tự nhiên Hãy tìm tất số nguyên tố p cho số

( ) 195

2 10

1010 2010 10

= + + +

A n n p có thểviết dạng hiệu sốchính phương

(HSG Lâm Đồng 2016 - 2017).

Bài 82 Tìm nghiệm nguyên dương x để 3x+171là sốchính phương

(HSG Lai Châu 2015 - 2016)

Bài 83 Tìm tất cảcác sốtựnhiên x cho 5x+12x sốchính phương

(HSG Bắc Giang 2015 - 2016)

Bài 84 Tìm tất số nguyên n cho A số phương với

4

4 22 37 12 12

= + + + −

A n n n n

(Chuyên Yên Bái 2016 - 2017).

Bài 85 Tìm sốnguyên kđể

8 23 26 10

− + − +

k k k k sốchính phương

(Chuyên Hải Dương 2015 - 2016).

Bài 86 Tìm sốtựnhiên n (n > 1) bé cho 12+22+ + ⋅⋅⋅ +32 n2

n sốchính phương (Tạp chí tốn học tuổi trẻ số362).

(54)

Bài 87: Tìm tất số tự nhiên n cho hai số 9n+16 16n+9 số

phương

Bài 88: Lấy sốtự nhiên có chữ sốchia cho số có chữ số viết theo thứtự ngược lại thương dư 15 Nếu lấy số trừ số tổng bình phương chữ sốtạo thành sốđó Tìm sốtựnhiên

Bài 89 Viết số1, 2, 3, …, 2007 thành dãy theo thứtựtùy ý số A Hỏi số

2007

2008 2009

A+ + có phải sốchính phương hay khơng? Vì sao?

(Tạp chí tốn học tuổi trẻ số377)

Bài 90 Cho số hữu tỉx, y thỏa mãn 5 2

2x

x +y = y Chứng minh 1−xy bình phương

của số hữu tỉ

Bài 91 Cho m n, hai sốnguyên dương lẻsao cho n21 chia hết cho [m2 1 n2] Chứng

minh 2

[m  1 n ] sốchính phương

Bài 92 Chứng minh ba sốchính phương tuỳý tồn hai sốmà hiệu chúng chia hết cho

Bài 93 Chứng minh

1999 2017

nn (nN) sốchính phương

(HSG Tỉnh Quảng Ngãi 2017 – 2018)

Bài 94 Giả sử n sốnguyên dương thoảmãn điều kiện

3

n  n sốnguyên tố Chứng

minh n chia dư 1và 7n26n2017 khơng phải sốchính phương

(Chun Tỉnh Quảng Ngãi 2017-2018)

Bài 95 Cho x y, sốnguyên thoảmãn 2x2 x 3y2y

Chứng minh xy; 2x2y1và 3x3y1 sốchính phương

(HSG Tỉnh Thanh Hoá 2015-2016)

Bài 96 Cho biểu thức 2

2(1 2017 )

A    Hỏi A có bình phương số

ngun hay khơng?

(Tốn học tuổi thơ số120)

Bài 97 Cho a b sốtựnhiên thoảmãn 2

2016a  a 2017bb (1)

Chứng minh ab sốchính phương

(Toán học tuổi thơ số120)

Bài 98 Cho x y z, , số nguyên tốcùng thoảmãn (xz y)(  z) z2 Chứng

minh tích

2017 xyz sốchính phương

(Toán học tuổi thơ số120)

Bài 99: Xác định sốđiện thoại THCS thành phố Thủ Dầu Một, biết sốđó dạng 82 xx yy

với x yyx sốchính phương

(55)

(HSG Bình Dương 2016 – 2017)

Bài 100: Tìm tất cảcác sốtựnhiên nsao cho sốchính phương

(HSG Quảng Bình 2018 – 2019)

Bài 101: Tìm sốnguyên tố thỏa mãn sốchính phương

(HSG Bắc Ninh 2018 – 2019)

Bài 102: Cho B=1.2.3 2.3.4 3.4.5 + + + +n n.( −1 ) (n−2) với *

n∈ Chứng minh B khơng sốchính phương

(HSG Bắc Ninh 2018 – 2019)

Bài 103: Cho số nguyên tố p (p>3)và hai số nguyên dương a,b cho p2 +a2 =b2

Chứng minh a chia hết cho 12 2(p+ +a 1) số phương

(HSG Quảng Nam 2018 – 2019)

Bài 104: Từ625 sốtựnhiên liên tiếp 1; 2; 3; …; 625 chọn 311 sốsao cho khơng có hai số có tổng 625 Chứng minh 311 sốđược chọn, bao giờcũng có sốchính phương

(HSG Hưng Yên 2017 – 2018)

Bài 105: Tìm sốtựnhiên cho sốchính phương

(HSG Hải Dương 2016 – 2017)

Bài 106: Tìm sốcó chữ số cho số sốchính phương

(HSG Hưng Yên 2015 – 2016)

Bài 107: Cho Chứng minh số sốchính phương

(HSG Đăk Lăk 2015 – 2016) Chương III

CÁC BÀI TOÁN VỀ SỐ NGUYÊN TỐ, HỢP SỐ

A. KiÕn thøc cÇn nhí

1 Định nghĩa số nguyên tố, hợp số.

1) Số nguyên tố số tự nhiên lớn 1, có ước số Ví dụ: 2, 3, 5, 11, 13,17, 19

2) Hợp số số tự nhiên lớn có nhiều ước Ví dụ: có ước số: ; nên hợp số

3) Các số khơng phải só ngun tố khơng phải hợp số 4) Bất kỳ số tự nhiên lớn có ước số nguyên tố

n

C 2019= +2020

p p 4p 93− +

n n2+2n+ n2+2n 18 9+ +

( )

ab ab n=ab ba

2017 cs1 a 111 1=

2016cs0

b 1000 5=  M ab 1= +

(56)

2 Một số tính chất

• Nếu số nguyên tố p chia hết cho số nguyên tố q p q=

• Nếu tích abc chia hết cho số ngun tố p thừa số tích abc chia hết cho số nguyên tố p

• Nếu a b khơng chia hết cho số ngun tố p tích ab không chia hếtcho số nguyên tố p

3 Cách nhận biết số nguyên tố

a) Chia số cho số nguyên tố biết từ nhỏ đến lớn

• Nếu có phép chia hết số khơng phải số nguyên tố

• Nếu chia lúc số thương nhỏ số chia mà phép chiavẫn cịn số dư số số ngun tố

b) Một số có ước số lớn số khơng phải số ngun tố 4 Phân tích số thừa số nguyên tố:

•Phân tích số tự nhiên lớn thừa số nguyên tố viết số dạng tích thừa sốngun tố

+ Dạng phân tích thừa số nguyên tố số ngun tố số + Mọi hợp số phân tích thừa số nguyên tố

Chẳng hạn A a b c , a, b, c số nguyên tố = α β γ α β, , , γ ∈N* Khi số ước số A tính (α+1)(β+1 ) (γ +1)

Tổng ước số A tính α − β+ − γ+ −

− − −

+1 1

a b. c

a b c

5 Số nguyên tố

Hai số a b nguyên tố ( )a,b =1 Các số a, b, c nguyên tố (a,b,c)=1

Các số a, b, c đôi nguyên tố ( ) ( ) ( )a,b = b,c = c,a =1 6 Cách nhận biết số nguyên tố

Cách

- Chia sốđó cho sốnguyên tốtừnhỏđến lớn: 2; 3; 5;

(57)

- Nếu có phép chia hết sốđó khơng sốngun tố

- Nếu thực phép chia lúc thương sốnhỏhơn sốchia mà phép chia có sốdư sốđó sốngun tố

Cách

- Một sốcó hai ước sốlớn sốđó khơng phải sốngun tố

- Ước sốnguyên tốnhỏnhất hợp số A sốkhông vượt A

- Với quy tắt khoảng thời gian ngắn, với dấu hiệu chia hết ta nhanh chóng trảlời sốcó hai chữ sốnào ngun tốhay khơng

B. MỘT SỐ DẠNG TOÁN SỐ NGUYÊN TỐ, HỢP SỐ

Dạng 1: Chứng minh số số nguyên tố hợp số

Bài tốn Chứng minh với sốtựnhiên n>1thì

1

n +n + hợp số

Hướng dẫn giải

Ta có: ( )( )

1 1

n +n + = n + +n n − +n

n>1 nên

1

n + + >n suy n5+n4+1 hợp số

Bài toán Nếu p p28 sốnguyên tốthì p22 sốnguyên tố

Hướng dẫn giải

Xét p3k1 (k nguyên)

8

p   , hợp số

Xét p3k2 p28 3 , hợp số

Vậy p3k, mà p sốnguyên tốnên p3

Khi

2 11

p   , sốnguyên tố

Bài toán 3. Chứng minh 2n −1 sốnguyên tố (n>2) 2n+1là hợp số

Hướng dẫn giải

Trong ba số nguyên 2n −1; ; 2n n+1 có số chia hết cho Mặt khác, 2n không chia hết

cho 3, hai số 2n −1; 2n+1 phải có sốchia hết cho 3, nghĩa

hai số phải có hợp số Để cho 2n−1 sốnguyên tố ( )

2

>

n nên chắn chắn 2n +1 hợp số

(58)

Bài toán 4. Cho p 8p−1là sốnguyên tố Chứng minh 8p+1là hợp số

Hướng dẫn giải

Vì 8p−1là sốnguyên tốnên p≠2

Nếu p=3 8p+ =1 25 hợp số

Nếu p>3 8p(8p−1 8)( p+1)3 Vì p 8p−1 sốnguyên tốlớn nên 8p+1

chia hết cho hay 8p+1là hợp số

Dạng 2: Tìm số nguyên tố thỏa mãn điều kiện cho trước * Cơ sở phương pháp:

* Trong n sốnguyên liên tiếp có số chia hết cho n.

* Mọi sốnguyên tốlớn có dạng .

* Mọi sốnguyên tốlớn có dạng 3k±1 .

* Mọi sốnguyên tốlớn có dạng .

Chứng minh:

*) Gọi m sốnguyên tốlớn

Mỗi sốtựnhiên chia cho có sốdư 0, 1, 2, sốtựnhiên viết

dưới dạng 4n – 1; 4n ; 4n + 1; 4n +

Do m sốnguyên tốlớn nên chia hết m khơng có dạng 4n 4n +

Vậy sốnguyên tốlớn đềcó dạng:

Khơng phải số có dạng đều số nguyên tố Chẳng hạn 4 - = 15 không số nguyên tố *) Gọi m sốnguyên tốlớn 3

Mỗi sốtựnhiên chia cho có sốdư 0, 1, 2, 3, 4, sốtựnhiên viết được dạng 6n – 1; 6n ; 6n + 1; 6n + ; 6n +

Do m sốnguyên tốlớn nên chia hết m khơng có dạng 4n 6n; 6n + 2; 6n+

Vậy sốnguyên tốlớn đềcó dạng: .

Khơng phải số có dạng đều số nguyên tố Chẳng hạn + = 25 khơng số ngun tố.

* Ví dụ minh họa:

Bài tốn 1.Tìmtất sốngun tốp cho p + 2và p + 4là sốnguyên tố Hướng dẫn giải

Với p = 2thì p+2= 4và p + 4= khơng phảilà số nguyên tố Với p = thìp + = p + = số nguyên tố

Với p > mà p số nguyên tố nên p có dạng p = 3k + p = 3k +

4n±1

6n±1

4n±1 4n±1

6n±1 6n±1(nN)

(59)

Nếu p+ =2 3k+ =3 3( k+1 3) không sốnguyên tố

Nếu p = 3k + thìp+ =4 3k+ =6 3( k+2 3) khơng số nguyên tố;

Vậy với thìp + vàp + 4là số nguyên tố

Bài toán 2.Tìm tất sốnguyên tốp cho p + 2; p + 6; p + 8; p + 14 đềulà sốnguyên tố

Hướng dẫn giải

Trường hợp 1: p =5k mà p nguyên tố nên p = 5, đó:

p + = 7; p + = 11; p + = 13; p + 14 = 19 sốnguyên tốnên p = thỏa mãn toán Trường hợp 2: p = 5k + 1, đó: p + 14 = 5k + 15 = 5(k + 3) có ước 1, (p + 14) nên p + 14 không sốnguyên tố

Vậy với p = 5k + khơng có tồn p nguyên tốthỏa mãn toán

Trường hợp 3: p = 5k + 2, đó: p + = 5k + 10 = 5(k + 2) có ước 1, (p + 10) nên p + 10 không sốnguyên tố

Vậy với p = 5k + khơng có tồn p nguyên tốthỏa mãn toán

Trường hợp 4: p = 5k + 3, đó: p + = 5k + = 5(k + 1) có ước 1, (p + 2) nên p + không sốnguyên tố

Vậy với p = 5k + tồn p ngun tốthỏa mãn tốn

Trường hợp 5: p = 5k + 4, đó:p + = 5k + 10 = 5(k + 2) có ước 1, (p + 6) nên p + không sốnguyên tố

Vậy với p = 5k + tồn p ngun tốthỏa mãn tốn Do p = sốcần tìm

Bài tốn 3.Tìm sốtựnhiên n cho

3

1

n

là sốnguyên tố

Hướng dẫn giải

3

1

n   n   nchia cho dư 1(vì n chia cho dư n3 chia

hết cho dư 2) Đặt n3k1(kN) Ta có

3 3

3 2

1 (3 1) 27 27

3 (3 1)

9 9

n k k k k

k k k k k k

    

       

Để

9

n

là sốnguyên tố, phải có k1 Khi n4

3

1 64

7

9

n  

  , số nguyên tố

Đáp số: n4.

3

p= k+

3

p=

(60)

Bài tốn Tìm sốngun tố p cho 43p1 lập phương sốtựnhiên

Hướng dẫn giải

Đặt

43p 1 n (nN) 43p (n 1)(n2 n 1)

Số 43p có bốn ước nguyên dương 1, 43, , 43p p nên có ba trường hợp:

a)

1

1 43

n

n n p

   

   

 Khi n2

2

43p2   2 7, loại

b) 2 43

1

n

n n p

   

   

 Khi n44

2

44 44 1981

p     , loại

c) 43

1

n n

n p

    

  

 Khi n n(  1) 42 n 6,p5 (là sốnguyên tố)

Đáp số: p5.

Dạng 3: Nhận biết số nguyên tố, phân bố nguyên tố tập hợp số tự nhiên

Các toán nhận biết số nguyên tố, phân bố số nguyên tố N, giúp cho học sinh hướng suy nghĩ để chứng minh xem xét số có phải số nguyên tố hay khơng? Thơng qua việc phân tích xét hết khả xảy ra, đối chiếu với giả thiết định lý, hệ học để loại bỏ trường hợp mâu thuẫn Bài toán số tập tổng quát phân bố số nguyên tố N Qua cho học sinh thấy phân bố số nguyên tố “càng sau rời rạc” Từ tốn phát triển thành toán khác giúp học sinh rèn luyện kỹ xảo chứng minh

* Ví dụ minh họa:

Bài toán Cho p sốnguyên tốvà số8p + 8p - sốnguyên tố, hỏi sốthứ3 (ngoài sốnguyên tố, sốcòn lại) sốnguyên tốhay hợp số?

Hướng dẫn giải

Với p = ta có 8p + = 25 hợp số, 8p -1 số nguyên tố

Với ta có 8p -1, 8p, 8p + số nguyên tố liên tiếp nên có số chia hết cho

Do p nguyên tố khác nên 8p không chia hết cho 3,do 8p -1 8p+1 có số chia hết cho

Vậy số thứ hai số nguyên tố hợp số

Bài toán Nếu p < 2p + số nguyên tố 4p + nguyên tố hay hợp số

Hướng dẫn giải

3

p

(61)

Xét số tự nhiên liên tiếp: 4p; 4p + 1; 4p + Trong số có số bội

Mà p < 5, p P nên p có dạng 3k + 3k + +) Nếu p = 3k + 4p = 4(3k + 1) <=> 3Q + = p

và 4p + = 4(3k + 1) + <=> p = 3.Q : Mặt khác: 4p + = 2(2p +1) = 3Q nên 3Q :

=> 2(2p + 1) : 3; (2;3) = nên (2p + 1) : (trái với giả thiết) +) Nếu p có dạng 3k +

Khi 4p + = 4(3k + 2) + = 12k + = 3M : => 4p + hợp số

Vậy số có số bội

Bài tốn Trong dãy số tự nhiên tìm 1997 số liên tiếp mà khơng có số nguyên tố hay không ?

Hướng dẫngiải

Chọn dãy số:

a1= 1998! + a1: a2= 1998! + a2 : a3= 1998! + a3: a1997= 1998! + 1998 a1997: 1998

Như vậy: Dãy số a1; a2; a3; a1997gồm có 1997 số tự nhiên liên tiếp khơng có số số ngun tố

Bài toán (Tổng quát số 3)

Chứng minh tìm dãy số gồm n số tự nhiên liên tiếp (n > 1) khơng có số số nguyên tố ?

Hướng dẫn giải

Ta chọn dãy số sau:

a1 = (n+1)! + a1:2 a1>2 nên a1 hợp số a2= (n+1)! + a2:3 a2>3 nên a2là hợp số

an= (n+1)! + (n+1) an:(n+1) an> (n+1) nên anlà hợp số

Dãy a1; a2; a3; anở gồm có n số tự nhiên liên tiếp khơng có số số ngun tố

Dạng 4: Có số nguyên tố dạng ax + b (với x N (a,b) = 1)

(62)

Mục đích tập dạng là: Rèn luyện cho học sinh khả tư sâu, cách xem xét kết luận vấn đề tốn học cách xét hết khả xảy ra, dùng vấn đề toán học chứng minh biết để loại bỏ khả xảy làm sáng tỏ vấn đề cần phải chứng minh

Sau thành thạo dạng toán học sinh THCS hiểu sâu sắc hơn, có khái niệm rõ ràng Thế nàolà chứng minh vấn đề tốn học có kỹ năng, kỹ xảo chứng minh cần thiết

Tuy nhiên, với dạng tốn này, trình độ THCS em giải tập dạng đơn giản Việc chứng tập dạng phức tạp hơn, em gặp nhiều khó khăn dễ dàng chứng minh Chẳng hạn chứng minh vơ số số ngun tố có dạng 4a + 1; 6a + phức tạp nhiều * Ví dụ minh họa:

Bài tốn 1.Chứng minh rằng: có vơ số số ngun tố có dạng: 3x – (x < 1)

Hướng dẫn giải

Mọi số tự nhiên không nhỏ có dạng: 3x; 3x + 1; 3x - +) Những số có dạng 3x (với x > 1) hợp số

+) Xét số có dạng 3x + 1: số (3m + 1) số (3n + 1) Xét tích (3m + 1)(3n + 1) = 9mn + 3m + 3n + = 3x + Tích có dạng: 3x +

+) Lấy số nguyên tố p có dạng 3x – (với p số nguyên tố) ta lập tích p với tất số nguyên tố nhỏ p trừ ta có:

M = 2.3.5.7 p – = 3(2.5.7 p) – M có dạng: 3x –

Có khả xảy ra:

* Khả 1: M số nguyên tố, số nguyên tố có dạng (3x – 1) > p, toán chứng minh

* Khả 2: M hợp số: Ta chia M cho 2, 3, 5, ,p tồn số dư khác nên ước nguyên tố M lớn p, ước khơng có số có dạng 3x + (đã chứng minh trên) Do ước ngun tố M phải có dạng 3x (hợp số) 3x +

Vì tất có dạng 3x + M phải có dạng 3x + (đã chứng minh trên) Do đó, ước nguyên tố M phải có dạng 3x + 1, ước lớn p

Vậy: Có vơ số số ngun tố dạng 3x –

Bài tốn 2. Chứng minh rằng: Có vơ số số nguyên tố có dạng 4x + (với x ∈ N)

(63)

Hướng dẫn giải

Các số ngun tố lẻ khơng thể có dạng 4x 4x + Vậy chúng tồn dưới1 dạng

4x + 4x + Ta chứng minh có vơ số số nguyên tố có dạng 4x + +) Xét tích số có dạng 4x + là: 4m + 4n +

Ta có: (4m + 1)(4n + 1) = 16mn + 4m + 4n + = 4(4mn + m + n) + = 4x +

Vậy tích số có dạng 4x+ số có dạng 4x +

+) Lấy số nguyên tố p có dạng 4x – 1, ta lập tích 4p với tất số nguyên tố nhỏ p trừ ta có:

N = 4(2.3.5.7 p) – Có khả xảy

* Khả 1:

N số nguyên tố => N =4(2.3.5.7 p) – có dạng 4x –

Những số ngun tố có dạng 4x – số có dạng 4x + tốn chứng minh

* Khả 2:

N hợp số: Chia N cho 2, 3, 5, , p số dư khác => ước nguyên tố N lớnhơn p

Các ước khơng thể có dạng 4x 4x + (vì hợp số) Cũng khơng thể tồn ước có dạng 4x + N phải có dạng 4x + Như ước ngun tố N có ước có dạng 4x – mà ước hiển nhiên lớn p

Vậy:Có vơ số số ngun tố có dạng 4x – (hay có dạng 4x + 3)

Trên mộ số toán chứng minh đơn giản định lý Đirielet: Có vơ số số nguyên tố dạng ax + b x N ,(a,b) =

Dạng 5: Các toán số nguyên tố

Hai số a b nguyên tốcùng ƯCLN(a, b) = 1.

Các sốa, b, c nguyên tốcùng ƯCLN(a, b, c) = 1. Các sốa, b, c đôi nguyên tốcùng

ƯCLN(a, b) = ƯCLN(b, c) = ƯCLN(c, a) = 1.

* Ví dụ minh họa:

Bài tốn Chứng minh rằng:

a)Hai số tự nhiên liên tiếp (khác 0) hai số nguyên tố b) Hai số lẻ liên tiếp hai số nguyên tố

c) 2n + 3n + ( ) hai số nhuyên tố

Hướng dẫn giải

nN

(64)

a) Gọi Vậy n n + hai số nguyên tố

b) Gọi Nhưng d

làước số lẻ Vậy d=1

c) Gọi ƯC

Bài toán Cho a b hai sốnguyên tốcùng Chứng minh hai sốsau hai sốnguyên tốcùng

a) a a + b b) a2và a + b. c) ab a + b.

Hướng dẫn giải

a) Gọi ƯC Ta lại có nên ƯC ,

d = 1(vì a, b hai số nguyên tố nhau) Vậy (a, a + b) =

b) Giả sử a2 và a + b chia hết cho số nguyên tố d a chia hết cho d, b chia hết cho d Như a b chia hết cho số nguyên tố d, trái với giả thiết (a, b) =1

Vậy a2và a + b hai số nguyên tố

c) Giả sử ab a + b chia hết cho số nguyên tố d Tồn hai thừa số a b, chẳng hạn a, chia hết cho d, b chia hết cho d, trái với (a, b) = Vậy (ab, a + b) =

Bài tốn 3.Tìm sốtựnhiên n đểcác số9n + 24 3n + sốnguyên tốcùng

Hướng dẫn giải

Giả sử 9n + 24 3n + chia hết cho số nguyên tố d

Điều kiện để (9n + 24, 3n + 4) = Hiển nhiên 3n + khơng chia hết cho Muốn phải có hai số 9n + 3n + không chia hết cho Ta thấy:

9n + số lẻ 9n lẻ n lẻ, 3n + số lẻ 3n lẻ n lẻ

Vậy điều kiện để (9n + 4, 3n + 4) = n số lẻ

Dạng 6: Sử dụng nguyên lý Dirichlet toán số nguyên tố

( , 1) ( 1) 1

duc n n+ ⇒ + −n n d ⇒ d⇒ =d

{ }

(2 1, 3) (2 3) (2 1) 1,

duc n+ n+ ⇒ n+ − n+ d⇒ d⇒ ∈d d ≠2

d∈ (2n+1, 3n+ ⇒1) 3(2n+ −1) 2(3n+1)d ⇒1d⇒1d

d∈ ( ,a a b+ ⇒) (a b+ −) a d ⇒b da dd∈ ( , )a b

{ }

9n+24 3(3− n+4)d ⇒12d ⇒ ∈d 2;3

2

dd ≠3 d ≠3

2

d

⇔ ⇔

⇔ ⇔

(65)

Bài toán Cho p>5 sốnguyên tố Chứng minh tồn sốcó dạng 111 11

chia hết cho p

Hướng dẫn giải

Ta xét dãy số: 1,11,111, ,111

p

 

Nếu dãy khơng có sốnào chia hết cho p ta cho tương ứng sốvới sốdư phép chia Tập hợp sốdư chỉcó 1, 2, 3,,p−1 gồm p−1 phần tử(vì khơng thể

có tập này)

Nhưng cóp sốởdạng nên theo ngun lý Dirichlet tồn hai sốcó sốdư Giả sửcác sốđó là: 111

m

 111

n

 với m>n

Khi 1≤ < ≤n m p

Như vậy: 111 111 111 1000 111 1.10n

m n m nn m n

− = =

    

    Tích chia hết cho p (p,10)=1 , suy 111

m n

 chia hết cho p nằm dãy Mà 1≤ − ≤m n p mâu thuẫn với giả thiết khơng có sốnào dãy chia hết cho

p

Bài toán Chứng minh 12 sốnguyên tốphân biệt chọn sốký

hiệu p1, p2, p3, p4, p5, p6 cho (p1−p2)(p4−p3)(p5+p6)1800

Hướng dẫn giải

Vì ba sốnguyên tốđầu tiên 2, 3, nên 12 sốngun tốphân biệt cho ln có

nhất sốlớn Vì sốnguyên tốlớn nên: sốtrên chia cho có sốdư

hoặc Theo nguyên lý Dirichlet phải tồn số chia cho có số dư

Mà sốnày lại khơng chia hết cho 5, thếtrong sốấy có sốmà ta có thểgiả

sửlà p p1, cho (p1−p2)5 Ngồi hiển nhiên ta có (p1−p2)3 dẫn đến (p1−p2)15

Xét số lại theo nguyên lý Dirichlet tồn số có số dư chia hết cho

Đem sốnày chia cho cho hai khảnăng xảy ra:

Nếu có số (chẳng hạn p p3, 4)mà (p3−p4)5 Rõ ràng (p3−p4)2 (p3−p4)3 Vì (5;3; 2)=1 nên ta có (p3−p4)30 Lấy hai số p p5, (ngoài p p p p1, 2, 3, 4) chọn

thì p p5, lẻ(do sốnguyên tốkhác 2) nên (p5+p6)2

Từđó suy (p1−p2)(p4−p3)(p5+p6)30.30.2=1800

(66)

Nếu số chia cho có số dư khác 1; 2;3; Giả sử (p5−1 5) , (p6−4 5) (p5+p5−5 5) hay (p5+p6)5

Với sốcịn lại p p3, 4 rõ ràng (p3− p4)3 (theo cách chọn sốtrên)

Do p p p p3; 4; 5; lẻnên (p5+ p6) (2, p3−p4)2

Từđó suy (p5+ p6)10 (p3−p4)6

Do (p1−p2)(p4−p3)(p5+p6)30.10.6=1800

Vậy tồn p1, p2, p3, p4, p5, p6 sốnguyên tốphân biên cho

(p1−p2)(p4−p3)(p5+ p6)1800

Dạng 7: Áp dụng định lý Fermat

p sốnguyên tốvà ( , ) 1a p = ap−1 ≡1(mod )p

* Ví dụ minh họa:

Bài toán Cho n∈*, chứng minh 2210 1n+ +19

và + + + +

4

3

2 n n hợp số

Hướng dẫn giải

a) Ta chứng minh + +  10

2

2 n 19 23 với n≥1

Ta có: 210 ≡1(mod 1) ⇒210 1n+ ≡2 (mod 22) ⇒210 1n+ =22k+2(k∈). Theo Định lí Fermat:

22

2 1(mod 23) + + ⇒ 210 = 22 2≡

2 n k (mod 23) + ⇒ 210 = 

2 n 19 23 Mặt khác: +

+ > 10

2

2 n 19 23 nên + + 10

2

2 n 19 hợp sốvới n∈* b) Ta chứng minh + +

+ + 

4

3

2 n n 11 với n≥1 Bài tốn Tìm sốngun tố p cho 1p + chia hết cho p.

Hướng dẫn giải

Giả sử p sốnguyên tốthỏa mãn 1p + p

Theo Định lí Fermat:

2p 2(mod p) ⇒2 2p− p⇒ =3 (2 1) (2 2)p+ − p− p⇒ =p 3 Với p=3 ta có 2p+ =1 3

(67)

Bài toán Cho p sốngn tốlớn Chứng minh có vơ số sốtựnhiên thỏa

− 1n

n chia hết cho p

Hướng dẫn giải

Ta có 2p−1≡1(mod p), ta tìm n m p= ( −1) sao cho n.2n≡1 (mod p). Ta có: n.2n =m p( −1).2m p( 1)− ≡m p( −1)(mod p) ⇒n.2n≡ − ≡m 1(modp)

⇒ =m kp−1(k∈*).

Vậy, với n kp=( −1)(p−1) (k∈*) n.2 12− p

Bài tốn Cho p sốnguyên tố, chứng ming số 1p− chỉcó ước nguyên tốcó dạng

+

2pk

Hướng dẫn giải

Gọi q ước nguyên tốcủa 1p − thì q lẻ, nên theo Định lí Fermat:

− − ⇒ − − − = − − ⇒ −

  

1 ( , 1)

2q 1q (2 1,2p q 1) 2p q q q p, (q−1, ) 1p = thì 1q, vơ lí.

Mặt khác, q−1 chẵn suy q−1 2 p q⇒ =2pk+1

C BÀI TẬP ÁP DỤNG

Bài Tìm số nguyên tố p cho số sau số nguyên tố: a) p + p + 10

b) p + 10 p + 20

c) p +2, p + 6, p +8, p + 12, p + 14

Bài Chứng minh n n2+ số nguyên tố cũng số nguyên tố. Bài Chứng minh a, a + k, a + 2k ( ) số nguyên tố lớn k chia hết cho

Bài Chứng minh p số nguyên tố lớn (p - 1)(p + 1) chia hết cho 24 Bài Một số nguyên tố p chia cho 42 có dư hợp số r Tìm r

Bài Một số nguyên tố p chia cho 30 có số dư r Tìm r biết r không số nguyên tố

Bài Chứng minh số hợp số với

Bài Tìm n số cho 10101 0101 (n chữ số n + chữ số xen kẽ nhau) số nguyên tố Bài Các số sau số nguyên tố hay hợp số

3

2

n +

, *

a kN

 

11 1211

n n

1

n

(68)

a) A = 11 1(2001 chữ số 1); b) B = 11 (2000 chữ số 1); c) C = 1010101;

d) D = 1112111;

e) E = 1! + 2! + 3! + +100!; g) G = - 28;

h) H= 311141111

Bài 10 Cho ,chứng minh số sau hợp số: a) A = ;

b) B = ; c) C =

Bài 11 p số nguyên tố lớn 5, chứng minh (mod 240) Bài 12 Chứng minh dãy có vơ số hợp số

Bài 13 Chứng minh với số ngun tố p có vơ số dạng chia hết cho p

Bài 14 Tìm để số nguyên tố

Bài 15 Tìm số cho số nguyên tố

Bài 16 Tìm tất số ngun tố p có dạng ( ) Bài 17 Cho , chứng minh hợp số với n>1

Bài 18 Giải phương trình nghiệm nguyên (1)

trong a, b số nguyên cho trước a > b Bài 19 Giải phương trình nghiệm nguyên sau:

a)

b)

Bài 20 Chứng minh với số tự nhiên n, số sau hai số nguyên tố nhau: a) 7n + 10 5n + ;

b) 2n + 4n +

Bài 21 Cho a b hai số nguyên tố Chứng minh số sau hai số nguyên tố nhau:

a) b a - b (a > b) ; b) a2+ b2và ab

Bài 22 Chứng minh số c nguyên tố với a với b c ngun tố với tích ab

Bài 23 Tìm số tự nhiên n, cho:

*

nN

2

2 n+ +3

4

2 n+ +7

6

2

2 n+ +13

4

p

10n

n

a = +

2n n

*

nN

1

nn + −n

, *

x yN 4

4

x + y

( 1)( 2)

n n+ n+ +

1

n

*

nN

4n

A=n +

2 4(ax x)( −b)+ − =b a y

2 585

x +y =

2

1210

x +y =

(69)

a) 4n - chia hết cho 13 ; b) 5n + chia hết cho ; c) 25n + chia hết cho 53

Bài 24 Tìm số tự nhiên n để số sau nguyên tố nhau: a) 4n + 2n + ;

b) 7n + 13 2n + ; c) 9n + 24 3n + ; d) 18n + 21n +

Bài 25 Chứng minh có vơ số số tự nhiên n để n + 15 n + 72 hai số nguyên tố

Bài 26 (Trích đề thi học sinh giỏi lớp Đồng Nai năm học 2018-2019)

Tìm sốcác sốnguyên dương không vượt 1000 nguyên tốcùng với 999 Bài 27 (Trích đề thi học sinh giỏi lớp Ninh Bình năm học 2018-2019)

Tìm tất cảcác bộba sốnguyên tố cho Bài 28 (Trích đề thi học sinh giỏi lớp Bắc Ninh năm học 2018-2019)

Tìm sốngun tố thỏa mãn sốchính phương Bài 29 (Trích đề thi học sinh giỏi lớp Phú Yên năm học 2018-2019)

Tìm hai số nguyên tố p, qsao cho

8q+ =1 p

Bài 30 (Trích đề thi học sinh giỏi lớp Thái Bình năm học 2018-2019)

Tìm tất số nguyên dương (x y z; ; )sao cho 2019 2019

+ +

x y

y z số hữu tỉ

2+ 2+

x y z sốnguyên tố

Bài 31.(Trích đề thihọc sinh giỏi lớp Quảng Nam năm học 2018-2019)

Cho số nguyên tố p(p>3)và hai số nguyên dương a,b cho p2 +a2 =b2

Bài 32.(Trích đề thi học sinh giỏi lớp Thanh Hóa năm học 2017-2018)

Cho sốnguyên dương thỏa mãn sốnguyên tố chia hết cho Giả sử số nguyên thỏa mãn chia hết cho Chứng minh hai số chia hết cho .

Bài 33.(Trích đề thi học sinh giỏi lớp Thanh Hóa năm học 2016-2017)

Cho p số nguyên tố lớn Chứng minh chia hết cho 60 Bài 34.(Trích đề thi học sinh giỏi lớp Nghệ An năm học 2016-2017)

Tìm tất cảcác sốnguyên tốkhác m, n, p, q thỏa mãn

(p;q; r) pqr= + + +p q r 160

p p 4p 93− +

,

a b p a= 2+b2 p 5−

,

x y ax2−by2 p

x,y p

2016

p –

1 1 1 1.

m n p q mnpq+ + + + =

(70)

Bài 35.(Trích đề thi học sinh giỏi lớp Thanh Hóa năm học 2015-2016)

Tìm nghiệm ngun phương trình: Bài 36.(Trích đề thi học sinh giỏi lớp Vĩnh Long năm học 2015-2016)

Cho p q sốnguyên tốlớn thỏa mãn Tìm sốdư chia cho 12

Bài 37 (Trích đề thi học sinh giỏi lớp Hà Nội năm học 2015-2016)

Tìm tất cảcác sốnguyên tốx cho sốnguyên tố Bài 38 (Trích đề thi học sinh giỏi lớp Nghệ An năm học 2014-2015)

Tìm sốtựnhiên n cho số2015 có thểviết thành tổng n hợp sốnhưng không thểviết thành tổng n + hợp số

Bài 39 (Trích đề thi học sinh giỏi lớp Thanh Hóa năm học 2014-2015)

Tìm tất cảcác sốnguyên tốp, qsao cho tồn sốtựnhiên mthỏa mãn:

Bài 40 (Trích đề thi học sinh giỏi lớp Hải Dương năm học 2014-2015)

Tìm sốnguyên tốpsao cho số sốnguyên tố Bài 41 (Trích đề thi học sinh giỏi lớp Cẩm Thủynăm học 2011-2012)

Tìm sốtựnhiên n để A n= 2012+n2002+1là sốnguyên tố Bài 42 (Trích đề thi học sinh giỏi lớp Tiền Hảinăm học 2016-2017)

Tìm tất cảcác sốnguyên dương a, b, c thỏa mãn: a b

b c

− số hữutỉ

2 2

a b c+ + sốnguyên tố Bài 43 (Trích đề thi học sinh giỏi lớp Gia Lộcnăm học 2015-2016)

Tìm sốnguyên tố k để k 42+ và k 162+ đồng thời sốnguyên tố Bài 44 (Trích đề thi học sinh giỏi lớp Lục Namnăm học 2018-2019)

Cho p sốnguyên tốlớn Chứng minh p20−1 chia hết cho 100

Bài 45 (Trích đề thi học sinh giỏi lớp Kim Thànhnăm học 2018-2019)

Cho p sốnguyên tốlớn Chứng minh p 242−  Bài 46 (Trích đề chọnhọc sinh giỏi lớp Amsterdamnăm học 2018-2019)

Tìm tất bộba sốnguyên dương (p;q; n), p, q sốnguyên tố

thỏa mãn: p p q q 3( + +) ( + ) (=n n 3+ )

Bài 47 (Trích đềvào 10Chun tốn Hải Phịng năm học 2019-2020)

Tìm sốngun tốp, qthoảmãn đồng thời hai điều kiện sau:

( )x; y 54x y 3+ =

p q 2= +

p q+

x 2 x+

2 pq m 1.

p q m

+ =

+ +

2 2

2p 1; 2p− +3; 3p +4

(71)

i) chia hết cho ii) chia hết cho

Bài 48 (Trích đề vào 10Chun tốn Quảng Bình năm học 2019-2020)

Cho số nguyên tố Chứng minh phương trình khơng có nghiệm hữu tỉ

Bài 49 (Trích đềvào 10Chuyên Tin Lam Sơn năm học 2018-2019)

Tìm sốnguyên dương a, bnguyên tốcùng thỏa mãn 2 2 41

+ = +

a b

a b

Bài 50 (Trích đềvào 10Chuyên Tin Lam Sơn năm học 2015-2016)

Cho dãy số tự nhiên 2; 6; 30; 210; xác định sau: số hạng thứ k tích k số nguyên tố đầu tiên(k=1; 2;3; ) Biết có hai số hạng dãy số có hiệu

bằng 30000 Tìm hai số hạng

Bài 51 (Trích đềvào 10Chun Vinh năm học 2018-2019)

Cho sốtựnhiên n≥2và sốnguyên tố pthỏa mãn p−1chia hết cho nđồng thời

1

n − chia

hết cho p Chứng minh n+plà sốchính phương

Bài 52 (Trích đềvào 10Chuyên Quảng Nam năm học 2018-2019)

Tìm hai số nguyên tố p q, biết p+q p+4q số phương Bài 53 (Trích đềvào 10Chuyên Hải Dương năm học 2018-2019)

Tìm tất cảcác sốtựnhiên n k, để n8+42k+1là sốnguyên tố Bài 54 (Trích đềvào 10Chuyên Vĩnh Long năm học 2018-2019)

Tìm số tự nhiên x thỏa mãn biểu thức

14 49

= − + + +

P x x x số nguyên tố

Bài 55 (Trích đềvào 10Chuyên Phú Thọ năm học 2015-2016)

Chứng minh số nguyên lớn thoả mãn số nguyên tố chia hết cho

Bài 56 (Trích đềvào 10Chuyên Amsterdam năm học 2014-2015)

1) Cho số nguyên dương n thỏa mãn n 10 hai số nguyên tố Chứng minh

2) Tìm tất cảcác sốnguyên tốp sốnguyên dương x,y thỏa mãn 2 ( 2)

1 ( 2)

p x x

p y y

− = +

 

− = +

Bài 57 (Trích đềvào 10Chuyên TP Hồ Chí Minh năm học 2014-2015)

Cho sốnguyêndương a, b, c cho 1

a+ =b c

a) Chứng minh a + b không thểlà sốnguyên tố

b) Chứng minh c > a + c b + c không thểđồng thời sốnguyên tố Bài 58 (Trích đềvào 10Chun Thái Bình năm học 2014-2015)

2

p q+p p2+q

2

pq +q q2− p

abc ax2 +bx+ =c 0

n n24 n216

n

4

(n −1) 40

(72)

Cho a, b, c, d sốnguyên dương thỏa mãn: a2+ ab + b2 = c2+ cd + d2 Chứng minh a + b + c + d hợp số

Bài 59 (Trích đềHSG lớp Gia Viễn năm học 2014-2015)

Tìm sốtựnhiên nđể plà sốnguyên tốbiết: p=n3−n2 + −n 1 Bài 60 (Trích đềHSG lớp Thanh Chương năm học 2012-2013)

Chứng minh ∀ ∈n *thì n3 + +n 2là hợp số Bài 61 (Trích đềHSG lớp Bắc Ninh năm học 2018-2019)

Cho a b c, , sốnguyên khác 0, acsao cho

2

2 .

a b a b c c

+ =

+ Chứng minh

2 2

a +b +c khơng phải sốngun tố

Bài 62 (Trích đềHSG lớp Trực Ninh năm học 2017-2018)

Cho pvà 2p+1là sốnguyên tốlớn Chứng minh 4p+1là hợp số Bài 63 (Trích đềHSG lớp 8)

Cho số nguyên tố p > Biết có số tự nhiên n cho cách viết thập phân số pn có 20 chữ số Chứng minh 20 chữ số có chữ số giống

Bài 64 (Trích đềHSG lớp Triệu Sơn 2016-2017)

Một số nguyên tố p chia cho 42 có số dư r hợp số Tìm hợp số r Bài 65 (Trích đềHSG lớp Hoằng Hóa 2018-2019)

Tìm tất số nguyên tố , p q cho 7p+q pq+11 số nguyên tố Bài 66 (Trích đềHSG lớp Sơng Lơ 2018-2019)

Biết abcd nguyên tố có bốn chữ số thỏa mãn ab cd; sô snguyeen tố

và = + −

b cd b c Hãy tìm abcd

Bài 67 (Trích đềHSG lớp TP Bắc Ninh 2018-2019)

Cho số số nguyên tố ( ) Chứng minh ba sốp, q, r có hai sốbằng

Bài 68 (Trích đềHSG lớp Gia Bình 2018-2019)

Giả sử p p2+2 sốnguyên tố Chứng tỏ p3+p2+1cũng sốnguyên tố

Bài 69 (Trích đềHSG lớp Nghĩa Đàn 2018-2019)

Tìm hai số nguyên tố x y, thỏa mãn 2

45

xy =

Bài 70 (Trích đềHSG lớp Như Thanh 2018-2019)

1) Chứng minh hai số 2n+1 10n+7 hai sốnguyên tốcùng với

sốtựnhiên n

2) Tìm số x, ynguyên tốđể

2

23

x + =y

Bài 71 (Trích đềHSG lớp Nông Cống 2018-2019)

, ,

c b a

p= +b a q=a +c r=c +b a b c, , ∈N*

(73)

Tìm sốnguyên tốab a( > >b 0), biết ab ba− sốchính phương

Bài 72 Tìm tất số nguyên tố p để 4p 12 + và 6p 12+ cũng số nguyên tố. Bài 73 Chứng minh 2 1n − là số nguyên tố (n 2> ) thì 2 1n+ là hợp số.

Bài 74 Cho p, q, r, s số nguyên tố lớn Chứng minh p q r s2− 2+ −2 chia hết cho 24

Bài 75 Tìm tất cặp số nguyên tố cho p 2q2− =1

Bài 76 Chứng minh với số nguyên tố p p3+p 1−

2 khơng phải tích hai

số tự nhiên liên tiếp

Bài 77 Tìm số nguyên tố p, q, r thỏa mãn p qq+ p =r

Bài 78 Tìm số nguyên tố p,q,r thỏa mãn điều kiện sau:

≤ < < ≤ 2− 2 − ≤2 p q r; 49 2p r ; 2q r 193

Bài 79 Tìm tất ba số nguyên tố a,b,c đôi khác thoả mãn điều kiện

( )

< + + <

20abc 30 ab bc ca 21abc

Bài 80 Tìm số nguyến tố p, q số nguyên x thỏa mãn x px 3q 05+ + =

Bài 81.Tìm số nguyên tố p để p 1+

2

+

2

p

2 số phương

Bài 82 Chứng minh tồn số nguyên dương x thỏa mãn (x 2x 1+ )( + )

2012 số phương x hợp số

Bài 83 Tìm tất số nguyên tố p cho p p 22− −

2 lập phương số tự nhiên

Bài 84 Cho bảy số nguyên tố khác a,b,c,a b c,a b c,a c b,b c a+ + + − + − + − hai ba số a, b, c có tổng 800 Gọi d hiệu số lớn số nhỏ bảy số nguyên tố Hỏi giá trị lớn củad nhận

Bài 85 Cho số nguyên tố p Giả sử x y số tự nhiên khác thỏa mãn điều kiện

+

2

x py

xy số tự nhiên Chứng minh

+ = +

2

x py p 1 xy

Bài 86 Tìm số nguyên dương n lớn cho số 2016 viết thành

+ + + +

1 n

a a a a số a ;a ;a ; ;a1 2 3 n hợp số Kết thay đổi thay số 2016 số 2017

Bài 87 Tìm tất số nguyên tố p, q, r thỏa mãn phương trình (p q r 3+ )( + )( + )=4pqr

( )p;q

(74)

Bài 88 Cho số tự nhiên n , xét số ≥ a ;a ; ;a1 2 n số nguyên tố phân biệt

1 n

p ; p ; ; p thỏa mãn điều kiện p a a1 1− 2 =p a a2 2− 3 = = p an n−a1 Chứng minh

= = =

1 n

a a a

Bài 89 Tồntại hay không số nguyên tố a, b, c thỏa mãn điều kiện ab+2011 c=

Bài 90.Tìm tất số nguyên tố p cho với số ngun tố p ln tồn số nguyên dương n, x, y thỏa mãn pn =x3+y3

Bài 91 Tìm tất số nguyên dương n cho phần nguyên số nguyên tố

Bài 92 Cho p số nguyên tố Tìm số nguyên k cho k kp2− là số

nguyên dương

Bài 93 Tìm tất số nguyên tố p q thỏa mãn p q3− =(p q+ )2

Bài 94 Cho a, b số nguyên p số nguyên tố lẻ Chứng minh p4 là ước a2+b2 và a a b( + )2 thì p4 cũng ước a a b( + )

Bài 95 Tìm số ngun khơng âm a,b cho a2−b 5a 3b 42− + + là số nguyên tố.

Bài 96 Cho đa thức f x( )= ax3+bx cx d2+ + với a số nguyên dương Biết

( ) ( )=

f – f 2012 Chứng minh f – f 2( ) ( ) hợp số

Bài 97 Cho đa thức bậc ba f(x) với hệ số x3 là số nguyên dương biết

− =

f(5) f(3) 2010 Chứng minh f(7) f(1)− hợp số

Bài 98 Tìm tất ba số nguyên dương (m; p;q) cho p, q số nguyên tố

+ =

m

2 p q

Bài 99.Tìm sáu số nguyên tố p ; p ; p ; p ; p ; p1 2 3 4 5 6 thỏa mãn 2+ + 2+ + =

1

p p p p p p Bài 100 Cho số nguyên tố p dạng 4k Tồn hay không số nguyên a thỏa điều +

kiện (a p2+ )

Bài 101.Tìm để:

a) sốnguyên tố b) sốnguyên tố Bài 102

a) Tìm sốnguyên sốp để2p + lập phương sốtựnhiên b) Tìm sốnguyên tốp để13p + lập phương sốtựnhên Bài 103.Tìm tất cảcác sốnguyên tố thỏa

Bài 104.Tìm sốnguyên tố thỏa

Bài 105 Chứng minh sốnguyên tốthì với

+ +

3

n 8n 3n

*

nN

4

4

n + n2003+n2002+1

,

x y x2−2y2 =1 , ,

x y z xy + =1 z

1 2+ n +4 (n nN*) n=3k kN

(75)

Bài 106 Cho thỏa mãn Chứng minh rằng: hợp sốvới

Bài 107 Tìm tất cảcác sốnguyên tốp dạng ( )

Bài 108.Tìm tất cảcác sốcó hai chữ số cho sốnguyên tố

Bài 109.a) Cho số nguyên tố(gọi nguyên tốFermat) Chứng minh k = k = 2n

b) Cho 2k - 1 số nguyên tố (gọi số nguyên tố Mersenne) Chứng minh k số nguyên tố

Bài 110 Tìm k để 10 số tự nhiên liên tiếp: k + 1; k + 2; k + 3; k + 10 có nhiều số nguyên tố

Bài 111. Chứng minh rằng: (p – 1)! chia hết cho p p hợp số, không chia hết cho p p số nguyên tố

Bài 112 Chứng minh rằng: ước nguyên tố 1994! – lớn 1994

Bài 113 Chứng minh rằng: n > n n! có số nguyên tố (từ suy có vô số số nguyên tố)

Bài 114 Giả sửp sốnguyên tố lẻvà Chứng minh m hợp sốlẻkhông chia hết cho (mod m)

Bài 115. Chứng minh dãy số với chứa vô hạn sốlà lũy thừa sốnguyên tố

Bài 116.Tìm bảy số ngun tốsao cho tích chúng tổng lũy thừa bậc sáu bảy sốđó

Bài 117 Giải phương trình nghiệm nguyên (1)

Bài 118 Tìm tất cảcác cặp sốnguyên dương ( ) cho số nguyên dương ước sốcủa 1995

Bài 119 Một xí nghiệp điện tử ngày giao cho cửa hàng sốmáy tivi Số máy sốcó ba chữ sốmà tăng chữ sốđầu lên n lần, giảm chữ sốthứ hai thứba n lần sẽđược số lớn gấp n lần số máy giao Tìm n số máy tivi giao

Bài 120.Tìm số ngun tố cho tích chúng gấp lần tổng chúng

Bài 121. Cho n sốnguyên dương Chứng minh tồn n sốtựnhiên liên tiếp

sao cho chúng hợp số

Bài 122. Cho sốnguyên dương n thỏa mãn 2n −1là sốnguyên tố Chứng minh n

sốnguyên tố

Bài 123.Tìm sốnguyên tố p đề 2p+ p2 sốnguyên tố

, , , *

a b c dN ab=cd n n n n

A=a +b +c +d

nN

( 1)

n n+ −

1

n

ab ab

ab

2k +1

9

8 p

m= −

1

3m− ≡1

2003 23+ k k =1, 2,

2

7

xy =

,

x y

2

x y

x y

+ −

(76)

Bài 124. Cho p q, số ngun tốvà phương trình x2−px+ =q có nghiệm ngun

dương Tìm p q

Bài 125 Cho p q r, , sốnguyên tốvà n sốtựnhiên thỏa pn+qn =r2 Chứng

minh n=1

Bài 126. Cho p sốnguyên tốdạng 4k+3 Chứng minh x2+y2 chia hết cho p

x y chia hết cho p

Bài 127.Tìm sốtựnhiên m n, cho x=33m2+ −6n 61+4 sốnguyên tố

Bài 128.Tìm tất cảcác sốtựnhiên a,b,c cho 3

3

a +b + −c abc sốnguyên tố

Bài 129.Tìm sốnguyên tố a b c, , cho ab bc ca+ + >abc

Bài 130.Tìm sốnguyên tố a b c, , sốnguyên dương k cho 2 2

16

a +b + c = k +

Bài 131.Tìm sốnguyên tố p q cho p2|q3+1 q2|p6−1

Bài 132.Ta gọi p, qlà hai sốnguyên tốliên tiếp, p q khơng có sốngun tố khác Tìm ba sốnguyên tốliên tiếp p, q, r cho p q2+ 2+r2 cũng sốnguyên tố

Bài 133 Cho số 512525

5

A= −

− Chứng minh A hợp số

Bài 134 Chopp+2 số nguyên tố (p>3 ) Chứng minh rằngp+1 6

Bài 135 Cho p p+4là số nguyên tố (p>3) Chứng minh p+8 hợp số Bài 136.(Chuyên Vũng Tàu 2016-2017)

Tìm cặp số nguyên tố (p q, ) thỏa mãn 2

5

pq =

Bài 137. Chứng minh 12 sốnguyên tốphân biệt chọn sốký

hiệu p1, p2, p3, p4, p5, p6 cho (p1−p2)(p4−p3)(p5+p6)1800

Bài 138.(Đềthi HSG Toán TP.HCM năm học 2004 – 2005)

Tìm tất cảcác sốnguyên dương n cho phần nguyên

3

8

3

n n

n

+ + là số

nguyên tố

Bài 139. Cho p q, hai số nguyên tố cho p> >q p− =q Chứng minh rằng: (p+q)12

(77)

Chương IV

PHƯƠNG TRÌNH NGHIỆM NGUYÊN A KiÕn thøc cÇn nhí

1 Giải phương trình nghiệm nguyên

Giải phương trình f(x, y, z, ) = 0chứa ẩn x, y, z, với nghiệm nguyên tìm tất số nguyên (x, y, z, ) thỏa mãn phương trình

2 Một số lưu ý giải phương trình nghiệm nguyên

Khi giải phương trình nghiệm ngun cần vận dụng linh hoạt tính chất chia hết, đồng dư, tính chẵn lẻ,… để tìm điểm đặc biệt ẩn số biểu thức chứa ẩn phương trình, từ đưa phương trình dạng mà ta biết cách giải đưa phương trình đơn giản Các phương pháp thường dùng để giải phương trình nghiệm ngun là:

• Phương pháp dùng tính chất chia hết

• Phương pháp xét số dư vế

• Phương pháp sử dụng bất đẳng thức

• Phương pháp dùng tính chất số phương

• Phương pháp lùi vơ hạn, ngun tắc cực hạn

B MỘT SỐ PHƯƠNG PHÁP GIẢI PHƯƠNG TRÌNH NGHIỆM NGUYÊN

I. PHƯƠNG PHÁP DÙNG TÍNH CHIA HẾT

Dạng 1: Phát tính chia hết ẩn

Bài tốn Giải phương trình nghiệm nguyên 3x 17y 159+ = ( )1 Hướng dẫn giải

Giả sửx, y số nguyên thỏa mãn phương trình (1) Ta thấy 159 3x chia hết 17y 3 ⇒y 3 (do 17 nguyên tốcùng nhau)

Đặt y 3t t Z= ( ∈ ) thay vào phương trình ta 3x 17.3t 159+ = ⇔ +x 17t 53.= Do đó: x 53 17t(t Z)

y 3t

 = −

∈  =

 Thửlại ta thấy thỏa mãn phương trình cho

Vậy phương trình có nghiệm (x, y) = (53 – 17t, 3t) với t sốnguyên tùy ý

(78)

Bài tốn Tìm nghiệm nguyên phương trình 2x 13y 156+ = (1)

Hướng dẫn giải

- Phương pháp 1: Ta có 13y 13 156 13 nên 2x 13 ⇒x 13 (vì (2,3) = 1) Đặt x 13k (k Z)= ∈ thay vào (1) ta được: y= −2k 12+

Vậy nghiệm nguyên phương trình là: x 13k (k Z) y 2k 12

 =

∈  = − +

- Phương pháp 2: Từ(1) x 156 13y 78 13y

2

⇒ = = − ,

Để x Z 13y Z

∈ ⇒ ∈ Mà (13,2) = 1⇒y 2 Đặt y 2t(t Z)= ∈ ⇒ =x 78 13t−

Vậy nghiệm nguyên phương trình là: x 78 13t(t Z) y 2t

 = −

∈  = −

Chú ý: Phương trình có dạng ax by c+ = với a,b,c sốnguyên. * Phương pháp giải:

- Phương pháp 1: Xét tính chia hết hạng tử.

- Phương pháp 2: Khửẩn, sửdụng tính chia hết tìm điều kiện để phân sốtrởthành số ngun.

Bài tốn 3. Giải phương trình nghiệm ngun 23x 53y 109+ = .

Hướng dẫn giải

Ta có x 109 53y 23(4 2y) 17 7y 2y 17 7y

23 23 23

− − + − −

= = = − +

Ta phải biến đổi tiếp phân số 17 7y 23

đểsao cho hệ sốcủa biến y

Phân tích:Ta thêm, bớt vào tử sốmột bội thích hợp 23 17 7y 17 7y 46 46 7(9 y) 46 2 7(9 y)

23 23 23 23

− − + − − − −

= = = − +

Từđó x 2y 7(9 y) 23

= − + , Để x Z y Z 23

∈ ⇒ ∈ , (7,23) = Đặt y 23t (t Z)− = ∈ ⇒ = −y 23t

Vậy nghiệm nguyên phương trình là: x 23t (t Z) y 53t 16

 = −

∈  = − 

(79)

Bài tốn Tìm nghiệm ngun phương trình 11x 18y 120+ = ( )1

Hướng dẫn giải

Ta thấy 11x 6 ⇒x 6 suy x 6k k Z= ( ∈ ) thay vào (1) rút gọn ta được: 11k 3y 20+ =

Biểu thịẩn mà hệ sốcủa có giá trịtuyệt đối nhỏ(là y) theo k ta được:y 20 11k

=

Tách riêng giá trịnguyên biểu thức này: y 4k k

− = − + Lại đặt: k t t Z k 3t 1( )

3

− = ∈ ⇒ = +

Do đó: y 3t t 11t;= − ( + + = −) x 6k 3t 18t 6= = ( + =) + Thay biểu thức vào phương trình (1) thấy thỏa mãn

Vậy nghiệm phưng trình (x, y) = (18t + 6; 3-11t) với t ∈Z

Chú ý: a) Nếu đềbài yêu cầu tìm nghiệm nguyên dương phương trình (1) sau tìm nghiệm tổng quát ta giải điều kiện:

18t t

3 11t 11

 + >

⇔ − < <  − >

Do t = t sốnguyên Nghiệm nguyên dương (1) (x, y) = (6, 3).

Trong trường hợp tìm nghiệm ngun dương (1) ta cịn giải sau: 11x + 18y = 120 Do y 1≥ nên 11x 120 18.1 102.≤ − =

Do x nguyên nên x 9≤ Mặt khác x 6 và x nguyên dương nên x = 6⇒ =y 3

b) Có nhiều cách tách giá trịnguyên biểu thức y= 20 11k− ,

3 chẳng hạn: k

y 4k

= − + (cách 1)

1 2k y 3k

3

+

= − − (cách 2)

( )

2 k y 3k

3

= − + (cách 3)

Ta thấy: - Cách gọn cách ở cách hệ số k phân thức 1, sau đặt

k t

= ta không cần thêm ẩn phụnào

- Trong cách 3, nhờđặt thừa sốchung mà hệ số k phần phân số -1, sau đặt 1 k t

3

= cũng không cần dùng thêm thừa sốphụnào nữa.

(80)

Bài toán Tìm nghiệm nguyên dương phương trình: 6x2+5y2 =74

Hướng dẫn giải Ta có: 6x2 +5y2 =74⇔6 x( 2−4) (=5 10 y− 2)( )2

Từ (2) suy 6 x 5( 2− ) , mặt khác ( )6,5 1= ⇒(x 52− ) ⇒x2 =5t t N+ ( ∈ )

Thay x 5t2− = vào (2) ta có: 30t 10 y= ( − 2)⇔y2 =10 6t−

Ta có: 2

4 t

5t 5

x 0,y t ,t N

10t t 5 3

 > − 

 + > 

> > ⇔ ⇔ ⇔ − < < ∈

− >

  <



Suy ra: t∈{ }0;1 Với t = không thỏa mãn yêu cầu toán

Với t = ta có: x22 x

y y

 =  = ±

 ⇔

 =  = ±

 

 Mặt khác x, y nguyên dương nên x = 3, y =

Vậy phương trình có nghiệm (x, y) = (3, 2)

Dạng 2: Phương pháp đưa phương trình ước số * Cơ sở phương pháp:

Ta tìm cách đưa phương trình cho thành phương trình có vếlà tích biểu thức có

giá trịngun, vếphải sốnguyên.

Thực chất biến đổi phương trình vềdạng: A(x; y).B(x; y) c= trong A(x; y),B(x; y)

là biểu thức nguyên, c sốnguyên.

Xét trường hợpA(x; y),B(x; y) theo ước c.

* Ví dụ minh họa:

Bài tốn 1.Tìm nghiệm ngun phương trình: 2xy x y 3− + = Hướngdẫn giải

( ) ( )

( )( )

− + =

⇔ − + =

⇔ − + − = −

⇔ − + =

2xy x y 4xy 2x 2y

2x 2y 2y 2y 2x

Ta gọi phương trình phương trình ước số: vếtrái tích thừa số nguyên, vếtrái số Ta có x y sốnguyên nên 2x + 2y – số nguyên ước

(2x + 1) (2y - 1) làcác ước sốcủa nên ta có:

(81)

2x + 1 -1 -5

2y - -5 -1

Vập phương trình có nguyện nguyên (x, y) = (3, 0); (-1, -2); (2, 1); (-3, 0)

Kinh nghiệm giải: Để đưa vế trái 2xy x y− + về phương trình dạng tích, ta biến đổi

thành x 2y 1( ) (1 2y 1)

− + − cách nhân vế phương trình với bớt ca1 để đưa về phương trình ước số Luyện tập kinh nghiệm ví dụ2 sau đây.

Bài tốn 2.Tìm nghiệm ngun phương trình: 5x 3y 2xy 11− = −

Hướng dẫn giải

( ) ( ) ( )( )

− = − ⇒ − + − − + =

  − +

⇔ −  + = ⇔ − = ⇔ − + =

 

3 15

5x 3y 2xy 11 x(5 2y) (5 2y) 11

2

3 2x

5 2y x 2y 2y 2x (*)

2 2

(2x + 3) (2y - 5) ước sốcủa nên ta có:

2x + -1 -7

2y - -7 -1

Vập phương trình có nguyện nguyên (x, y) = (-1, 6); (-2, -1); (2, 3); (-5, 2)

Nhận xét: Đối với nhiều phương trình nghiệm nguyên việc đưa phương trình cho thành phương trình có vếlà tích biểu thức có giá trịnguyên, vếphải sốnguyên khó khăn ta có thểáp dụng sốthủthuật thể ví dụ3sau đây.

Bài tốn 3.Tìm nghiệm nguyên phươngtrình: x2 −2xy 3y 5x 0+ − + = .

Hướng dẫn giải

2

2

2 2 2

(2y 5) (2y 5)

x 2xy 3y 5x x x(2y 5) 3y

4

2y 4y 20y 25 12y 28 2y 4y 8y

x x

2 4

+ − +

− + − + = ⇔ − + + + + + =

 +  − − − + +  +  + −

⇔ −  + = ⇔ −  −

   

2 2

2

2y 4(y 1) 2y

x x (y 1)

2 4

 +  + −  +  −

⇔ −  − = ⇔ −  − + =

   

( ) ( ) ( )

( )( ) ( )( )

− − −

⇔ − + = ⇔ − − − + = −

⇔ − − − − − − + + = − ⇔ − − − = −

2

2

2

2x 2y 7

(y 1) 2x 2y y

4

2x 2y 2y 2x 2y 2y 2x 4y 2x (*)

(82)

Vì x, y nguyên nên từPT(*) ta có trường hợp sau: 1) 2x 4y

2x

 − − =  − = −  x y  = − ⇔  = −

 2)

2x 4y 7 2x

 − − = −  − =  x y  = ⇔  = 

3) 2x 4y 2x

 − − = −  − =  x y  = ⇔  =

 4)

2x 4y 7 2x

 − − =  − = −  x y  = ⇔  = − 

Vậy nghiệm nguyên (x;y) phương trình là: (-2; -3); (2; 1); (5; 1);(1; -3)

*Nhận xét: Trong cách giải ta sử dụng phương pháp biến đổi tam thức bậc hai (ax2+bxy cy ,ax+ 2+bx c+ )): trước hết ta chọn một biến để đưa về hằng đẳng thức (Bình phương tổng, hiệu) chứa biến đó: ở đây ta chọn biến x là :

2 (2y 5)

x x(2y 5)

4

+

− + + , phần lại đa thức ta lại làm với biến y:

(2y 5) 3y 7

− +

+ + 4y 8y 32

4

+ −

= − 4(y 1) 72

4

+ −

= −

Các bạn có thểtư tìm hướng giải sau:

( ) ( )

2

x 2xy 3y 5x 0− + − + = ⇔x − 2y x 3y a a *+ + + + = Xét phương trình: x2−(2y x 3y a * *+ ) + + + = ( )

Với a sốchưa biết cần thêm vào, xác định a sau:

( ) ( ) ( )

2 **

2

2y 3y a 4y 20y 25 12y 28 4a 4y 8y 4a

∆ = + − + +

= + + − − −

= + − −

Chọn a để ∆( )** sốchính phương nên 4a a

− − = ⇒ = .khi :

( ) ( ) ( ) ( )

2

1

**

2y x 2y x 4y

4 x x ,x

2 2

+ − + + + + +

∆ = + ⇒ = = = =

Vậy: ( )* x x 4y 7 (2x 2x 4y 7)( )

2

 +   

⇔ −  − = − ⇔ − − − = −

  

Vì x, y nguyên nên ta có trường hợp sau: 1) 2x 4y

2x

 − − =  − = −  x y  = − ⇔  = −

 2)

2x 4y 7 2x

 − − = −  − =  x y  = ⇔  = 

3) 2x 4y 2x

 − − = −  − =  x y  = ⇔  =

 4)

2x 4y 7 2x

 − − =  − = −  x y  = ⇔  = − 

Vậy nghiệm nguyên (x;y) phương trình là: (-2; -3); (2; 1); (5; 1);(1; -3)

(83)

Bài tốn 4.Tìm nghiệm ngun phương trình x 12x y2+ = ( )1

Hướng dẫn giải

Phương trình tương đương với :

( )2 ( )( )

2 2

x 12x y+ = ⇔ x 6+ −y =36⇔ x y x y 6+ + − + =36

Suy (x + y + 6) (x – y + 6) ước 36

Mà 36 có 18 ước nên: (x y 6+ + ∈ ± ± ± ± ± ± ±) { 1; 2; 3; 4; 6; 9; 18; 36± }

Kết quảta tìm nghiệm nguyên là:( ) (0,0 ; 12,0 ; 16,8 ; 16, ; 4,8 ; 4, 8− ) (− ) (− − ) ( ) ( − )

Nhận xét: Phương pháp đưa vềphương trình ước sốcó bước: Phân tích thành ước xét các trường hợp Hai bước có thểkhơng khó trường hợp sốphải xét có nhiều

ước số chúng ta cần dựa vào tính chất biến (ví dụ: tính chẵn lẻ, số dư vế) để giảm số trường hợp cần xét.

Trong trường hợp ví dụ 4ta nhận xét sau:

Do y có số mũ chẵn nên y nghiệm – y nghiệm nên ta giả sử y 0≥ Khi x y x y+ − ≤ + + ta giảm trường hợp

x y x y x y

, ,

x y x y x y 36

x y 36 x y x y 18

, ,

x y x y 18 x y

x y x y 12 x y 6

, ,

x y 12 x y x y 6

x y 6 x y 6

 + + =  + + = −  + + = −  + − =  + − = −  + − = −

  

 + + =  + + = −  + + =  − + =  + − = −  + − =

  

 + + = −  + + =  + + = −  + − = −  + − =  + − = −

  

 + + =  + − = 

Bây giờ có 10 trường hợp, ta lại thấy (x y+ + ) (+ x y+ − )=2y nên (x y , x y+ + ) ( + − )

cùng tính chẵn lẻ.Do ta cịn trường hợp:

x y x y 18 x y 6 x y 6

, , ,

x y 18 x y x y 6 x y 6

 + + = −  + + =  + + = −  + + =  + − = −  + − =  + − = −  + − =

   

Tiếp tục xét hai phương trình x y 6, x y 6 x y 6 x y 6

 + + = −  + + =  + − = −  + − =

  hai phương trình có nghiệm y = ta có xét y = từđầu Ta có phương trình ban đầu: x x 12( + )=y2 , xét hai khảnăng:

Nếu y = x = x = - 12

Nếu y 0≠ thì x y x y+ − < + + áp dụng hai nhận xét ta chỉphải xét trường hợp

(84)

x y x y 18 ,

x y 18 x y

 + + = −  + + =  + − = −  + − =

 

Giải kết luận phương trình có nghiệm ( ) (0,0 ; 12,0 ; 16,8 ; 16, ; 4,8 ; 4, 8− ) (− ) (− − ) ( ) ( − )

Dạng 3: Phương pháp tách giá trị nguyên

* Cơ sở phương pháp:

Trong nhiều tốnphương trình nghiệm ngunta tách phương trình ban đầu thành phần có giá trịngunđểdễdàng đánhgiátìm nghiệm, đa sốcác tốn sửdụng phương pháp này thường rút ẩn (có bậc nhất) theo ẩn cịn lại.

* Ví dụ minh họa:

Bài tốn Tìm nghiệm ngun dương phương trình sau: xy 2y 3y 0− − + =

Hướng dẫn giải

Ta có xy 2y 3y 0− − + = ⇒y x 3( − )=2x 1.−

Ta thấy x = không nghiệm nên x 3≠ đó: = − −

2x y

x

Tách ởphân thức −

2x

x giá trịnguyên:

( − )+ −

= = = +

− − −

2 x

2x

y

x x x

Do y sốnguyên nên

5

x sốnguyên, (x – 3) ước

+) x – = x = 4, y = + = +) x -3 = -1 x = 2, y = – = -3 (loại) +) x – = x = 8, y = +1 =

+) x – = -5 x = -2 (loại)

Vậy nghiệm (x, y) (4, 7) , (8, 3)

Bài tốn Tìm sốnguyên x y thoảmãn phương trình: x2 +xy 2y x 0− − − =

Hướng dẫn giải

Nhận xét: phương trình ẩn y có bậc nên rút y theo x Ta có: x2+xy 2y x 0− − − = ⇔y x 2( − )= − + +x2 x *( )

Với x = thì: ( )* ⇔ =0 (vô lý)

(85)

Với x 2≠ ta có: ( )* y x x 22 x

x x x

− + +

⇔ = + = − − +

− − −

Đểy nguyên x 2( − ) Vậy (x – 2) ước đó:

(x 2− ∈ − −) { 3, 1,1, 3}⇒ ∈ −x { 1,1,3,5}

Vậy phương trình có nghiệm: (x, y) = (3; - 1) ; (5; -5); (1; -5); (-1; - 1)

Bài tốn Tìm sốngun dương x, y cho 6x 5y 18 2xy+ + = (1)

Hướng dẫn giải

Ta có:

( )

5y 18 10y 36

x 2x

6 2y 2y

66 2y 66 33

2x 2x

6 2y 2y y

− − − −

= ⇔ =

− −

− + − − −

⇔ = = + ⇔ = +

− − −

Như x muốn nguyên dương (3 – y) phải ước – 33 Hay

(3 y− ) {∈ ± ± ±1; 3; 11; 33 ± } Lại y 1≥ ⇒ − ≤ ⇒ ∈ ± − −3 y y { 1; 3; 11; 33− } Ta có bảng sau:

3 - y -1 -3 -11 -33

y 14 36

x 19 - 14

Thửlại ta cặp thỏa mãn (19, 4); (8, 6); (4, 14); (3, 36)

Nhận xét: - Dễ xác định phương pháp để giải toán này, biểu diễn x theo y được x 5y 18

6 2y

− − =

Ta thấy biểu thức khó phân tích ví dụ trên, nhiên để ý ta thấy tử sốlà – 5y mẫu sốlà -2y, mạnh dạn nhân vào tử sốđểxuất 2y giống mẫu.

- Bài tốn giải phương pháp đưa vềphương trình ước số Do ởbài toán nhân ở x để biến đổi, phải có bước thử lại xem x, y có thỏa mãn phương trình cho hay khơng.

Bài tốn Tìm nghiệm nguyên phương trình: 2y x x y x2 + + + = 2+2y2+xy

Hướng dẫn giải

Ta có: 2y x x y x2 + + + = +2y2 +xy⇔2y x x x y x 1 12( − −) ( − −) ( − + =) ( ) Nhận thấy x = khơng nghiệm phương trình (1)

Chia cả2 vếcủa (1) cho (x – 1) ta được: 2y x y2 0 2( )

x

− − + =

(86)

PT có nghiệm x, y nguyên, suy

x 1− nguyên nên { }

x x 1;

x

 = − ∈ − ⇒ 

= 

Thay x = x = vào phương trình đểý đến y nguyên ta y = Vập phương trình cho có nghiệm (2; 1) (0; 1)

II. PHƯƠNG PHÁP SỬ DỤNG TÍNH CHẴN LẺ CỦA ẨN HOẶC XÉT SỐ DƯ TỪNG VẾ

* Cơ sở phương pháp:

Chúng ta dựa vào tính chẵn lẻ ẩn xét số dư hai vếcủa phương trình nghiệm nguyên với số nguyên dùng luận luận để giải tốn * Ví dụ minh họa:

Dạng 1: Sử dụng tính chẵn lẻ

Bài tốn 1.Tìm x, y ngun tố thoả mãn y2−2x2 =1

Hướng dẫn giải Ta có y2−2x2 = ⇒1 y2 =2x 12 + ⇒y số lẻ

Đặt y = 2k + (với k nguyên).Ta có (2k + 1)2 = 2x2 +

⇔ x2 = k2 + 2k ⇒ x chẵn , mà x nguyên tố ⇒ x = 2, y = Vậy nghiệm phương trình (x, y) = (2, 3) Bài tốn 2.Tìm nghiệm ngun dương phương trình (2x 5y 2+ + )( x + +y x2+x)=105

Hướng dẫn giải Ta có: (2x 5y 2+ + )( x + +y x2+x)=105

Ta thấy 105 lẻ ⇒ 2x + 5y + lẻ ⇒ 5y chẵn ⇒ y chẵn, 2|x| + y + x2 + x = 2|x| + y + x(x+ 1) lẻ có x(x+ 1) chẵn, y chẵn ⇒ 2|x| lẻ ⇒ 2|x| = ⇒ x =

Thay x = vào phương trình ta

(5y + 1) ( y + 1) = 105 ⇔ 5y2 + 6y – 104 = 0⇒ y = y = 26

− ( loại) Thử lại ta có x = 0; y = nghiệm phương trình

Vậy nghiệm phương trình (x, y) = (0, 4)

(87)

Dạng 2: Xét tính chẵn lẻ xét số dư vế

Bài tốn Chứng minh phương trình sau khơng có nghiệm ngun:

2 2

a) x −y =1998 b) x +y =1999

Hướng dẫn giải

a) Do x sốnguyên nên x 2k= x 2k k Z= + ( ∈ ) x2 =4k2∨x2 =4k2+4k 1+ x chia dư Tương tự2 ta có y2 chia ln dư 0

Suy ra: x2−y2 chia cho dư hoặc Mà 1998 chia cho dư phương trình cho khơng có nghiệm nguyên

b) Như chứng minh câu a ta có: x ,y2 chia cho ln dư nên x2+y2 chia cho dư hoặc Mà 1999 chia cho dư phương trình cho khơng có nghiệm nguyên

Chú ý: Chúng ta cần lưu ý kết quảởbài toán này:

2

*) x −y chia cho không dư 2

2

*) x +y chia cho không dư 3

Bài tốn Tìm nghiệm ngun phương trình: 9x y+ = +y Hướng dẫn giải

Ta có: 9x y+ = + ⇔y 9x y y 1+ = ( + )

Ta thấy vếtrái phương trình sốchia cho dư nên y y 1( + ) chia cho dư Do chỉcó thể y 3k 1= + y 3k k Z= + ( ∈ )

Khi đó: 9x 2+ =(3k 3k 2+ )( + )⇔9x 9k 9k= 2+ ⇔ =x k k 1( + ) Thửlại: x k k , y 3k 1= ( + ) = + thỏa mãn phương trình cho Vậy nghiệm phương trình ( )x,y =(k k ,3k 1( + ) + ) với k Z∈ Bài tốn Tìm x, y số tự nhiên thoả mãn x2 +3y =3026

Hướng dẫn giải Xét y 0= ⇒x2 +30 =3026⇒x2 =3025 Mà x ∈ N ⇒ x = 55

Xét y > ⇒ 3y chia hết cho 3, x2 chia cho dư 1⇒ x2 + 3y chia cho dư

mà 3026 chia cho dư (loại)

(88)

Vậy phương trình có nghiệm (x,y) = (55,0)

Bài toán Chứng minh phương trình x 7y 513− = khơng có nghiệm nguyên

Hướng dẫn giải Xét x 7k k Z= ( ∈ ) x 7.3

Xét x 7k k Z= ± ( ∈ ) x3 chia cho dư

Xét x 7k k Z= ± ( ∈ ) x3 chia cho dư

Xét x 7k k Z= ± ( ∈ ) x3 chia cho dư

Do vế trái phương trình chia cho dư hoặc vế phải phương trình chia dư Vậy phương trình cho vơ nghiệm

Bài tốn Tìm nghiệm nguyên phương trình x2−5y2 =27

Hướng dẫn giải

Do x sốnguyên nên ta biểu diễn x dạng: x 5k= x 5k 1= ± x 5k 2= ± với k Z∈

- Xét x = 5k x 5y2− =27⇔( )5k 2−5y2 =27⇔5 5k y( 2− 2)=27

Điều vơ lý vếtrái chia hết cho với k y nguyên vếphải không chia hết cho

- Xét x 5k 1= ± x 5y2 − =27⇔(5k 1± )2 −5y2 =27

( )

2 2

25k 10k 5y 27 5k 2k y 23

⇔ ± + − = ⇔ ± − =

Điều vô lý vế trái chia hết cho với k y ngun cịn vế phải khơng chia hết cho

- Xét x 5k 2= ± x 5y2− =27⇔(5k 2± )2−5y2 =27

( )

2 2

25k 10k 5y 27 5k 4k y 23

⇔ ± + − = ⇔ ± − =

Điều vơ lý vế trái chia hết cho với k y nguyên vế phải khơng chia hết cho

Vậy phương trình cho vô nghiệm

III. PHƯƠNG PHÁP DÙNG BẤT ĐẲNG THỨC

Dạng 1: Sử dụng bất đẳng thức cổ điển

* Cơ sở phương pháp:

(89)

Trong nhiều toán ta thường sửdụng bất đẳng thức để chứng minh vếkhông nhỏ hơn (hoặc khơng lớn hơn) vếcịn lại Muốn cho phương trình có nghiệm dấu bất đẳng

thức phải xảy nghiệm phương trình.

Một số bất đẳng thức Cổđiển thường sửdụng như: 1 Bất đẳng thức Cauchy (tên quốc tếlà AM – GM)

Nếu a ,a ,a , ,a1 2 3 n là sốthực không âm thì: n

1 n a a a a

a a a a n

+ + + + ≥ Đẳng thức xảy a1 =a2 =a3 = a= n

2 Bất đẳng thức Bunhiacopxki với hai sốthực (a ,a ,a , ,a1 2 3 n)(b ,b ,b , ,b1 2 3 n)ta

( 2 2)( 2 2) ( )2

1 n n 2 3 n n

a a a a+ + + + b b b b+ + + + ≥ a b a b a b a b+ + + +

Đẳng thức xảy tồn sốthực k (k 0≠ ) cho a kbi = i với i = 1, 2, 3,…, n.

* Ví dụ minh họa:

Bài tốn 1.Tìm sốngun dương x, y thỏa mãn phương trình: (x x2+ )( 2+y2)=4x y2

Hướng dẫn giải

Áp dụng bất đẳng thức AM-GM ta có:

x 2x+ ≥ Dấu “=” xảy x = 2

x +y ≥2xy Dấu “=” xảy x = y

Do x, y dương nên nhân vếcủa bất đẳng thức ta (x x2+ )( 2+y2)≥4x y2 Dấu xảy chỉkhi x = y =

Bài toán 2. Giải phương trình nghiệm nguyên dương sau:

( )3 ( )

6 2 2

x z 15x z 3x y z y+ − = − +5

Hướng dẫn giải

Ta có: ( )1 ⇔x z6+ 3+(y2+5)3 =15x z 3x y z2 + 2 ⇔x z6+ 3+(y2+5)3 =3x z y2 ( 2+5) Áp dụng bất đẳng thức AM-GM ta có: x z6+ 3+(y2 +5)3 ≥3x z y2 ( 2+5)

Dấu “=” xảy x2 =y2+ =5 z

Từ x2−y2 =(x y x y− )( + )=5 giải nghiệm (x, y, z) = (3, 2, 9).

Bài tốn 3. Giải phương trình nghiệm ngun sau (x y 1+ + )2 =3 x y 1( + 2+ )

(90)

Hướng dẫn giải

Áp dụng bất đẳng thức Bunhiacopxkita có: (1 1 x y 1+ + )( 2+ 2+ ≥) (x y 1+ + )2 Dấu “=” xảy 1 x y

x y 1= = ⇔ = =

Vậy nguyệm nguyên phương trình (x, y) = (1, 1)

Bài tốn 4.Tìm nghiệm nguyên phương trình: x2+xy y+ =x y 2

Hướng dẫn giải

Với x 2≥ y 2≥ ta có:

( ) −

 ≥

 ⇒ ≥ + = + + + + + > + +

 ≥

 ≥

2 2 AM GM

2 2 2 2 2 2

2 2

x y 4x x y 2 x y x y x y x y 2 xy x y xy.

x y 4y

Vậy x 2≤ y 2≤

Nếu x = -2 x = phương trình khơng có nghiệm nguyên Thửx = -1, 1, ta thấy phương trình có nghiệm (0;0), (1; - 1), (-1; 1)

Dạng 2: Sắp xếp thứ tự ẩn

* Cơ sở phương pháp:

Khi phương trình đối xứng với ẩn x y z, , , , ta thường giả sử x≤ ≤ ≤y z để

giới hạn miền nghiệm phương trình bắt đầu tìm từnghiệm bé trởđi * Ví dụ minh họa:

Bài tốn 1. Tìm nghiệm nguyên dương phương trình: 2xyz x y z= + +

Hướng dẫn giải

Giả sử x y z≤ ≤ Ta có: 2xyz x y z 3z= + + ≤

Chia vếcho z dương ta 2xy 3≤ ⇒xy xy 1≤ ⇒ =

Do x = y = Thay vào phương trình ban đầu ta được: 2z = z + hay z = Vậy nghiệm phương trình cho (x, y, z) = (1, 1, 2); (1, 2, 1); (2, 1, 1) Bài toán 2.Giải phương trình nghiệm nguyên dương: 1 1

x y z+ + =

Hướng dẫn giải

Do x, y, z có vai trị nên ta giả sử: x y z≤ ≤ Khi đó: 1= + + ≤ ⇒ ≤ ⇒ ∈1 1 x 3 x {1; 2; x Z} ( ∈ +)

x y z x

(91)

Với x = phương trình cho vơ nghiệm Với x = ta có: 1 1 y

2 y z y

= + + ≤ + ⇒ ≤ Mặt khác y x 2≥ = ⇒ ∈y {2,3,4} +) y = phương trình vơ nghiệm

+) y = z = +) y = z =

Với x = 3ta có: 1 1 y 3 y z y

= + + ≤ + ⇒ ≤ Mặt khác y x 3≥ = ⇒ = ⇒ =y z Vậy phương trình có nghiệm (x, y, z) = (2, 3, 6); (2, 4, 4); (3, 3, 3)

Bài toán 3. Giải phương trình nghiệm nguyên dương: 1 z x y+ =

Hướng dẫn giải

Biến đổi thành: xyz x y= +

Do đối xứng x y nên có thểgiảthiết x y≤ Ta có xyz x y y y 2y= + ≤ + = ⇒xz 2.≤

Ta lựa chọn nghiệm trường hợp sau: x = 1, z = 1; x = 2, z = 1; x =1, z = Ta suy nghiệm (x, y, z) (1, 1, 2) (2, 2, 1)

Nhận xét: bài tốn vai trị x, y, z khơng bình đẳng nên ta khơng giải sử

x y z≤ ≤ ta chỉcó thể giả sử x y≤

Bài toán 4. Tìm nghiệm nguyên dương phương trình: ( x + y + z + t ) + 10 = xyzt

Hướng dẫn giải Ta giả sử x ≥ y ≥ z ≥ t ≥

Ta có: ( x + y + z + t ) + 10 = xyzt

3

5 5 10 30

2 t 15 t t

yzt xzt xyt xyz xyzt t

⇔ = + + + + ≤ ⇒ ≤ ⇒ = ∨ = Với t = ta có:

( )

{ }

2

5 x y z 10 2xyz 5 15 30

2 z 15 z 1; 2; yz xz xy xyz z

+ + + + =

⇔ = + + + ≤ ⇒ ≤ ⇒ =

(92)

Nếu z =1 ta có x y 20 2xy( ) (2x 2y 5)( ) 65 x 35 x

y y

 =  =

+ + = ⇔ − − = ⇒ ∨

= =

 

Ta nghiệm (35, 3, 1, 1) ; (9, 5, 1, 1) hoán vị chúng Với z = 2, z = phương trình khơng có nghiệm ngun

Với t = ta có:

( )

( ) ( )( )

2

5 x y z 20 4xyz

5 5 20 35 35

4 z z t 8x 8y 265 yz xz xy xyz z

+ + + + =

⇔ = + + + ≤ ⇒ ≤ ≤ ≥ ≥ ⇒ − − =

Do x ≥ y ≥ z ≥ nên 8x – ≥ 8y – ≥ 11

⇒ (8x – 5) (8y – 5) = 265 vơ nghiệm

Vậy nghiệm phương trình (x, y, z)= ( 35; 3; 1; 1); (9; 5; 1; 1) hoán vị

Dạng 3: Chỉ nghiệm nguyên

* Cơ sở phương pháp: Chúng ta xét khoảng giá trị ẩn thể dạng: vài số nghiệm phương trình, chứng minh phương trình khơng cịn nghiệm khác

* Ví dụ minh họa:

Bài tốn 1.Tìm nghiệm ngun dương phương trình sau: 3 4x + x =5x

Hướng dẫn giải

Chia hai vếcủa phương trình cho 5x ta được:

x x

3 1 5

  +  =

   

   

Thửthấy x = không nghiệm phương trình Với x = VT = VP = thỏa mãn toán

Với x 3≥

x

3 5

   

⇒  ≤ 

   

x

4 5

   

   

   

x x 2

3 4 1 5 5

       

⇒  +  <  +  =

       

Vậy x = nghiệm phương trình

Bài tốn 2.Tìm nghiệm ngun dương phương trình sau: 2x +3x =35

Hướng dẫn giải Thử thấy x = 0; x = 1; x = không thỏa mãn 2x +3x =35

Với x = 23 +33 =35 (đúng)

Với x ≥ 23 +33 >35

Vậy x = nghiệm phương trình

(93)

Dạng 4: Sử dụng điều kiện ∆ ≥0 để phương trình bậc hai có nghiệm * Cơ sở phương pháp:

Ta viết phương trình f(x, y) = 0dưới dạng phương trình bậc hai ẩn, chẳng hạn x y tham số Điều kiện đểphương trình có nghiệm nguyên ∆ ≥0 * Ví dụ minh họa:

Bài tốn 1.Tìm nghiệm ngun phương trình x2+y2−2x y 9.+ =

Hướng dẫn giải

Ta xem phương trình cho phương trình ẩn x tham sốy, ta viết lại sau:

( )

2

x 2x y− + + −y =0

Đểphương trình cho có nghiệm :

( )

( )

2

2

' y y y y 10

4y 4y 40 2y 41

∆ ≥ ⇔ − + − ≥ ⇔ + − ≤

⇔ + − ≤ ⇔ + ≤

Do đó: (2y 1+ ) {2∈ 1;9; 25} Ta có:

2y+1 -1 -3 -5

2y -2 -4 -6

y -1 -2 -3

x Loại Loại Loại Loại -1 -1

Vậy nghiệm phương trình (x, y) = (3, 2); (-1, 2); (3, -3); (-1, -3)

Bài toán 2. Giải phương trình nghiệm nguyên x 2y2+ =2xy 2x 3y *+ + ( )

Hướng dẫn giải

Ta xem phương trình cho phương trình ẩn x tham sốy, ta viết lại sau:

( )

2

x y x 2y 3y 0− + + − =

Ta có: ∆ =' (y 1+ )2−(2y 3y2 − )=y 2y 2y 3y2 + + − 2+ = − +y 5y 12 + Đểphương trình có nghiệm ngun thì:

∆ ≥ ⇔ − + + ≥ ⇔ − ≤ − ≤

− + − +

⇔ ≤ ≤ ⇔ < <

2 29 29

' y 5y y

2 2

5 29 y 29 y

2 2

(94)

Vì y nguyên nên y∈{0,1,2,3,4,5} thay vào phương trình ta tính giá trịcủa x Giải ta nghiệm phương trình (x, y) = (0, 0); (0, 2)

Nhận xét: Ởví dụnày cốtình tính ∆' cho bạn thấy tính hoặc ∆'

có dạng tam thức bậc : f x( )=ay2+by c+ với a < ta áp dụng phương pháp này, a >

thì áp dụng phương pháp đưa vềphương trình ước số.

IV PHƯƠNG PHÁP DÙNG TÍNH CHẤT CỦA SỐ CHÍNH PHƯƠNG

Dạng 1: Dùng tính chất chia hết số phương * Cơ sở phương pháp:

- Số phương khơng thể có chữ tận 2, 3, 7, 8;

- Số phương chia hết cho sốngun tố p thìcũngchia hết cho

p

- Số phương chia cho có số dư 1; - Số phương chia có số dư 1; - Số phương chia cho có số dư 0, * Ví dụ minh họa:

Bài tốn 1.Tìm nghiệm ngun phương trình: 9x y y 1+ = ( + )

Hướng dẫn giải

Ta có:

( )

( ) ( )

+ = +

⇔ + = +

⇔ + = + +

⇔ + = +

2

2

9x y y 36x 20 4y 4y 36x 21 4y 4y 12x 2y

Số phương chia hết chia hết cho 9, ta lại có 12x + khơng chiahết 3(12x + 7) khơng chia hết cho Do phương trình vơ nghiệm

Cách khác:

( )

( ) ( )

+ = + ⇔ + − − =

∆ = + + = + = +

2

9x y y y y 9x

1 9x 36x 21 12x

Ta có ∆ chia hết cho không chia hết khơng số phương khơng tồn y ngun Vậy phương trình vơ nghiệm

Dạng 2: Biến đổi phương trình dạng 2 1 2 n n

a A +a A + +a A =k, ( 1, , )

i

A i= n đa thức hệ số nguyên,ai số nguyên dương, k số tự nhiên

(95)

* Cơ sở phương pháp:

Sửdụng đẳng thức đáng nhớ

(a b+ ) , đưa phương trình vềdạng Sau

dựa vào tính chất a Ai, i đểphân tích thành k=a k1 12+a k2 22+ + a kn n2 (với ki∈),

dẫn đến giải hệphương trình

2

1

2

2

2

n n

A k

A k

A k

 =  =     = 

* Ví dụ minh họa:

Bài tốn Tìm nghiệm ngun phương trình x2+y2− − =x y 8

Hướng dẫn giải

Ta có:

( ) ( ) ( ) ( )

2 2

2

2

2 2 2

x y x y 4x 4y 4x 4y 32

4x 4x 4y 4y 34 2x 2y 34

2x 2y

+ − − = ⇔ + − − =

⇔ − − + − + = ⇔ − + − =

⇔ − + − = +

Ta thấy 34 chỉcó dạng phân tích thành hai sốchính phương 32 và 52

Do đó:

( )

( )

( )

( )

2 2

2 2

2 2

2 2

2x 2x 3 2y 2y

2x 2x

2y 2y

 − = 

− = 

 

 − =  − =

 

 ⇒ 

 − =  − =

 

 − =  − =

 

Giải ta nghiệm (x, y) = (2, 3); (-1, -2); (-2; -1); (3, 2)

Bài tốn Giải phương trình nghiệm ngun x2 −4xy 5y+ =2(x y)− .

Hướng dẫn giải

Ta có x2−4xy 5y+ =2(x y)− ⇔x2−4xy 5y+ 2−2x 2y 0+ =

2 2

2 2

x 2x(2y 1) (2y 1) (2y 1) 5y 2y

(x 2y 1) y 2y (x 2y 1) (y 1) 2(*)

⇔ − + + + − + + + =

⇔ − − + − − = ⇔ − − + − =

Xét phương trình (*) ta có: (x 2y 1− − )2 ≥0 x,y∀ ⇒(y 1− )2 ≤2 Mà x nguyên nên (y 1− ) { }2∈ 0,1

* Với (y 1− )2 =0 (x 2y 1− − )2 =2 (loại)

(96)

* Với (y 1)2 y 1 y

y 1 y

 − =  = − = ⇒ ⇔

− = − =

 

- y = (x 1)2 x x

x x

 − =  = ⇒ − − = ⇒ ⇒

− = − =

 

- y = (x 1)2 x 1 x

x 1 x

 − =  = ⇒ − − = ⇒ ⇒

− = − =

 

Vậy phương trình cho có nghiệm: ( ) ( ) ( ) ( ) ( )x,y = 6,2 ; 4,2 ; 2,0 ; 0,0 Bài tốn Giải phương trình nghiệm ngun 5x2−2xy y+ =17.

Hướng dẫn giải

Ta có 5x 2xy y2− + =17⇔(x y− )2+4x2 =17 ⇔(x y)− =17 4x− (*) Xét phương trình (*) ta có (x y)2 0, x,y 17 4x2 0 x2 17

4

− ≥ ∀ ⇒ − ≥ ⇒ ≤

Mà x sốnguyên nên x2∈{0;1; 4} - Với x2 = ⇒0 (x y)− =17 (loại). - Với x2 = ⇒1 (x y)− =13 (loại) - Với x2 = ⇔ = ±4 x 2,

Với x 2 (2 y)2 1 y y

2 y y

 − =  = = ⇒ − = ⇔ ⇔

− = − =

 

Với x 2 (2 y)2 1 y y

2 y y

 + =  = − = − ⇒ + = ⇔ ⇔

+ = − = −

 

Vậy nghiệm nguyên phương trình là: (2; 1), (2; 3), (-2; -1); (-2; -3) Bài tốn 4.Tìm nghiệm nguyên phương trình x y xy x+ + = +y2

Hướng dẫn giải

Biến đổi: x y xy x+ + = +y2 ⇔(x 1− ) (2 + y 1− ) (2 + x y− )2 =2.

Tổng ba số phương nên tồn số Trường hợp: x – = ta (1; 0), (1; 2)

Trường hợp: y – = ta được: (0; 1), (2; 1) Trường hợp x – y = ta được: (0; 0), (2; 2)

Vậy phương trình có nghiệm (x, y) = (1; 0), (1; 2), (0; 1), (2; 1), (0;0), (2; 2) Bài toán Tìm nghiệm nguyên phương trình 2x2 +4x 19 3y = −

(97)

Hướng dẫn giải

( ) ( ) ( )

+ = −

⇔ + = −

2

2

2x 4x 19 3y

2 x y *

Ta thấy 3 y 2( − 2) ⇒ −7 y 22 ⇒y lẻ

Ta lại có 7 y− ≥0nên y2=1 Khi (*) có dạng x 1( + )2 =18

Ta được: x 1+ = ±3do đóx1 =2; x2 = −4

Các cặp số (2; 1), (2; -1), (-4; 1), (-4; -1) thỏa mãn (2) nên nghiệm phương trình cho

Dạng 3: Xét số phương liên tiếp

* Cơ sở phương pháp:

Phương pháp dựa nhận xét sau:

1 Không tồn n Z∈ thỏa mãn: a2 <n2 <(a 1+ )2 với a Z∈

2 Nếu a2 <n2 <(a 2+ )2với a,n Z∈ thì n = a + Tương tự với lũy thừa bậc 3

3 Nếu x x x n( + ) ( + ) (<y y y n+ ) ( + ) (< x a x a x a n+ )( + + ) ( + + )

Thì y y y n( + ) ( + ) (= x i x i x i n+ )( + + ) ( + + ) với i 1,2, ,a 1∈{ − } * Ví dụ minh họa:

Bài tốn 1.Tìm nghiệm ngun phương trình: 1 x x+ + +x3 =y3 ( )1

Hướng dẫn giải

Ta có:

2

2 11 19

x x x 0; 5x 11x x

2 10 20

   

+ + = +  + > + + =  +  + >

   

Nên

(1 x x+ + 2+x3) (− x2+ + < + +x 1 x x) 2+x3 < + +(1 x x2+x3) (+ 5x 11x 2+ + )

Do đó: x3 <y3 <(x 2+ )3 ⇒y3 =(x + )3

Kết hợp với (1) ta có: (x 1)3 1 x x x2 x x 0( ) x . x

 = + = + + + ⇒ + = ⇒ 

= − 

Nghiệm phương trình là: (0;1) (-1;0)

(98)

Bài tốn 2. Giải phương trình nghiệm nguyên: x y 2y 3y 03− 3− 2− − = ( )2

Hướng dẫn giải

( )2 ⇔x3 =y 2y3+ +3y 3+ ( )

Ta có: y2 ≥0; 5y2+ >2 0nên

(y3+2y2+3y 1+ −) (5y2+2)<y3+2y2 +3y 1+ ≤(y3+2y2+3y y + +) Do đó: (y 1− )3<x3 ≤(y 1+ )3 ⇒x3 =y3 hoặc x3 =(y + )3

Nếu x3 =y3 kết hợp với (3) ta có: 2y2+3y 0+ = ⇒ = − ⇒ = −y 1 x 1. Nếu x3 =(y + )3 Phối hợp với (3) ta có y2 = ⇒ =0 y 0 , lúc x = 1. Vậy nghiệm phương trình cho (-1; -1) (1; 0)

Bài toán 3. Giải phương trình nghiệm nguyên: x (x 1)2+ + =y4 +(y 1)+

Hướng dẫn giải

Biến đổi phương trình vềdạng

+ + = + + + + = + + = ∈

2 2 2

x x y (y 1) 2y(y 1) (y y 1) k ,k Z (1)

- Nếu x 0> ⇒x2 <x2+ + <x (x 1)+ ⇒x2 <k2 <(x 1)+ 2 khơng có sốnguyên k thỏa mãn. - Nếu x y2 y 1 1

x

 =

⇒ + + = ±  =

Ta có nghiệm ngun phương trình (0; 0), (0; -1), (-1; 0); (-1; -1)

- Nếu x< − ⇒1 (x 1)+ <x2+ + <x x2⇒(x 1)+ <k2 <x2 khơng có sốngun k thỏa mãn.

Bài tốn 4. Giải phương trình nghiệm nguyên

( )

4 2

x +x −y + +y 10 6=

Hướng dẫn giải

( )6 ⇔y y x( − =) 4+x 10 72+ ( )

Ta có: x4+x2 <x4+x 102 + <(x4+x 102 + ) (+ 6x2+2 )

(99)

Do đó: ( ) ( ) ( )( ) ( ) ( )( )

( ) ( )( )

2

2 2

2

y x x x x x y y x x

y y x x

 − = + +

 + < − < + + ⇒

 − = + +

Kết hợp với (7) ta suy ra: x22

x

 =

 =

Từđó: x= ±2, x= ±1

Do ta có thểtìm nghiệm phương trình (6)

Dạng 4: Sử dụng điều kiện là số phương * Cơ sở phương pháp:

Với phương trình nghiệm nguyên có dạngf x,y( )=0 có thể viết dạng phương trình

bậc ẩn chẳng hạn ẩn x, điều kiện ∆ ≥0 để phương trình có nghiệm

ngun phải sốchính phương Vận dụng điều ta giải tốn.

Chú ý: là số chính phương chỉ là điều kiện cần chưa đủ để phương trình có

nghiệm ngun, sau tìm giá trị cần thửlại vào phương trình ban đầu.

* Ví dụ minh họa:

Bài tốn 1. Giải phương trình nghiệm nguyên 3x2+y2+4xy 4x 2y 0+ + + =

Hướng dẫn giải

Ta có: 3x2+y2+4xy 4x 2y 0+ + + =

( ) ( )

2

y 2x y 3x 4x

⇔ + + + + + =

Coi phương trình (1) phương trình ẩn y tham sốx ta có:

( )2 ( 2 ) 2 2 2

' 2x 3x 4x 4x 4x 3x 4x x

∆ = + − + + = + + − − − = −

Đểphương trình có nguyện ngun ∆' phải sốchính phương hay ∆ =' x n2− = với n N∈

(x n x n− )( + )=4 giải ta x = x = -2 Với x = y =

Với x = -2 y = -5

Vậy phương trình có nghiệm (x, y) = (2, 3) ; (-2, -5)

Bài tốn Giải phương trình nghiệm ngun x y xy x 2y 12 2− = 2+ ( )

Hướng dẫn giải

(100)

Phương trình cho viết lại: (x y2− ) 2−xy x− =0 2( )

Do x nguyên nên (x2−2)≠0 coi phương trình (2) phương trình ẩn y tham sốx ta có:

( ) ( )

2 2 2

x 4x x x 4x

∆ = + − = −

Đểphương trình có nguyện ngun ∆ phải sốchính phương -Xét x = từ(1) suy y =

-Xét x 0≠ (4x 72− )phải sốchính phương 4x m2− = 2với m số nguyên, ta có (2x m 2x m− )( + )=7 ta tìm x = x = -2

Với x = thay vào (2) ta được: y2+ − = ⇒ ∈y 0 y 1; { − } Với x = -2 thay vào (2) ta được: y2− − = ⇒ ∈ −y 0 y { 1; }

Nghiệm nguyên phương trình (x, y) = (2, 1); (2, -2); (-2, -1); (-2, 2)

Dạng 5: Sử dụng tính chất: Nếu hai số nguyên liên tiếp có tích số

phương hai số ngun liên tiếp * Cơ sở phương pháp:

Giả sửa(a + 1) = k2(1) với a Z,k N.∈ ∈

Giải sửa ≠ 0, a + ≠ k2≠ Do k sốtựnhiên nên k > 0.

Từ(1) suy ra: a2+ a = k2

( ) ( )

⇒4a2 +4a 4k= ⇒4a2 +4a 4k+ = + ⇒1 2a 1+ =4k2 +1 2

Do k > nên 4k2 <4k2 + <1 4k2 +4k 1+ ( )3

Từ (2) (3) suy ( ) (2k < 2a 1+ ) (2 < 2k 1+ )2, vô lý

Vậy a(a + 1) = k2thì tồn hai số a, a + 0. * Ví dụ minh họa:

Bài tốn 1.Tìm nghiệm ngun phương trình: x2 +xy y+ =x y2

Hướng dẫn giải

Thêm xy vào hai vế: x2 +2xy y+ =x y2 +xy⇔(x y+ )2 =xy xy 1( + ) ( )*

Ta thấy xy xy + hai số ngun liên tiếp, có tích số phương nên tồn số

Xét xy = Từ (1) có x2+ y2= nên x = y =

Xét xy + = Ta có xy = -1 nên (x, y) = (1; -1), (-1; 1)

Thử lại ba cặp số (0; 0), (1; -1), (-1; 1) nghiệm phương trình cho

(101)

Bài tốn Tìmnghiệm ngun phương trình: x2 +2xy 5y 6= + ( )1

Hướng dẫn giải

Ta có ( )1 ⇔x2+2xy y+ =y2+5y 6+ ⇔(x 1+ ) (2 = y y 2+ )( + )

Do (y + 3) (y + 2) sốngun liên tiếp mà có tích số phương nên sốphải

Nếu y + = y = -3, x = -1 Nếu y + = y = -2, x = -1

Vậy phương trình có nghiệm ngun (x, y) = (-3, -1); (-2, -1)

Dạng 6: Sử dụng tính chất: Nếu hai số nguyên dương nguyên tố có tích một số phương số số phương

* Cơ sở phương pháp:

Giả sử ab = c2(1) với a,b,c N , a,b∈ * ( )=1.

Giả sửtrong a b có số, chẳng hạn a, chứa thừa số nguyên tốp với sốmũ lẻthì số b

không chứa thừa sốp nên c2 chứa thừa sốp với sốmũ lẻ, trái với giảthiết c2là sốchính phương. * Ví dụ minh họa:

Bài tốn 1.Tìm nghiệm ngundươngcủa phương trình: xy z= ( )1

Hướng dẫn giải

Trước hết ta có thểgiả sử(x, y, z) = Thật ba số (x ,y ,z0 0 0) thỏa mãn (1) có ƯCLN d, giả sử x0 =dx ,y1 0 =dy ,z1 0 =dz1thì (x ,y ,z1 1 1) nghiệm phương trình (1)

Với (x, y, z) = x, y, z đơi nguyên tốcùng nhau, hai ba sốx, y, z có ước chung d sốcịn lại chia hết cho d

Ta có z2= xy mà (x, y) = nên x = a2, y = b2với a,b N∈ * Suy z2= xy = (ab)2, z = ab

Như vậy:

 =  =   = 

2 x ta y tb z tab

với t sốnguyên dương tùy ý

Đảo lại, hiển nhiên sốx, y, z có dạng thỏa mãn (1) Công thức cho ta công thức nghiệm nguyên dương (1)

(102)

Bài tốn Tìm tất cảcác cặp sốnguyên (x; y) thỏa mãn

4 2

x −2x +6x −4y −32x 4y 39 0+ + =

Hướng dẫn giải

Ta có:

− + − − + + =

<=> − + − + = − + <=> − + + = −

4 2

4 2

2 2

x 2x 6x 4y 32x 4y 39 x 2x 6x 32x 40 4y 4y (x 2) (x 2x 10) (2y 1)

Vì y sốnguyên nên 2y – ≠ ⇒x ≠

Vì (2y – 1)2và (x – 2)2là sốchính phương khác nên x2+ 2x + 10 sốchính phương. Đặt x2+ 2x + 10=m2 (m N )∈ * suy ra (x 1) m+ 2+ = ⇔ + −(x m)(x m)+ + = −9 *( ) Do (x + + m) > (m + – m) nên

( )

x m x

x m m

x m x

*

x m m

x m x

x m m

 + + =  =  + − = −  =

 

 

 + + =  = −

 

⇔  ⇔ 

+ − = − =

 

 

 + + =  = −

 

 + − = −  =

 

•x = ⇒(2y – 1)2= 25 ⇒y = y = –2

•x = –5 ⇒(2y – 1)2= 1225 ⇒y = 18 y = –17

•x = –1 ⇒(2y – 1)2= 81 ⇒y = y = –4

Vậy bộ(x;y) nguyên thỏa yêu cầu tốn (3;3),(3;–2),(–5;18),(–5;–17),(–1;5),(–1;–4)

V PHƯƠNG PHÁP LÙI VƠ HẠN, NGUYÊN TẮC CỰC HẠN

Dạng 1: Phương pháp lùi vô hạn

* Cơ sở phương pháp:

Dùng để chứng minh phương trình f(x, y, z, ) nghiệm tầm thường

x = y = z = khơng cịn nghiệm khác Phương pháp diễn giải nhưsau:

Giải sử (x ,y ,z , 0 0 0 ) là nghiệm phương trình f(x, y, z, ), nhờphép biến đổi suy luận

ta tìm nghiệm khác (x ,y ,z , 1 1 1 )sao cho nghiệm có quan hệ với nghiệm ban đầu

tỷ sốk Ví dụ x0 =kx ,y1 0 =ky ,z1 0 =kz1 ;

Rồi từ bộ (x ,y ,z , 2 2 2 )có quan hệ với (x ,y ,z , 1 1 1 )bởi tỷ sốk đó.

(103)

Ví dụx1=kx ,y2 1 =ky ,z2 1=kz2 Q trình dẫn đến x ,y ,z , 0 0 0 chia hết cho ks vớs sốtự

nhiên tùy ý, điều xảy x = y = z = Chúng ta đến ví dụ cụthểnhư sau:

* Ví dụ minh họa:

Bài tốn 1. Giải phương trình nghiệm nguyên sau x2 +y2 =3z2

Hướng dẫn giải

Gọi (x ,y ,z0 0 0)là nghiệm phương trình Xét (mod 3) ta chứng minh x ,y0 0

chia hết cho Thật rõ ràng vế phải chia hết cho suy (x0+y 30) Ta có

( ) ( )

2

0

x ≡0;1 mod ; y ≡0;1 mod ( 2) 2

0 0

x +y 3 ⇒x 3,y 3 

Đặt x0 =3x ; y1 0 =3y ; z1 0 =3z1 thếvào rút gọn ta ( 2)

1 0

3 x +y =z ⇒z 3

0

z 3z

⇒ =

Thế z0 =3z1vào ( 2)

1

3 x +y =z rút gọn ta được: 2

1 1

x +y =z Do

(x ,y ,z0 0)là nghiệm phương trình (x ,y ,z1 1)cũng nghiệm phương trình Tiếp tục suy luận dẫn đến k

0 0

x ,y ,z 3 điều xảy x0 =y0 =z0 =0 Vậy phương trình có nghiệm (x, y, z) = (0, 0, 0)

Bài tốn Giải phương trình nghiệm nguyên sau: x2+y2+z2 =2xyz

Hướng dẫn giải

Gọi (x ,y ,z0 0 0)là nghiệm phương trình trên, ta có 2

0 0 0

x +y +z =2x y z suy

( 2 2)

0 0

x +y +z chẵn (do 2x y z0 0) nên có trường hợp xảy ra:

Trường hợp 1: Có sốlẻmột sốchẵn khơng tính tổng quát giả sử x ,y0 0 lẻ, z0 chẵn Xét mod ta có: 2 ( )

0 0

x +y +z ≡2 mod 2x y z 40 0 (do z0 chẵn) ⇒ Vô lý/

Trường hợp 2: Cả3 sốđềchẵn Đặt x0 =2x ,y1 0 =2y ,z1 0 =2z1 thếvào rút gọn ta có:

2 2

1 1 1

x +y +z =4x y z lập luận ta x ,y ,z1 1 1chẵn Quá trình tiếp tục đến k( *)

0 0

x ,y ,z k N ∈ điều xảy x0 =y0 =z0 =0

Vậy phương trình có nghiệm (x, y, z) = (0, 0, 0)

Dạng 1: Nguyên tắc cực hạn

* Cơ sở phương pháp:

(104)

Về hình thức phương pháp khác với phương pháp lùi vô hạn ý tưởng sử dụng nhau, chứng minh phương trình ngồi nghiệm tầm thường khơng cịn nghiệm khác

Phương pháp bắt đầu việc giả sử (x ,y ,z , 0 0 0 ) nghiệm phương trình f(x, y, z, .) với điều kiện buộc với (x ,y ,z , 0 0 0 ) Ví dụ x0 nhỏ

0 0

x +y +z + nhỏ Bằng phép biến đổi số học ta tìm nghiệm khác

(x ,y ,z , 1 )trái với điều kiện buộc Ví dụ tìm (x ,y ,z , 0 )với x0 nhỏnhất ta lại tìm (x ,y ,z , 1 1 1 )thỏa mãn x1 <x0 từ dẫn tới phương trình cho có nghiệm x0 =y0 =z0 =0

* Ví dụ minh họa:

Bài tốn 1. Giải phương trình nghiệm nguyên sau 8x 4y4+ 4+2z4 =t4 ( )1

Hướng dẫn giải

Giải sử (x ,y ,z0 0 0)là nghiệm phương trình với điều kiện x0nhỏnhất Từphương trình (1) suy t sốchẵn Đặt t 2t= 1 thếvào phương trình (1) rút gọn ta được: 4 4

0 0

4x +2y +z =8t rõ ràng z0 chẵn Đặt 4 4

0 0 1

z =2z ⇒2x +y +8z =4t ⇒y chẵn

Đặt 4 4

0 1 1

y =2y ⇒x +8y +4z =2t ⇒x chẵn

Đặt 4 4 ( )

0 1 1 1 1

x =2x ⇒8x +4y +2z = ⇒t x ; y ; z ; t nghiệm phương trình dễthấy x1 <x0(vơ lý) ta chọn x0nhỏnhất Do phương trình có nghiệm (x,y,z,t) (= 0,0,0,0 )

Tổng kết: Một tốn nghiệm ngun thường giải nhiều phương pháp, bạn

đọc nên tìm nhiều cách giải cho tốn đểrèn luyện kĩ Sau giải một toán nhiều phương pháp đểtổng kết.

Bài tốn Tìm nghiệm ngun phương trình sau: x2+xy y+ =x y2 ( )1 Lời giải

Cách Đưa vềphương trình ước số

( ) ( )

( ) ( )

( )( )

2 2

2 2

2 2

2

2

x xy y x y 4x 4xy 4y 4x y 4x 8xy y 4x y 4xy

2x 2y 2xy 1 2xy 2x 2y

2xy 2x 2y 2xy 2x 2y

+ + =

⇔ + + =

⇔ + + = +

⇔ + = + −

⇔ + − + =

⇔ + + + + − − =

(105)

Sau giải phương trình ước số

Cách Dùng tính chất sốchính phương phương trìnhước số

( )

( ) ( )

2 2

2 2 2 2

2 2 2

4x 4xy 4y 4x y 2x y 3y 4x y 2x y y 4x

+ + =

⇔ + + =

⇔ + = −

Nếu y = x = ta có (0, 0) nghiệm phương trình Nếu y 0≠ 4x 32− phải sốchính phương

Ta có: 4x k k N2− = 2( ∈ ) đưa về (2x k 2x k+ )( − )=3 Ta tìm x = x = -1 từđó tìm y

Cách Đưa vềphương trình bậc x

(y x2− ) 2−yx y− =0 ( )2

Xét y = (2) có dạng: -x – 1= x = -1 Xét y = -1 (2) có dạng x – = x =

Xét y≠ ±1 (2) phương trình bậc hai x có:

( ) ( )

2 2 2

y 4y y y 4y

∆ = + − = −

Ta phải có ∆ sốchính phương Nếu y = từ(2) suy x =

Nếu y 0≠ 4y2−3 phải sốchính phương

Ta có 4y k k N2− = 2( ∈ ) (⇒ 2y k 2y k+ )( − )=3,ta y= ±1 do xét y= ±1 Cách Sửdụng bất đẳngthức

Khơng tính tổng quát giả sử x y≤ , thếthì x2 ≤y ,xy xy y2 ≤ ≤ 2 Do đó: x y2 =x2+xy y+ ≤y2 +y2+y2 ≤3y2

Nếu y = x =

Nếu y 0≠ chia hai vếcho y2 ta x2 ≤3 Do x2 = ⇒ = ±1 x 1 Vậy phương trình có ba nghiệm (1, -1) , (-1, 1), (0, 0)

Cách Sửdụng tính chất sốchính phương

Thêm xy vào hai vế x2+2xy y+ =x y2 2+xy⇔(x y+ )2 =xy xy 1( + )

Ta thấy xy (xy + 1) hai sốngun liên tiếp có tích sốchính phương nên tồn sốbằng

Xét xy = từ(1) có x2 +y2 = ⇒ = =0 x y 0

(106)

Xét xy = -1 nên x = , y = -1 x = -1, y =

Thửlại thấy phương trình có ba nghiệm (0, 0); (1, -1); (-1, 1) C BÀI TẬP ÁP DỤNG

Bài 1: Tìm nghiệm nguyên phương trình 2xy− − =x y Bài 2: Tìm nghiệm nguyên phương trình 2

2009

+ + =

x x y

Bài 3: Tìm nghiệm nguyên phương trình 2

5 10

x + y + z + xyxz=

Bài 4: Giải phương trình nghiệm nguyên

3x −2xy+ −y 5x+ =2

Bài 5: Tìm nghiệm nguyêncủa phương trình 2

(x +y x)( +y )=(xy)

Bài 6: Giải phương trình nghiệm nguyên 3

2

xy = xy+

Bài 7: Tìm nghiệm nguyên dương phương trình 5(x+ + + =y z) 2xyz Bài 8: Tìm nghiệm nguyên phương trình

a) 4

1+ +x x +x +x = y ;

b) 3

1+ +x x +x = y

Bài 9: Giải phương trình nghiệm nguyên 4x+9y=48

Bài 10: Tìm sốtựnhiên lẻ n để 26n+17 sốchính phương

Bài 11: Tìm sốnguyên x y z, , cho x4+y4 +z4 =2012

Bài 12: Tìm nghiệm nguyên dương hệphương trình 213 22 22

13

x y z

x y t

 + = 

+ = 

Bài 13: Tìm nghiệm nguyên phương trình 3

3

xyz =

Bài 14: Tìm nghiệm nguyên phương trình

2 2

2 2 4

x + y + zxyyzz= −

Bài 15: Tìm nghiệm nguyên phương trình

( )( )( )

1 48

x + y + z + = xyz

Bài 16: Tìm nghiệm nguyên hệphương trình

2

2

9 16 12

x z

y t

xt yz

 + =

 + =

 + = 

Bài 17: Tìm nghiệm phương trình: x3+y3−x y xy2 − =5

Bài 18: Tìm nghiệm nguyên phương trình : x(x + 1)(x + 2)(x + 3) = y2 (1) Bài 19: Tìm tất cảnghiệm nguyên phương trình: (x y y x2− )( 2− )=(x y− )3 Bài 20: Tìm tất cảcác sốx, y nguyên dương thỏa mãn phương trình: 1

x y 617+ = Bài 21: Giải phương trình nghiệm nguyên dương 1

x y p+ = p sốnguyên tố

(107)

Bài 22: Tìm nghiệm nguyên dương phương trình: 1 1 x y 6xy 6+ + = Bài 23: Tìmnghiệmnguyêncủa phươngtrình 6x 15 10z 3+ + =

Bài 24: Chứng minh phương trình sau khơng có nghiệmngun:

( )

+ + =

2 2

x y z 1999

Bài 25: Tìm nghiệm dương phương trình x+ y = 50

Bài 26: Giải phương trình nghiệm nguyên: y= x x 1+ − + x x 1− −

Bài 27: Giải phương trình tập sốnguyênx2015 = y(y 1)(y 2)(y 3) 1+ + + +

(Chuyên Quảng Trung – Bình Phước 2015)

Bài 28: Tìm sốtựnhiên x sốnguyên y cho 2x+ =3 y2

Bài 29: Tìm sốtựnhiên x, y thỏa mãn: (2 2x+ )( x+2 2)( x+3 2)( x+4 5)− y =11879. Bài 30: Tìm tất cảcác cặp (x, y, z) sốnguyên thỏa mãn hệphương trình:

3 3

x y z

x y z

 + + =  + + = 

Bài 31: Tìm sốnguyên x, y, z thỏa mãn đẳng thức: 2 x y z 1( ) ( ) 2x xy x 2z

 − + = 

 − + − = 

Bài 32: Tìm sốthực a đểcác nghiệm phương trình sau sốnguyên:

( ) ( )

2

x ax a 2− + + =0

Bài 33: Tìm sốnguyên dương x y thoảmãn phương trình:

( 2 2 )2 ( 4 4) 2

x +4y +28 −17 x +y =238y 833.+

(Chuyên Nguyễn Trãi – Hải Dương 2016 – 2017)

Bài 34: Tìm tất cảcác cặp sốtựnhiên x, y thỏa mãn: 2 xx =9y2 +6y 16+

(Chuyên Hà Nội 2016 – 2017)

Bài 35: Tìm nghiệm nguyên phương trình: x y x y x y xy2 2( + )+ + = +

(Tríchđềvào lớp 10 chuyên ĐHKHTN, ĐHQGHN năm 2014)

Bài 36: Tìm tất cảcác cặp sốnguyên dương (x; y) thỏa mãn (x y+ ) (3 = x y 6− − )2

(Trích đềthi vào lớp 10 Chuyên Lê Hồng Phong- Nam Định 2014-2015)

Bài 37: Tìm nghiệm nguyên phương trình x2−y2 =xy 8+

(Trích đềvào Chun Bình Dương 2017)

Bài 38: Tìm nghiệm nguyên phương trình x 4y 3+ =

(108)

(Trích đềvào Chuyên Lê Hồng Phong – Nam Định)

Bài 39: Tìm nghiệm nguyên phương trình sau x2+y2+5x y2 2+60 37xy=

(Trích đềvào Chuyên bạc Liêu 2017)

Bài 40: Giải phương trình nghiệm nguyên y 2x x x 3− − = ( + )2 ( )1

(Trích đềvào Chuyên Hưng Yên 2017)

Bài 41: Giải phương trình nghiệm nguyên x2+2y 2xy 4x 8y 12− − + + = ( )

(Chuyên Lương Thế Vinh – Đồng Nai 2017)

Bài 42: Tìm x, y nguyên cho x+ y = 18

(Chuyên Bình Định 2015)

Bài 43: Tìm sốnguyên x y thoảmãn phương trình9x y+ = +y

(Chuyên Phan Bội Châu – NghệAn 2014)

Bài 44: Tìm cặp sốnguyên (x;y) thỏa mãn phương trình:2015(x2+y ) 2014(2xy 1) 252 − + =

(Chun TP HồChí Minh 2014)

Bài 45: Tìm nghiệm phương trình: x3+y3−x y xy2 − =5

(Chuyên Lam Sơn 2014)

Bài 46: 1) Tìm tất cảcác sốnguyên tốp sốnguyên dương x,y thỏa mãn

p 2x(x 2) p 2y(y 2)

 − = + 

− = + 

2) Tìm tất cảcác sốnguyên dương n cho tồn sốnguyên dương x, y, z thoảmãn

3 3 2

x +y +z =nx y z

(Chuyên Hà Nội Amsterdam 2014)

Bài 47: Tìm nghiệm nguyên dương hệphương trình: x y z3 3 2

x y z

 + =  

+ = 

(ChunHồng Văn Thụ - Hịa Bình 2015)

Bài 48: Tìm nghiệm nguyên phương trình:x2−2y(x y) 2(x 1)− = +

(Chuyên Hùng Vương Phú Thọ2015) Bài 49: Tìm sốnguyênx,y thỏa mãn x4+x2−y2− +y 20 0.=

(Chuyên Nguyễn Trãi Hải Dương 2015)

Bài 50: a) Chứng minh không tồn sốnguyên (x, y, z) thỏa mãn x4+y4 =7z4+5 b) Tìm tất cảcác nguyện nguyên thỏa mãn đẳng thức (x 1+ ) (4− x 1− )4 =y3

(Chuyên KHTN Hà Nội 2011)

(109)

Bài 51: Tìm tất cảcác cặp sốnguyên ( ; )x y thỏa mãn 2x2+5y2 =41 2+ xy

(ChuyênNam Định 2018-2019)

Bài 52: Tính tất cảcác cặp sốnguyên dương (x y; ) thỏa mãn: x2019 = y2019−y1346−y673+2

(ChuyênLam Sơn – Thanh Hóa 2018-2019)

Bài 53: Cho phương trình 3

2 9!(1)

x + y + z = với x y z; ; ẩn 9! Là tích sốnguyên

dương liên tiếp từ1 đến

a) Chứng minh có sốnguyên x y z; ; thỏa mãn (1) x y z, , chia hết cho b) Chứng minh không tồn sốnguyên x y z, , thỏa mãn (1)

(ChuyênVĩnh Phúc 2018-2019)

Bài 54: Tìm nghiệm nguyên phương trình

2

xxy+ = +x y

(ChuyênBến Tre2018-2019)

Bài 55: Tìm số nguyên x y z, , thỏa mãn đồng thời: x2+4y2+ +z2 2xz+4(x+ =z) 396

2

3

x +y = z

(ChuyênĐăk Lăk2018-2019)

Bài 56: Tìm cặp sốnguyên (x y; )thỏa mãn điều kiện 2x2−4y2−2xy−3x− =3

(ChuyênĐồng Nai 2018-2019)

Bài 57:Tìm nghiệm nguyên phương trình:

(ChuyênTuyên Quang 2018-2019)

Bài 58: Tìm x y, nguyên dương thỏa mãn: 16(x3−y3)=15xy+371

(ChuyênThái Nguyên2018-2019)

Bài 59: Tìm cặp sốnguyên thỏa mãn

(ChuyênBắc Ninh 2018-2019)

Bài 60: Tìm nghiệm nguyên phương trình x2−xy y+ =2x−3y−2

(ChuyênVĩnh Long2018-2019)

Bài 61: Tìm tất cảcác cặp sốnguyên ( ; )x y thỏa mãn đẳng thức x y2 2−x2−6y2 =2 xy (ChuyênQuảng Nam 2018-2019)

Bài 62: Tìm tất cảcặp sốnguyên x y, thỏa mãn y2+2xy−3x− =2

(ChuyênLào Cai 2018-2019)

Bài 63: Tìm tất cảbộ sốnguyên ( )a b; thỏa mãn 3(a2+b2)−7(a b+ )= −4

(ChuyênBình Phước 2018-2019)

Bài 64: Tìm tất cảcác cặp sốnguyên thỏa mãn

(ChunTốn Lam Sơn – Thanh Hóa2019-2020)

Bài 65: Tìm tất cảcác nghiệm nguyên phương trình:

(ChuyênTin Lam Sơn – Thanh Hóa2019-2020)

2

3x −2xy+ −y 5x+ =2

,

x y x2−2y2 =1

(x ; y) y2+ =y x4+x3+x2+x

2

5

xxyx+ y+ =

(110)

Bài 66: Tìm tất nghiệm nguyên dương phương trình

(ChuyênHưng Yên2019-2020)

Bài 67: Tìm tất cảcác sốtựnhiên đểphương trình (ẩn số ) có nghiệm sốnguyên

(ChuyênBình Thuận 2019-2020)

Bài 68: Tìm tất cặp số nguyên (x, y) thỏa mãn

(ChuyênPhú Yên2019-2020)

Bài 69: Tìm sốnguyên không âm thỏa mãn:

(ChuyênQuảng Nam 2019-2020)

Bài 70: Tìm tất cảcặp sốnguyên (x; y) thỏa mãn

(ChuyênCần Thơ2019-2020)

Bài 71: Tìm tất cảcác nghiệm nguyên dương phương trình

(ChuyênĐăk Nơng 2019-2020)

Bài 72: Tìm nghiệm ngun dương phương trình

(ChuyênQuảng Ngãi 2019-2020)

Bài 73: Giải phương trìnhnghiệm nguyên

(ChuyênBình Phước 2019-2020)

Bài 74: Tìm tất cặp số nguyên dương thỏa mãn:

(ChuyênBắc Ninh 2019-2020)

Bài 75: Tìm tất cặp số nguyên thỏa mãn

(ChuyênTiền Giang 2019-2020)

Bài 76: Tìm tất cảcác cặp sốnguyên dương (m;n) thỏa mãn phương trình 2m.m2= 9n2 -12n +19.

(ChuyênBà Rịa Vũng Tàu 2019-2020)

Bài 77: Tìm tất cảcác cặp sốnguyên thỏa mãn

(ChuyênHà Nội 2019-2020)

Bài 78: Tìm tất cảcác cặp sốnguyên thỏa mãn

(ChuyênSư phạm Hà Nội 2019-2020)

Bài 79: Tìm x, y thỏa mãn: x y 2( + − ) = x.y

2

xy −(y 45)− +2xy x 220y 2024 0+ − + =

n x2 −n x n 02 + + = x

2

x y 85 x y 13

+ = +

,

a b,n

2

3 2

2

n a b

n a b

 = + 

+ = + 

2

2020(x +y ) 2019(2xy 1) 5− + =

2

2x y− =1 x +3y

x+ + + =y x+ y

2

4y = +2 199 x− −2x

 x y; (xy x y x+ + )( +y2 +1)=30

( )x y;

( )( )

2x+5y+1 2x− + +y x +x =65

2

(x − +x 1)(y +xy)=3x−1

( )x y; x y2 2−4x y2 +y3+4x2−3y2+ =1 0

(111)

(HSG Lớp An Giang năm 2015-2016)

Bài 80: Tìm tất cảcác nghiệm nguyên phương trình x2 +xy y+ =x y2

(HSG Lớp Thanh Hóa năm 2015-2016)

Bài 81: Tìm tất cảcác cặp sốnguyên (x;y) thoảmãn x5+y2 =xy 12+

(HSG Lớp TP Bắc Giang năm 2016-2017)

Bài 82: Tìm sốnguyên dương x, y, zthỏa mãn: 3x 18y2 − +2z2+3y z 18x 272 2− = (HSG Lớp Hải Dương năm 2014-2015)

Bài 83: Tìm nghiệm nguyên phương trình x y x y x y x 2 2( + )+ = + ( − )

(HSG Lớp Thanh Hóa 2018-2019)

Bài 84: Tìm tất cảcác nghiệm nguyên dương phương trình:

xy +2xy 243y x 0− + =

Bài 84: Tìm sốnguyênx, y thỏa mãn đẳng thức x2 =y2+ y 1+ Bài 85: Giải phương trình nghiệm nguyên y2 = +1 9 x 4x− −

Bài 86: Tìm sốngun a đểphương trình sau có nghiệm ngun dương 3a a− = −

Bài 87: Tìm tất cảcác cặp nguyên thỏa mãn

(HSG Lớp Lạng Sơn năm 2018-2019)

Bài 88: Tìm nghiệm nguyên dươngcủa phương trình :

(HSG Lớp Bình Phước năm 2018-2019)

Bài 89: Tìm tất cảcác cặp sốnguyên thỏa mãn phương trình

(HSG Lớp Nam Định năm 2018-2019)

Bài 89: Tìm tất cảcác cặp sốnguyên (𝑥;𝑦)thỏa mãn:

(HSG Lớp Hưng Yên năm 2017-2018)

Bài 90:Tìm nghiệm nguyên phương trình

(HSG Lớp Thanh Hóa năm 2017-2018)

Bài 91: Tìm cặp số nguyên thỏa mãn:

(HSG Lớp Hải Dương năm 2016-2017)

Bài 92: Tìm tất cảcác nghiệm nguyên phương trình

(HSG Lớp Hưng Yên năm 2016-2017)

Bài 93: Tìm sốnguyên x, y thoảmãn phương trình

( )x; y x y2 2+(x 2− ) (2+ 2y 2− )2−2xy x 2y 4( + − )=5

4

4y +6y x− =

(x; y)

(x y x y 6xy y x y− − )( + − )+ + 2( − − ) (=2 x y 1+ )( + )

( )2 4 3 2

x 2018− =y 6y 11y 6y− + −

2 2

y 5y 62 (y 2)x (y 6y 8)x.− + = − + − +

(x y; ) 2x2 +2y2+3x 6y 5xy 7.− = −

2

3x 16y 24− − = 9x 16x 32+ +

(x y x 2y+ )( + )= +x

(112)

(HSG Lớp TP HồChí Minh năm 2016-2017)

Bài 94: Tìm cặp sốnguyên thỏa mãn:

(HSG Lớp Vĩnh Phúc năm 2015-2016)

Bài 95: Tìm nghiệm nguyên dương phương trình:

(HSG Lớp NghệAn năm 2015-2016)

Bài 96: Tìm nghiệm nguyên phương trình:

(HSG Lớp Thanh Hóa năm 2015-2016)

Bài 97: Tìm nghiệm nguyên (x; y) phương trình:

(HSG Lớp Thanh Hóa năm 2014-2015)

Bài 98: Tìm cặp sốnguyên thỏa mãn:

(HSG Lớp Vĩnh Phúc năm 2014-2015)

Bài 99: Tìm sốnguyên dương x, y, z thỏa mãn

(HSG Lớp Khánh Hòa năm 2014-2015)

Bài 100: Tìm thỏa mãn

(HSG Lớp Thanh Hóa năm 2012-2013)

Bài 101: Tìm nghiệm ngun phương trình: 2

2xy + + + =x y x + 2y + xy

(HSG Lớp Bình Định năm 2018-2019)

Bài 102: Tìm sốnguyên x y, thỏa mãn 4x = +1 3y

(HSG Lớp Quảng Trịnăm 2018-2019)

Một sốbài toán từđềthi học sinh giỏi toán lớp 10!

Bài 103.Tìm tất cảcác sốtựnhiên x, y thỏa mãn phương trình:

( )4

x− y =3361− 11296320

(Đềđề nghịTHPT TP Cao Lãnh – Đồng Tháp)

Bài 104.Tìm nghiệm nguyên phương trình: ( )

2 4x 6y 9x 6y

313

x y

− + − =

+

(Đềđề nghịTHPT Bạc Lưu)

Bài 105.Tìm nghiệm nguyên phương trình: x2+ + =x 2xy y+

(Đềđề nghịChuyên Lê Khiết – Quảng Ngãi)

(x y; ) x x x 1.( 2+ + =) y−

x

3 171 y+ =

( )x; y 54x y 3+ =

( 2) ( )

5 x +xy y+ =7 x 2y+

( )x; y x x x( + + 2)=4y y ( − )

2

x =2x yzz4+

x,y,z N∈ x 3+ = y+ z

(113)

Bài 106. Chứng tỏrằng số: 444444 303030 3+ không viết dạng (x y 3+ )2 với x,y Z∈

(Đềđề nghịChuyên Quang Trung – Bình Phước)

Bài 107.Tìm tất cảcác sốnguyên dương x, y thỏa mãn phương trình:

( 2 ) ( )

9 x +y +2 3xy 1+ − =2008

(Đềđề nghịTHPT Hùng Vương – Lê Lai)

Bài 108.Tìm nghiệm nguyên phương trình: x3+x y xy2 + 2+y3 =8 x( 2+xy y 1+ 2+ )

(Đềđề nghịChuyên Lương Văn Chánh – Phú Yên)

Bài 109.Tìm nghiệm nguyên phương trình

( )

2

x 17y+ +34xy 51 x y+ + =1740

Bài 110.Tìm tất cảcác cặp (x, y, z) sốnguyên thỏa mãn hệphươngtrình

3 3

x y z

x y z

 + + =  + + = 

Bài 111.Tìm tất cảcác sốnguyên x, y, z thỏa mãn phương trình:

2 2 2

3x +6y +2z +3x y 18x 0.− − =

Bài 112.Tìm tấtcảcác cặp sốnguyên dương (a,b) thỏa mãn đẳng thức:

3 2

a −b 3(a+ −b ) 3(a b) (a 1)(b 1) 25+ − = + + + Bài 113 Tìmtất cảcác cặp sốnguyên dương ( )x; y thoảmãn phương trình:

( 2 2 )2 ( 4 4 2 )

x +4y +28 =17 x +y 14y+ +49

Một sốbài toán phương trình nghiệm ngun tạp trí tốn học tuổi trẻ

Bài 114 Tìm nghiệm nguyên phương trình x y xy x y 1.2( − −) = − + Bài 115 Tìm sốnguyên (a.b,c,d) thỏa mãn hệ ac 3bd

ad bc

 − =  + =

Bài 116 Một tam giác có sốđo cạnh sốnguyên x, y, z thỏa mãn

2 2

2x +3y +2z −4xy 2xz 20 0.+ − = Chứng minh tam giác tam giác

Bài 117 Chứng minh phương trình sau khơng có nghiệm ngun dương

( ) ( )2

2

x +y = x y+ + xy Bài 118 Tìm nghiệm tựnhiên phương trình

(114)

2 3 2

x y −4xy +y +x −2y 0.− =

Bài 119 Tìm nghiệm nguyên dương hệphương trình 3x y z3 2

x y z

 + =  + =

Bài 120.Tìm nghiệm nguyên phương trình 2 x y 2 x xy y

= − +

Chương V

CÁC BÀI TOÁN VỀ TỔ HỢP SUY LUẬN PHẦN 1: CÁC BÀI TOÁN ỨNG DỤNG NGUN LÝ DIRICHLET

A.KiÕn thøc cÇn nhí

1 Giới thiệu nguyên lý Dirichlet

Dirichlet (Đi-rích-lê) (1805 – 1859) nhà tốn học người Đức, cho người đưa định nghĩa đại vềhàm số Trên sởquan sát thực tế, ông phát biểu thành nguyên lí mang tên ông – nguyên lí Dirichlet: Không thể nhốt thỏvào lồng mà

mỗi lồng có khơng q thỏ Nói cách khác, nếu nhốt thỏvào lồng tồn

nhất lồng có từ3 trở lên Một cách tổng quát hơn, nếu có k lồng để nhốt m thỏ (với ) tồn lồng có chứa từ n + thỏ trở lên Ta dễdàng minh nguyên lí Dirichet phương phápphản chứng sau: Giả sửkhơng có lồng n + thỏ trởlên, tức lồng chứa nhiều n thỏ, số thỏchứa k lồng nhiều chỉcó thể kn Điều mâu thuẫn với giảthiết có m thỏvới .

Nguyên lí Dirichlet thật đơn giản, dễ hiểu vận dụng vào giải nhiều toán số học, đại số, hình học ciệc tồn hay nhiều đối tượng thỏa mãn điều kiện đặt

Khi sửdụng ngun lí Dirichlet vào tốn cụthể, điều quan trọng phải nhận (hay tạo ra) Lồng Thỏ cảLồngThỏ

2 Một số dạng áp dụng nguyên lý Dirichlet Nguyên lý Dirichlet bản:.

Nếu nhốtn + 1con thỏ vào n chuồng có chuồng chứt hai thỏ

Nguyên lý Dirichlet tổng quát:

k=kn r+ (0< ≤ −r k 1)

m=kn r+ (0< ≤ −r k 1)

(115)

Mệnh đề: Nếu có N đồ vật đặt vào k hộp tồn hộp chứa đồ vật

(Ở đây, [x] giá trị hàm trần số thực x, số nguyên nhỏ có giá trị lớnhơn x Khái niệm đối ngẫu với [x] – giá trị hàm sàn hay hàm phần nguyên x – số nguyên lớn có giá trị nhỏ x.)

Chứng minh:

Giả sử hộp chứa vật Khi tổng số đồ vật là; k ( - 1) < k = N

Điềunày mâu thuẩn với giả thiết có N đồ vật cần xếp

Nguyên lí Dirichlet đối ngẫu

Cho tập hữu hạn S ≠ ∅và S1, S2, …, Sn tập

của S cho | S1 | + | S2 | + … + | Sn | > k | S | Khi đó, tồn phần tử x S cho x phần tử chung k+ tập Si ( i = 1, 2, … n)

Nguyên lí Dirichlet mở rộng

Nếu nhốt n thỏ vào m ≥ chuồng tồn chuồng có thỏ, kí hiệu [α] để phần nguyên số α

Ta chứng minh nguyên lí Dirichlet mởrộng sau : Giả sử trái lại chuồngthỏ đến

con, số thỏ chuồng nhỏ Từ suy tổng số thỏ không vượt m

N k      

N k       N

k      

N k      

1

n m

m + −      

1 1

1

n m n n

m m m

+ − − −   = + =  +            

1

n m

−      

1

1

n

n m

  ≥ −    

(116)

Điều vơ lí có n thỏ Vậy giả thiết phản chứng sai

Nguyên lí Dirichlet mở rộng chứng minh.Nguyên lí Dirichlet tưởng chừng đơn giản vậy, công cụ hiệu dùng để chứng minh nhiều kết sâu sắc tốn học Nó đặc biệt có nhiều áp dụng lĩnh vực khác tốn học Ngun lí nhiềutrường hợp người ta dễ dàng chứng minh tồn mà khơng đưa đượcphương pháp tìm vật cụ thể, thực tế nhiều toán ta cầnchỉ tồn đủ rồi.Nguyên lí Dirichlet thực chất định lí tập hữu hạn Người ta có thểphát biểu xác ngun lí dạng sau

Nguyên lí Dirichlet dạng tập hợp

Cho A B hai tập hợp khác rỗng có số phần tử hữu hạn, mà số lượng phầntử A lớn số lượng phần tử B Nếu với quy tắc đó, phầntử A cho tương ứng với phần tử B, tồn hai phần tử khác A mà chúng tương ứng với phần tử B

Với cách vậy, ngun lí Dirichlet mở rộng có dạng sau

Hình Ngun lí Dirichlet dạng tập hợp mở rộng

Giả sử A,B hai tập hợp hữu hạn S (A),S(B) tương ứng kí hiệu sốlượng phần tử A B Giả sử có số tự nhiên k mà S(A)>k.S(B) ta có quy •

(117)

tắc cho tương ứng phần tử A với phần tử B Khi tồn k+1 phần tử A mà chúng tương ứng với cùngmột phần tử B

Chú ý: Khi k = 1, ta có lại ngun lí Dirichlet

Vì chương dành để trình bày phương pháp sử dụng nguyên lí Dirichlet để giải tốn hình học sơ cấp.Vì lẽ đó, tơi xin trình bày số mệnh đề sau ( thực chất số phát biểu khác nguyên lí Dirichlet áp dụng cho độ dài đoạn thẳng, diện tích hình phẳng, thể tích vật thể) hay sử dụng đến nhiều tốn hình học đề cập tới chương

Nguyên lí Dirichlet cho diện tích:

Nếu K hình phẳng, cịn hình phẳng cho với , , |K| diện tích hình phẳng K, cịn diện tích hình phẳng , , tồn hai hình phẳng (

) cho có điểm chung

( Ở ta nói P điểm tập hợp A mặt phẳng tồn hình trịn tâm P bán kính đủ bé cho hình trịn nằm trọn A)

Tương tự ngun lí Dirichlet cho diện tích, ta có nguyên lí Dirichlet cho độ dài đoạn thẳng, thể tích vật thể …

Nguyên lí Dirichlet vô hạn:

Nếu chia tập hợp vô hạn táo vào hữu hạn ngăn kéo, phải có ngăn kéo chưa vơ hạn táo

Nguyên lí Dirichlet mở rộng cho trường hợp vơ hạn đóng vai trị quan trọng lí thuyết tập điểm trù mật đường thẳng Nó có vai trị quan trọng lí thuyết số nói riêng tốn học rời rạc nói chung (trong có hình học tổ hợp)

Nguyên lý Dirichlet đối ngẫu vô hạn phần tử *Tập phần tử khoảng đường thẳng •

1, 2, , n

K K K KiK

1,

i= n |K | |< K1|+|K2| |+ + Kn|

|Ki | Ki i =1,n H Hi, j

1≤ < ≤i j n H Hi, j

(118)

Trong mục ta kí hiệu d(I) độ dài khoảng I R

+ Cho A khoảng giới nội, A 1, A2, … , An khoảng cho Ai A (i = 1, 2, …, n) d(A) < d(A1) + d(A2) + … + d(An) Khi nh ất có hai khoảng số khoảng có điểm chung

Chứng minh.

Thật vậy, giả sử khơng có cặp khoảng cho có điểm chung

Khi đó, d(A1 A … An) = d(A1) + d(A2) + … + d(An) > d(A)

Mặt khác, từ Ai A (i = 1, 2, …, n) suy d(A1 A … An )≤ d(A) Các bất đẳng thức mâu thuẫn với Vậy có hai khoảng số khoảng có điểm chung

Tập phần tử miền phẳng giới hạn đường cong phẳng khép kín Trong mục ta kí hiệu S(A) diện tích miền A mặt phẳng + Nếu A miền giới hạn đường cong phẳng khép kín, cịn

A1, A2, … , An miền cho Ai A (i = 1, 2, …, n) S(A) < S(A1) + S(A2) + … + S(An), có hai miền số miền nói có điểm chung Chứng minh.Tương tự chứng minh Định lí

1.2.2 Phương pháp ứng dụng

Nguyên lí dirichlet tưởng chừng đơn giản vậy, cơng cụ có hiệu dùng để chứng nhiều kết sâu sắc tốn học Ngun lí Dirichlet áp dụng cho tốn hình học, điều thể qua hệ thống tập sau:

Để sử dụng nguyên lý Dirichlet ta phải làm xuất tình nhốt “thỏ” vào “chuồng” thoả mãn điều kiện :

+ Số ‘thỏ” phải hiều số chuồng

∪ ∪

⊂ ∪ ∪

(119)

+ “Thỏ” phải nhốt hết vào “chuồng”, không bắt buộc chuồng phải có thỏ

Thường phương pháp Dirichlet áp dụng kèm theo phương pháp phản chứng Ngồi cịn áp dụng với phép biến hình

B CÁC DẠNG TỐN THƯỜNG GẶP

Dạng 1: Chứng minh tồn chia hết

* Cơ sở phương pháp:

Thông thường ta coi m số tự nhiên cho m “con thỏ”, số dư phép chia sốtựnhiên cho n “lồng”; sẽcó n lồng: lồng i gồm sốtựnhiên cho chia cho n dư i

* Ví dụ minh họa:

Bài tốn Chứng rằng:

a) Trong 2012 số tự nhiên ln tìm hai số chia cho 2011 có số dư (hay hiệu chúng chia hết cho 2011)

b) Trong 2012 sơ tự nhiên ln tìm sốchia hết cho 2012 ln tìm hai sốchia cho 2012 có sốdư

Hướng dẫn giải

a) Ta coi 2012 sốtựnhiên cho 2012 “con thỏ”; “lồng i” gồm số chia cho 2011 dư i nên có 2011 lồng: lồng 0, lồng 1, …, lồng 2010 Như có 2011 lồng chứa 2012 thỏnên theo nguyên lí Dirchlet tồn lồng chứa khơng hai thỏ, tức có hai sốchia cho 2011 có sốdư

b) Nếu 2012 số cho có số chia hết cho 2012 ta chọn ln số Nếu khơng có số chia hết cho 2012 chia cho 2012 nhận nhiều 2012 sốdư khác 1, 2, …, 2011 Theo nguyên lí Dirichlet, tồn hai sốchia cho 2012 có sốdư

Nhận xét Ta có thểtổng qt tốn sau:

1) Trong n + số tự nhiên ln tìm hai sốchia cho n có số dư (hay hiệu chúng chia hết cho n)

2) Trong n sốtự nhiên ln tìm sốchia hết cho n ln tìm hai sốchia cho n có sốdư

Bài tốn Chứng minh ln tìm sốcó dạng 20122012…2012 (gồm số 2012 viết liên tiếp nhau) chia hết cho 2013

Hướng dẫn giải (0≤ ≤i b)

(0≤ ≤i 2011)

(120)

Xét 2014 sốsau:2012, 20122012, , 2012 2012 (gồm 2014 số2102)

Đem 2014 số chia cho 2013, có 2014 sốmà có 2013 số dư phép chia cho 2013 (là 0, 1, 2, , 2012) nên tồn hai sốchia cho 2013 có sốdư, chẳng hạn a = 2012 2012 (gồm i bộ2012) b = 2012 2012 (gồm j 2012) với Khi

(gồm j – i bộ2012) sẽchia hết cho 2013

Lại có ƯCLN nên số 2012 2012 (gồmj – i 2012 sẽchia hết cho 2013 Bài tốn chứng minh

(Ởđây “thỏ” sốcó dạng 2012 2012, “lồng” sốdư phép chia cho 2013)

Nhận xét.Mấu chốt toán chọn 2014 (= 2013 + 1) số tự nhiên có dạng cho Từđó ta có thểphát biểu nhiều tốn tương tự, chẳng hạn như: Chứng minh ln tìm sốcó dạng 111 chia hết cho 29

Bài toán Cho sáu số tự nhiên Chứng minh sáu số ấy, tồn sốchia hết cho tồn vài sốcó tổng chia hết cho

Hướng dẫn giải

Trường hợp có sốbằng ta chọn số0 thỏa mãn yêu cầu đềra Trường hợp sáu sốđều lớn Xét sốsau

Đem sốnày chia cho ta nhận sốdư thuộc tập Nếu tồn chia hết cho tốn đãđược chứng minh

Nếu khơng có Sinào chia hết cho ta có sốchia hết cho chỉnhận loại số dư khác ; theo nguyên lý Dirichlet tồn hai số chia cho có số dư, chẳng hạn S2và S5do hiệu hai sốnày chia hết cho 6, tức chia hếtcho Bài toán chứng minh

(Ởđây “thỏ” sốSi, “lồng” sốdư phép chia cho 6) Nhận xét.Ta có thểphát biểu tốn tổng qt sau:

Cho n sốtự nhiên Chứng minh tồn sốchia hết cho n tồn vài sốcó tổng chia hết cho n

Bài tốn Chứng minh rằng:

1≤ ≤ ≤i j 2014

4

2012 2012.10 i

b− =a

4

(10 , 2013) 1i =

, , , , ,

a b c d e g

1

6

S a

S a b

S a b c

S a b c d

S a b c d e

S a b c d e g

= = + = + + = + + + = + + + + = + + + + +

{0,1, 2, 3, 4, 5}

( 1, 2, , 6) i

S i=

(1, 2, 3, 4, 5)

c d+ +e

1, 2, , n

a a a

(121)

a) Trong n sốtựnhiên liên tiếp tìm sốchia hết cho n

b) Trong 39 số tự nhiên liên tiếp ln tìm sốmà tổng chữ số chia hết cho 11

Hướng dẫn giải

a) Giả sử không tìm số n sốtự nhiên liên tiếp cho mà chia hết cho n Khi n số chia cho n nhận nhiều n – số dư khác , theo nguyên lí Dirichlet tồn hai số chia hết cho n có số dư, chẳng hạn a b với , a – b chia hết cho n, điều mâu thuẫn với Từ suy điều phải chứng minh

b) Lấy 20 sốtựnhiên liên tiếp đầu dãy, ta ln tìm sốcó chữ sốhàng đơn vị có chữ sốhàng chục khác 9.Giả sửđó N tổng chữ sốcủa N s Khi 11 số sẽnằm 39 sốđã cho Vì N tận nên tổng chữ sốcủa Vì N tận có chữ sốhàng chục khác nên tổng chữ sốcủa N + 10 s + 1, tổng chữ sốcủa N + 19 s + 10

Trong 11 sốtự nhiên liêntiếp ln tìm sốchia hết cho 11 Chẳng hạn sốđó : Nếu ta chọn số thỏa mãn yêu cầu toán; i = 10 ta chọn sốN + 19 thỏa mãn yêu cầu toán

Nhận xét.Mấu chốt để giải tốn câu b) phải tìm 11 số 39 số cho có tổng chữ sốthứtựlà 11 sốtựnhiên liên tiếp, đồng thời sửdụng kết quảcâu a)

Bài toán Cho số tự nhiên từ đến 2012 Hỏi chọn nhiều sốsao cho tổng hai sốbất kì chúng khơng chia hết cho hiệu nó?

Hướng dẫn giải

Nhận thấy, hai số chia cho dư hiệu chúng chia hết cho 3, tổng chúng chia cho dư 1; nên tổng chúng không chia hết cho hiệu chúng

Trong số tự nhiên từ1 đến 2012, có 671 sốchia cho dư sốcó dạng

Khi hai số 671 số có tổng chia dư 1, hiệu chia hết cho 3, nên tổng không chia hết cho hiệu chúng Ta chứng minh chọn nhiều sốtrong sốtừ1 đến 2012, 672 sốnày ln tìm cho (Thật vậy, giả sửngược lại hiệu sốnhỏnhất sốlớn số chọn không nhỏ Điều mâu thuẫn giả thiết với hiệu số lớn số nhỏ không vượt ), nghĩa a – b

- Nếu a – b = hiển nhiên a + b chia hết cho a – b (= 1)

(1, 2, 3, ,n−1)

a>b 0< − <a b n

, 1, 2, 3, 9, 19

N N + N+ N+ N+ N+

, 1, 2, ,

N N+ N+ N+ s s, +1,s+2, ,s+9

, 1, 2, 3, , 9, 10

s s+ s+ s+ s+ s+

(0 10)

s+i ≤ ≤i 0≤ ≤i N+i

3k+2

(k =0,1, 2, , 670)

672( 671 1)= +

, ( )

a b a>b a b− ≤2

3.671 2013=

2012 2011− =

(122)

- Nếu a – b = a + b sốchẵn nên a + b chia hết cho a – b (= 2)

Như từ2012 sốđã cho không thểchọn 671 số thỏa mãn điều kiện toán Suy sốlượng lớn sốphải tìm 671

Dạng 2: Bài tốn tính chất phần tử tập hợp

* Cở sở phương pháp: Thông thường ta phải lập tập hợp có tính chất cần thiết sửdụng ngun lí Dirichlet đểchứng tỏcó hai phần tửthuộc hai tập hợp * Ví dụ minh họa:

Bài tốn Cho sáu sốnguyên dương đôi khác nhỏhơn 10 Chứng minh ln tìm sốtrong có sốbằng tổng hai sốcịn lại

Hướng dẫn giải

Gọi sáu sốnguyên dương cho với Đặt gồm phần tửcó dạng amvới

Đặt gồm phần tử có dạng với

Ta thấy phần tửcủa hai tập hợp A B thuộc tập hợp gồm phần tử tổng sốphần tửcủa hai tập hợp A B

Theo nguyên lí Dirichlet tồn hai sốbằng mà chúng không thểthuộc tập hợp, nên có sốthuộc tập hợp A sốthuộc tập hợp B, tức ,

Ba số đơi khác Thật vậy, trái với giả thiết toán

Vậy tồn ba số sốđã cho mà (đpcm)

(Ở đây, có 10 “thỏ” 10 số có “lồng”

là số1, 2, 3, 4, 5, 6, 7, 8, 9)

Nhận xét.Để giải toán này, ta cần tạo hai tập hợp gồm phần tử nhỏ hợn 10 tổng số phần tửcủa hai tập hợp phải khơng nhỏhơn 10 Từ suy tồn hai phần tử hai tập hợp

Bài toán Cho X tập hợp gồm 700 số nguyên dương khác nhau, sốkhông lớn 2006 Chứng minh tập hợp X ln tìm hai phần tửx, y cho x – y thuộc

tập hợp

Hướng dẫn giải

Giả sử700 sốnguyên dương cho Ta xét tập hợp sau:

1, 2, 3, 4, 5,

a a a a a a 0< <a1 a2 < < a6 <10

2

{ , , , , }

A= a a a a a m∈{2, 3, 4, 5, 6}

2

{ , , , , }

B= aa aa aa aa aa ana1

{2, 3, 4, 5, 6}

n

{1, 2, 3, , 9}

5 10+ =

1

m n

a =aa

1

n m

a =a +a

1

, , m n

a a a aman am =an a1=0

1

, , m n

a a a an =am+a1

2, 3, 4, 5, 6, 1, 1, 1, 1,

a a a a a aa aa aa aa aa

{3; 6;9}

E=

1, 2, , 700

a a a

(123)

Tổng số phần tửcủa ba tập hợp A, B, C 700.3 = 2100, phần tử khơng vượt q 2006 + = 2015, mà 2100 > 2015 nên theo nguyên lí Dirichlet tồn hai phần tử Vì tập hợp A, B, C có phần tửđơi khác nên hai phần tửbằng phải thuộc hai tập hợp: A B, A C, B C

- Nếu hai phần tửthuộc A B, chẳng hạn suy - Nếu hai phần tửthuộc A C, chẳng hạn suy - Nếu hai phần tửthuộc B C, chẳng hạn suy

Như tồn lại hai số thuộc tập hợp A có hiệu 3, 6, Ta điều phải chứng minh

(Ở 2100 “thỏ” 2010 phần tử ba tập hợp A, B, C; 2015 “lồng” số từ đến 2015)

Nhận xét.Ta cịn có kết quảmạnh sau:

Cho X tập hợp gồm 505 sốnguyên dương khác nhau, sốkhông lớn 2006 Trong tập hợp X ln tìm hai phần tửx, y cho x – y thuộc tập hợp

Chứng minh

Gọi A tập hợp sốthuộc X mà chia hết cho 3, gọi B tập hợp sốthuộc X mà chia cho dư 1, gọi C làtập hợp sốthuộc X mà chia cho3 dư

Có505 sốxếp vào ba tập hợp, mà 505 = 3.168 + nên theo ngun lí Dirichlet tồn tập hợp có chứa từ169 sốtrởlên

Trong tập hợp này, hai số có hiệulà bội Tồn hai số x, y có hiệu nhỏ 12 Thật vậy, sốtrong tập hợp có hiệu khơng nhỏhơn 12 số lớn tập hợp khơng nhỏhơn 12.168 = 2016 > 2006, trái với đềbài

Vậy tập hợp X tồn hai phần tửx, y mà

Bài toán Cho hai tập hợp sốnguyên dươngphân biệtmà sốđều nhỏhơn n Chứng minh tổng số phần tử hai tập hợp khơng nhỏ n chọn tập hợp phần tửsao cho tổng chúng n

Hướng dẫn giải

Giả sửhai tập hợp sốnguyên dương cho

với ,

Xét tập hợp

Nhận thấy, có tất cản – số nguyên dương phân biệt nhỏ n, phần tửcủa A C nhỏ n tổng số phần tử A C khơng nhỏ n Theo ngun lí

1 700

1 700

1 700

{ , , };

{ 6, 6, 6};

{ 9, 9, 9};

A a a a

B a a a

C a a a

=

= + + +

= + + +

6

i j

a =a + aiaj =6

9

i j

a =a + aiaj =9

3

i j

a + =a + aiaj =3

{3; 6;9}

E=

x− ∈y E

1

{ , , , m}

A= a a a B={ ,b b1 2, , }bk a<n(i=1, 2, , )m bj <n (j=1, 2, , )k m l+ ≥n

1

{ , , , k}

C= n b n b− − n b

(124)

Dirichlet, tồn hai phần tử nhau, chúng không thuộc A C, phần tử thuộc A phần tử thuộc C, tức tồn hai số ap mà

(điều phải chứng minh)

(Ởđây coi m + k “thỏ” sốnguyên dương thuộc tập hợp A C, n – “lồng” sốnguyên dương từ1 đến n – 1)

Dạng 3: Bài toán liên quan đến bảng ô vuông

* Cở sở phương pháp: Một bảng vng kích thước n x n gồm n dịng, n cột đường chéo Mỗi dòng, cột, đường chéo đềucó n vng

Một bảng vng kích thước m x n gồm m dịng n cột * Ví dụ minh họa:

Bài tốn Cho mảng vng kích thước x Người ta viết vào ô bảng số -1, 0, 1; sau tính tổng sốtheo cột, theo dòng theo đường chéo Chứng minh tất cảcác tổng ln tồn hai tổng có giá trịbằng

Hướng dẫn giải

Bảng ô vng kích thước x có dịng, cột, đường chéo nên sẽcó 12 tổng số tính theo dịng, theo cột theo đường chéo Mỗi dịng, cột đường chéo có ghi sốthuộc tập {–1; 0; 1} Vì giá trịmỗi tổng thuộc tập hợp {–5; –4; –3; –2; –1; 0; 1; 2; 3; 4; 5} có 11 phần tử Có 12 tổng nhận tập 11 giá trịkhác nên theo nguyên lí Dirichlet tồn hai tổng nhận giá trị Bài toán chứng minh

(Ởđây “thỏ” tổng nên có 12 “thỏ”, “lồng” giá trịcủa tổng nên có 11 “lồng”) Nhận xét. Với cách giải tương tự, ta có tốn tổng qt sau:

Cho bảng vng kích thước n x n Người ta viết vào ô bảng số –1, 0, 1; sau tính tổng số theo cột, theo dòng theo đường chéo Chứng minh tất cảcác tổng ln tồn hai tổng có giá trịbằng Bài tốn Trên bảng vng kích thước x 8, ta viết sốtự nhiên từ1 đến 64, số viết vào ô cách tùy ý Chứng minh tồn hai ô vuông chung cạnh mà hiệu sốghi chúng không nhỏhơn

Hướng dẫn giải

Ta xét hàng có ghi số1 vàcột có ghi số64 Hiệu hai ô 63

Số cặp ô kề từ ô ghi số đến ô ghi số64 nhiều 14 (gồm cặp ô chung cạnh tính theo hàng cặp ô chung cạnh tính theo cột)

Ta có 64 = 14.4 + nên theo nguyên lí Dirichlet, tồn hai kềnhau mà hai sốghi có hiệu khơng nhỏhơn + = Bài toán chứng minh

q n b

p q p q

a = −n ba +b =n

(125)

(Ở đây, “thỏ” hiệu hai sốtrong 64 số (từ1 đến 64) nên có 63 thỏ; “lồng” sốcặp ô vuông kềnhau từô ghi số1 đến ghi số64 nên có nhiều 14 lồng)

Nhận xét

• Mấu chốt tốn quan tâm đến hai vng ghi sốnhỏnhất (số 1) số lớn (số64) sẽcó lớn 63; đồng thời xét từô ghi số1 đến ô ghi số64 chỉcần tối đa (8 – 1) + (8 – 1) = 14 ô Ởđây ta vận dụng ngun lí Dirichlet tổng qt: Có m thỏ, nhốt vào k lồng mà m = kn + r tồn lồng chứa khơng n + thỏ

• Nếu thay bảng chữnhật gồm x 10 ô vng, ghi sốtừ đến 80 khơng lặp cách tùy ý kết quảcầu tốn cịn hay khơng? Hãy chứng minh

Dạng 4: Bài toán liên quan đến thực tế

Cở sở phương pháp: Khi chứng minh sựtồn sốđối tượng thỏa mãn điều kiện đó, ta thường sửdụng nguyên lí Dirichlet

Điều quan trọng phải xác định “thỏ” “lồng” * Ví dụ minh họa:

Bài toán Một tổ học tập có 10 học sinh Khi viết tả, cảtổđều mắc lỗi, bạn Bình mắc nhiều lỗi (mắc lỗi) Chứng minh tổ có bạn mắc sốlỗi

Hướng dẫn giải

Ta coi “thỏ” học sinh (trừ bạn Bình) nên có thỏ; “lồng” sốlỗi tả học sinh mắc phải nêncó lồng: lồng i gồm học sinh mắc i lỗi (i = 1, 2, 3, 4) Có thỏnhốt vào lồng, mà = 4.2 + 1, nên theo nguyên lí Dirichlet tồn lồng chứa khơng + = thỏ, tức có bạn mắc sốlỗi

Bài tốn Ởmột vịng chung kết cờvua có đấu thủtham gia Mỗi đấu thủđều phải gặp đủ7 đấu thủcòn lại, người trận Chứng minh rằng, thời điểm đấu, bao giờcũng có hai đấu thủđã đấu sốtrận

Hướng dẫn giải

Ta coi “thỏ” đấu thủ nên có thỏ; “lồng” số trận đấu đấu thủ nên có lồng: “lồng i” gồm đấu thủđã thi đấu i trận (với i = 0, 1, 2, 3, 4, 5, 6, 7)

Ta thấy lồng lồng không đồng thời tồn tại, có đấu thủchưa đấu trận khơng có đấu thủnào đấu đủ7 trận, có đấu thủđã đấu đủ7 trận khơng có chưa đấu trận

Như vậy, có lồng chứa thỏ nên theo nguyên lí Dirichlet tồn lồng chứa khơng thỏ, tức thời điểm cược đấu ln tìm đấu thủđã đấu dùng sốtrận

(1≤ ≤ −r k 1)

(126)

Bài tốn Có nhà khoa học viết thư trao đổi với hai đề tài: bảo vệ mơi trường chương trình dân số Chứng minh có ba nhà khoa học trao đổi vềmột đềtài

Hướng dẫn giải

Gọi nhà khoa học A, B, C, D, E, F

Nhà khoa học A sẽviết thư trao đổi với nhà khoa học lại đềtài, có

nên theo ngun lí Dirichlet tồn nhà khoa học (chẳng hạn B, C, D) nhà khoa học A trao đổi vềcùngmột đềtài (chẳng hạn đềtài môi trường)

Trong ba nhà khoa học B, C, D có hai người trao đổi vềđềbài môi trường (chẳng hạn B, C) ta chọn A, B, C trao đổi vềmột đềtài

Nếu ba nhà khoa học B, C, D khơng có hai người nàotrao đổi vềđềtài mơi trường họ sẽtrao đổi với vềđềtài dân số, ta sẽchọn B, C, D trao đổi đề tài

(Ởđây coi nhà khoa học (trừA) “thỏ” nên có thỏ, coi đềtài “lồng” nên có lồng vận dụng ngun lí Dirichlet tổng quát)

Dạng 5: Bài toán liên quan đến xếp

* Cơ sở phương pháp: Các tốn xếp chỗ, phân cơng việc khơng địi hỏi nhiều kiến thức kĩ tính tốn, chúng chủ yếu kết hợp suy luận lơgic để xét khả có thểxảy với nguyên lí Dirichlet

* Ví dụ minh họa:

Bài tốn Có 20 người định bơi thuyền 10 thuyền đôi Biết hai người A B mà khơng quen tổng số người quen A người quen B không nhỏ 19 Chứng minh có thểphân cơng vào thuyền đơi cho thuyền hai người quen

Hướng dẫn giải

Nếu 20 người khơng có hai người quen tổng số người quen hai người Điều mâu thuẫn với giả thiết tổng số người quen hai người không nhỏhơn 19 Vậy tồn sốcặp quen

Ta xếp cặp quen nhauđó vào thuyền đôi Gọi k số lượng thuyền lớn mà ta có thểxếp cặp quen vào thuyền kí hiệu thuyền thứ i xếp hai người Aivà Biquen

Giả sử , kí hiệu tập hợp M gồm người chưa xếp vào thuyền nào, tức gồm người đôi không quen Chọn hai người A B tập hợp M Theo tổng số người quen A số người quen B không nhỏ 19 người quen A quen B xếp vào thuyền Như có 19 người quen hệquen A B xếp vào nhiều thuyền đơi (trừ1 thuyền A, B chưa

5=2.2 1+

(1≤ ≤i k)

9

k

(127)

xếp), mà 19 = 9.2 + nên theo nguyên lí Dirichlet tồn thuyền chở2 người quen A B Nhưng ta có thểxếp lại sau: k – thuyền giữ nguyên, thuyền thứ k xếp Ak B, thuyền thứk + xếp A Bk Điều mâu thuẫn với giả sử

Theo cách xếp ta tiếp tục xếp đến hết 10 thuyền cho thuyền hai người quen

Dạng 6: Chứng minh bất đẳng thức

* Cơ sở phương pháp: * Ví dụ minh họa:

Bài toán Chứng minh:trong ba sốthực ln tìm hai sốcó tích khơng âm

Hướng dẫn giải

Ta coi “thỏ” sốthực nên có thỏ; coi “lồng loại số(sốkhơng âm số âm) nên có lồng Có thỏ nhốt vào lồng nên theo nguyên lí Dirichlet tồn thỏ chứa lồng, tức tồn hai sốkhông âm (hoặc sốâm), tích chúng sẽthành số khơng âm

Bài toán Chứng minh bốn số khác tùy ý lấy từ tập hợp có hai sốx, y thỏa mãn

Hướng dẫn giải

Ta có

Xét ba tập hợp: Với số có dạng (với ) thuộc vào ba tập hợp B, C, D nên theo nguyên lí Dirichlet tồn hai sốthuộc tập hợp, tập hợp B C Gọi hai sốđó

ta có

PHẦN 2: CÁC BÀI TỐN SỬ DỤNG NGUYÊN LÝ CỰC HẠN A KiÕn thøc cÇn nhí

Ngun lí cực hạn có dạng đơn giản sau:

Nguyên lí 1: Trong tập hợp hữu hạn khác rỗng sốthực ln có thểchọn sốbé sốlớn

Nguyên lí 2: Trong tập hợp khác rỗng sốtựnhiên ln ln có thểchọn số bé

4

{1, 2, 3, , }

A= 0< x−4 y <1

x A

∀ ∈

1≤ x≤3

{ |1 2};

B= b ≤ ≤b C={ | 2c ≤ ≤c 3} D={3}

x xA

4 x,4 y,

4

0< xy <1

(128)

Ngun lí dùng đểgiải tốn mà tập hợp có đối tượng phải xét tồn đối tượng có GTLN, GTNN theo nghĩa Ngun lí cực hạn thường sửdụng kết hợp với phương pháp khác đặc biệt phương pháp phản chứng Nguyên lí vận dụng trường hợp tập giá trịcần khảo sát tập hữu hạn (Ngun lí 1) vơ hạn tồn GTLN GTNN (Nguyên lí 2) Để vận dụng nguyên lí cực hạn giải tập hình học tổ hợp, người ta thường dùng lược đồchung đểgiải tập sau:

- Đưa tốn xét vềdạng sửdụng ngun lí nguyên lí đểchứng tỏrằng tất cảcác giá trịcần khảo sát tốn có GTLN GTNN

- Xét tốn tương ứng nhận GTNN GTLN

- Chỉra mâu thuẫn đưa giá trịlớn nhỏhơn GTLN GTNN mà ta khảo sát Theo nguyên lí PP phản chứng ta suy điều phải chứng minh

B CÁC VÍ DỤ MINH HỌA

Bài tốn Chứng minh bốn đường trịn có đường kính bốn cạnh tứ giác lồi phủ kín tứ giác cho

Hướng dẫn giải

Lấy M điểm tùy ý tứ giác lồi Có hai khả xảy

1) Nếu M nằm đường biên tứ giác lồi, tức M nằm cạnh tứ giác ABCD Khi M nằm đường trịn có đường kính cạnh Trong trường hợp kết luận toán hiển nhiên

3) Nếu M nằm bên tứ giác lồi ABCD Khi ta có ∠AMB + ∠BMC + ∠

CMD + ∠DMA = 3600 Theo nguyên lý cực hạn tồn max

{∠AMB,∠BNC,∠CMD,∠DMA}= ∠BMC Khi ∠BMC ≥ 900 (1) Từ (1) suy M

nằm nằm đường trịn đường kính BC Vậy dĩ nhiên M bị phủ đường tròn Như M điểm tùy ý tứ giác ABCD, ta suy bốn hình trịn phủ kín tứ giác lồi cho Đó điều phải chứng minh

(129)

Bài toán Cho ABC tam giác nhọn Lấy điểm P tam giác Chứng minh khoảng cách lớn khoảng cách từ P tới ba đỉnh A, B, C tam giác không nhỏ hai lần khoảng cáchbé khoảng cách từ P tới ba cạnh tam giác

Hướng dẫn giải

Gọi A B C1, 1, tương ứng hình chiếu P xuống BC, AC, AB Ta có

( )

0

1 1 1 360

APC C PB BPA A PC CPB B PA

∠ + ∠ + ∠ + ∠ + ∠ + ∠ =

Theo nguyên lý cực hạn, tồn max{∠APC1,∠C PB1 ,∠BPA1,∠A PC1 ,∠CPB1,∠B PA1 } Không

giảm tổng quát, cho : max{∠APC1,∠C PB1 ,∠BPA1,∠A PC1 ,∠CPB1,∠B PA1 }= ∠BPA1 (2)

Từ (1) (2) dễ dàng suy ∠BPA1≥ 600(3) Từ (3) ta đến

1

1 os

2

PA

c BPA

PB

∠ = ≤ hay PB ≥ 2PA1 (4) Từ (4) suy

{ }

ax , ,

m PA PB PCPBPA ≥ min{PA PB PC1, 1, 1} Đó điều pcm

Bài tốn Trên mặt phẳng có số điểm có tinh chất với hai điểm hệ điểm ln tìm điểm thứ ba số điểm thẳng hàng với chúng Chứng minh tất điểm cảu hệ điểm thẳng hàng

Hướng dẫn giải

Giả sử kết luận toán không đúng, tức điểm cho không thẳng hàng Xét tập hợp sau A={h h/ >0 h khoảng cách từ điểm đường

thẳng nối hai điểm hệ }

Do giả thiết phản chứng nên A ≠Ø Mặt khác, A tập hợp có hữu hạn phần tử ( \do có số hữu hạn điểm cho) Theo nguyên lý cực hạn, tồn mọt giá trị nhỏ h* Giả sử h* khoảng cách từ điểm M xuống đường thẳng qua B,C ( Ở M,B, C thuộc vàosố điểm cho) Gọi ∆ đường thẳng nối B, C Do M∉∆ ( h* > 0), nên theo giả thiết tồn điểm D∈∆ Kẻ MH ⊥∆, MH = d* Rõ ràng ba điểm B, C, D pahir có hai điểm phía so với H

(130)

Khơng làm giảm tính tổng qt, ta cho C, Dnằm phía với H C nằm đoạn HD, Kẻ HE ⊥MD CF ⊥MD Rõ ràng ta có : CF < HE < MH Nói cách khác CF < d* Chú ý cho C,M,D nằm điểm cho, nên CF ∈A Do CF < d* Điều mâu thuẫn với định nghĩa d* Vậy giải thiết phản chứng sai, tức tất điểm cho phải thẳng hàng Đó đpcm

Bài tốn Trên mặt phẳng cho số hữu hạn điểm không nằm đường thẳng Chứng minh tồn ba điểm cho đường tròn qua ba điểm khơng chứa điểm bên

Hướng dẫn giải

Vì số điểm cho hữu hạn chúng không nằm đường thẳng, nên lấy bao lồi hệ điểm, ta đa giác Giả sử đa giác lồi A A1 Ap Như

các điểm lại cho phải nằm bao lồi Gọi A Ak, k+1là hai đỉnh liên tiếp của đa

giác lồi( nghĩa xét cạnh tùy ý A Ak k+1) Khi điểm cho nằm nửa

mặt phẳng xác định A Ak k+1 Từ giả thiết suy tập hợp điểm cho không thuộc

k k

A A+ khác rỗng Vì theo nguyên lý cực hạn, tồn C cho

1 ax 1

k k k k

A CA+ m A A A+

∠ = ∠ , giá trị lớn lấy theo i=1,n

i ≠ k, i ≠ k + 1( giả sử A A1, 2, An hệ hữu hạn điểm cho trước) Khi đường trịn ngoại

tiếp ta giác CA Ak k+1 đường tròn cần tìm

Bài tốn Bên hình vng cạnh cho n điểm cho khơng có ba điểm thẳng hàng Chứng minh tồn môt tam giác có đỉnh điểm cho diện tích S thỏa mãn bất đẳng thức

2

S n

< −

Hướng dẫn giải

(131)

Xét bao lồi n điểm nằm bên hình vng Vì khơng có ba điểm thẳng hàng, nên bao lồi đa giác lồi có k đỉnh( k≤n), ngồi điểm cho đỉnh đa giác lồi, nằm hẳn bên đa giác lồi Chỉ có hai khả xảy

1 Nếu k = n Khi số đường chéo xuất phát từ A1của đa giác bao lồi tạo thành cạnh đa giác ( n – 2) tam giác Gọi S diện tích tam giác nhỏ (n-2) tam giác

Vì tổng diện tích (n-2) tam giác nhỏ 1( ý diện tích hình vng chứa chọn ( n-2) tam giác này), suy

2

S n

< −

2 Nếu k < n Khi bên đa giác bao lồi A A1 2 Ak có (n-k) điểm Ak+1, Ak+2, , An Nối Ak+1với đỉnh A1; A2; Ak Khi có k tam giác Ak+1A1A2, Ak+1A2A3 ; ; Ak+1AkA1

Vì khơng có ba điểm thẳng hàng, Nên điểm Ak+2, , Anphải nằm hẳn k tam giác nói Giả sử Ak+2nằm hẳn tam giác Nối Ak+2 với ba đỉnh tam giác này, từ tam giác có ba tam giác Sau lần làm số tam giác tăng lên Như ta đến:

k+ 2(n- k – 1) = 2n – k – = (n – 2) + (n – k) tam giác.mà bên tam giác khơng có điểm thuộc n điểm cho Gọi S diện tích bé tam giác , thì:

(n – ) n – k ( ) 12

S

n

< <

+ − ( Do n – k >0)

Bất đẳng thức

2

S n

<

− chứng minh

Bài tốn Bên hìnhvng cạnh cho n điểm Chứng minh tồn tam giác có đỉnh điểm cho đỉnh hình vng cho diện tích S thỏa mãn bất đẳng thức:

2( 1)

S n

≤ +

Hướng dẫn giải

(132)

Gọi A, B, C, D bốn đỉnh hình vng A1; A2; Anlà n điểm nằm hình vuông Nối A1với đỉnh A, B, C, D Khi ta hình tam giác

*) Nếu A2nằm trong tam giác ( Giả sử A2nằm tam giác ADA1) Ta nối A2 với A, D, A1 Sau nối xong, số tam giác tăng thêm

*) Nếu A2nằm cạnh chung (Ví dụ A2 ∈A1D) nối A2với A C Khi số tam giác tăng thêm

Như trường hợp, số tam giác tăng thêm

Với điểm A3; Anta làm tương tự Cuối số tam giác tạo thành là: + 2(n-1) = 2n + tam giác Các tam giác có đỉnh đỉnh hình vng n điểm cho Khi đó, tổng diện tích 2n +2 tam giác diện tích hình vng (bằng 1) Theo ngun lý cực hạn, tồn tam giác có diện tích nhỏnhất 2n + tam giác Gọi diện tích S

2( 1)

S n

+ (Điều cần chứng minh)

Bài tập tương tự:

Bài toán 1.1 Cho n điểm nằm tam giác ABC có diện tích

cm CMR: Từ n điểm

đó với điểm A, B, C tồn tam giác có diện tích khơng lớn

2

1

3 2(+ n−1)cm

Bài toán 1.2 (tổng quát) Cho n điểm nằm đa giác lồi m đỉnh có diện tích

cm

CMR: Từ n điểm với m đỉnh đa giác, tồn tam giác có diện tích khơng lớn

2( 1)cm

m+ n

Bài tốn Trong bảng vng kích thước n nx ô vuông, người ta viết số cho tổng số có mặt “chữ thập” (tức hình gồm hàng cột) khơng nhỏ a Tính giá trị nhỏ tổng ô bảng

Hướng dẫn giải

(133)

Lấy hàng có tổng số hàng nhỏ Sau xét tổng tất có “chữ thập” lập nên từ hàng

n“chữ thập” vậy, theo điều kiện toán Ta suy ra, tổng số ghi n“chữ thập” không nhỏ n.a

Dễ thấy tổng nói tổng tất số bảng cộng thêm (n 1)lần tổng số hàng lấy

Gọi tổng số bảng N, tổng số hàng lấy m, từ suy luận ta suy ra:

N +(n 1).m n.a (1)

Theo định nghĩa số m, ta có: m N n

≤ (2)

Từ (1) (2) ta suy ra: ( 1)

2

N n

N n n a N a

n n

+ − ≥ ⇔ ≥

Mặt khác, chọn tất ô bảng

2

a

n− Khi tổng tất số ghi

mọi hình chữ thập là: (2 1)

2

a

n a

n

− =

Phép ghi hợp lệ Với cách ghi này, tổng số ghi vào ô bảng

2

2

n a n

Vậy giá trị nhỏ tổng cần tìm

2

n a N

n

= −

Bài tốn Trong khơng gian cho số hữu hạn điểm mà khơng có điểm chúng nằm mặt phẳng cho thể tích tứ diện tạo đỉnh điểm cho không lớn Chứng minh tất điểm phủ tứ diện tích 27

Hướng dẫn giải

Do số lượng điểm cho hữu hạn, nên số lượng tứ diện tạo thành hữu hạn Theo nguyên lý cực hạn, tồn tứ diện mà ta gọi A1B1C1D1) tích lớn Qua đỉnh A1; B1; C1; D1dựng mặt phẳng song song với mặt tứ diện, ta nhận tứ diện ABCD.Dễ dàng chứng minh

(134)

A1; B1; C1; D1 tương ứng trọng tâm tam giác BCD,ACD,ABD, ABC Từ ta có:

1 1 1

27 A B C D

ABCD

V

V = (1)

Do giả thiết VA B C D1 1 1 ≤1, nên từ (1) ta suy

27 ABCD

V ≤ (2)

Bây ta chứng minh tất điểm nằm tứ diện ABCD

Giả sử: Tồn điểm M(trong số điểm cho), cho M không thuộc tứ diện ABCD (3)

Khi đỉnh tứ diện ABCD (có thể cho đỉnh B) cho B M nằmtrong hai nửa không gian xác định (ACD)

Suy VMA C D1 1 >VB A C D1 1 1 (1)

Bất đẳng thức (4) chứng tỏ MA1C1D1là tứ diện tạo đỉnh điểm chốc thể tích lớn thể tích tứ diện B1A1C1D1 Điều mâu thuẫn vơi cách định nghĩa tứ diện A1B1C1D1 (vô lý) Suy điều phải chứng minh

MỘT SỐ DẠNG TỐN HÌNH HỌC TỔ HỢP THƯỜNG GẶP 1.Dạng tập tơ màu, bảng vng

Bài tốn Trong mỗiơ bàn cờ kích thước 5x5 có bọ dừa Vào thời điểm tất bọ dừa bị sang bên cạnh (ngang dọc) Có thể khẳng định sau bọ dừa di chuyển ln có bàn cờ khơng có bọ dừa khơng?

(Đề thi giao lưu HSG mơn Tốn lớp TP Vĩnh n năm học 2012-2013) Hướng dẫn giải

Ta tô đen - trắng bàn cờ hình vẽ Khi số ô đen nhiều số ô trắng Như số bọ dừa ô đen nhiều số bọ dừa ô trắng Do bọ dừa di chuyển sang ô bên cạnh(ngang dọc), sau di chuyển đen chứa bọ dừa ô trắng

Mà số bọ dừa ô đen nhiều số bọ dừa ô

trắng nên sau bọ dừa bị có ô đen bị bỏ trống

Vậy: Có thể khẳng định sau di chuyển ln có bàn cờ khơng có bọ dừa

Bài tốn Trên lưới vng cạnh Người ta tô màu ô màu đen trắng xen kẽ Tính bán kính lớn đường trịn quaba ô trắng

(135)

Hướng dẫn giải

Trên lưới ô vuông cạnh Người ta tô màu ô màu đen trắng xen kẽ Tính bán kính lớn đường trịn qua ba trắng

- Xét đường trịn thuộc trắng: Đường kính (1)

- Xét đường tròn qua nhiều trắng => đường trịn phải qua đỉnh trắng Khơng tính tổng qt, giả sử đường trịn qua đỉnh A trắng ABCD

+) Nếu đường tròn qua đỉnh liên tiếp trắng ( A, B) Khi ta lại xét hai trường hợp: *) Đường tròn qua A, B, E Khi đường trịn ,

2 I

       

*) Đường tròn qua A, B, G Khi đường trịn ; 10 K

 

 

 

  (2)

+) Nếu đường trịn qua đỉnh đối diện trắng, Giả sử (A, C) Ta lại xéthai trường hợp:

*) Qua A, C, M (Tương tự qua A, B, G) *) Qua A, C, N (Tương tự qua A, B, G) Cả hai trường hợp bán kính đường trịn 10

2 (3)

Từ (1), (2) (3) ta suy bán kính lớn đường trịn thỏa mãn u cầu đề 10

2

Bài toán Cho điểm điểm nối với tạo thành tam giác có cạnh tô hai màu xanh đỏ CMR: Bao tồn tam giác có cạnh màu

Hướng dẫn giải

(136)

Gọi A điểm, đoạn thẳng nối A với điểm lại tô hai màu xanh đỏ nên tồn cạnh màu Giả sử AB, AC, AD Xét trường hợp:

+Trường hợp 1:AB, AC, AD tô màu đỏ

Xét ∆BCD Nếu có cạnh tơ màu đỏ (giả sử BC) ∆ABC màu đỏ (hình

1)

Nếu khơng có cạnh ∆BCD tơ màu đỏ ∆BCD có cạnh màu xanh (hình

2)

+Trường hợp 2:AB, AC, AD tô màu xanh Chứng minh tương tự Vậy tồn tam giác có cạnh màu

Bài tốn Trên tờ giấy có kẻ vơ hạn ô vuông ô tô hai màu xanh đỏ Cho hình chữ nhật kích thước 2x3 có hai màu đỏ Xét hình chữ nhật có kích thước 2013x2014 Tính số đỏ

Hướng dẫn giải

Ta có nhận xét sau :

Mọi hình chữ nhật 1x3 chứa ô màu đỏ

Thật vậy, giả sử kết luận nhận xét khơng đúng, tức tồn hình chữ nhật 1x3 có số màu đỏ khơng khác Khơng giảm tổng qt giả sử hình chữ nhật AKHD kích thước 1x3 có hai đỏ ( khơng khơng có đỏ nào, khơng thể ba hình chữ nhật 2x3 có hai đỏ mà thơi )

Trường hợp AKHD khơng có đỏ lí luận tương tự Cũng cho hai đỏ AKHD ô , ô8

Xét hình chữ nhật BFNA Đó hình chữ nhật 2x3 , nên theo giả thiết có hai đỏ mà hai ô đỏ, 1,2,4,5 màu xanh

Xét hình chữ nhật BCHK, từ giả thiết ô 1,2,4,5 màu xanh nên ô ,6 màu đỏ

Xét hình chữ nhật ECDM kích thước 2x3, ô 3,6,8 màu đỏ suy mâu thuẫn Vậy giả thiết phản chứng sai Nhận xét chứng minh

Vì 2013 chia hết cho 2013: 3=671 hình chữ nhật kích thước 2013x2014 chia thành 2014x671 hình chữ nhật 1x3

Vậy số đỏ hình chữ nhật kích thước 2013x2014 2014x671 Số đỏ cần tìm 1351394

(137)

3.Dạng tạo đa giác bao

Bài toán Cho đa giác lồi có diện tích k

cm CMR: Tồn hình bình hành có

diện tích khơng vượt 2k

cm chứa toàn đa giác

Hướng dẫn giải

Gọi C đỉnh cách xa AB (hình vẽ)

+Trường hợp 1: Nếu AC đường chéo đa giác lồi Qua C kẻ a// b (A,Bb)

Gọi D,E đỉnh cách xa AC nhất, qua D kẻ đường thẳng d // AC, qua E kẻ đường thẳng c // AC Gọi MNPQ hình bình hành tạo a,b,c,d suy đỉnh đa giác nằm

trong biên hình bình hành MNPQ Ta chứng minh 2

MNPQ

Sk cm , thật vậy: Gọi Sdlà diện tích đa giác

Ta có: SACD+SACESd

2SMNPQSd= k ⇔ SMNPQ ≤2 (k cm2)

+Trường hợp 2: Nếu AC cạnh đa giác lồi Gọi E đỉnh cách xa AC ( Chứng minh tương tự)

Suy ta có điều phải chứng minh

4.Phương pháp qui nạp toán học:

Để chứng minh mệnh đề Anđúng với n ∈N*: -B1:chỉ mệnh đề với n=1 tức A1đúng

-B2: giả sử mệnh đề với n = k ( k ∈ N*) Akđúng -B3: chứng minh Ak+1đúng ( mệnh đề với n=k+1) Kết luận Anđúng với n ∈ N*

Bài toán

Người ta dùng loại gạch để lát phịng hình vng n x nvà ln để trống goc phịng

a) Hãy nêu cách lát phịng 4x4, 8x8 vng

b) Chứng tỏ người ta lát phịng kích thước 2kx 2k cho trống ô

Hướng dẫn giải

a) Xét hình vng kích thước 4x4 vng Ta chia hình vng thành hình vng kích thước x Hình thứ ta đặt viên gạch chơ góc bỏ trống, hình cịn lại ta lát

(138)

sao cho ô trống quay vào phần tâm hình vng Cuối ta lát viên gạch vào cịn thiếu (Hình 2)

-) Để lát hình vng x vng, ta lát tương tự (Chia hình vng x thành hình vng x 4) (H3)

b) Ta chứng minh PP quy nạp - Xét với k = 1: Bài tốn ln

- Giả sử toán với với k = n Nghĩa là, lát hình vng 2nx2nsao cho cịn trống ô (i; j)

- Ta chứng minh toán với k = n +1

Thật vấy: Xét hình vng 2n+1x 2n+1 = 4x(2nx2n) Ta chia hình vng 2n+1x 2n+1thành hình vuông 2nx2n Theo giả thiết quy nạp, lát hình 2nx2nsao cho trống (i, j) Ba hình vng 2nx2ncịn lại ta để trống góc lát vào trung tâm (tương tự H3) cuối lát viên gạch vào ô trống Bài toán chứng minh

(H1)

(H2)

(H3)

Bài toán CMR: số đường chéo đa giác lồi n cạnh (n≥4) Sn =

) (nn

Hướng dẫn giải

Ta chứng minh Sn =

2 ) (nn

(1) với n≥4

+) Ta thấy (1) với n=4 S4=2, tứ giáccó đường chéo

+) Giả sử khẳng định (1) với n=k (k≥4) tức đa giác lồi k cạnh có

2 ) (kk

đường chéo.+) Ta chứng minh đa giác lồi k+1cạnh có

2 ) )( (k+ k

đường chéo

Thật thêm đỉnh thứ k+1 (hình vẽ) có thêm k-2 đường chéo nối từ Ak+2 đến A2,

A3,…, Ak−1, cạnh A1Ak trở thành đường chéo Do đó, Sk+1 = Sk+(k-2)+1=

) (kk

+k-1=

2 ) )( (k+ k

(139)

Vậy khẳng định (1) với n thuộc N*, n≥4

MỘT SỐ DẠNG BÀI TỔNG HỢP KHÁC

Ví dụ 1: Cho bàn hình chữ nhật Hai người chơi sau: người thứ dùng đồng xu màu trắng đặt lên bàn, sau người thứ hai đặt đồng xu đen lên bàn vị trí mà trước chưa có đồng xu đặt khơng cịn chỗ để đặt đồng xu Biết tất đồng xu Người đến lượt mà không đặt đồng xu lên bàn người thua Chứng minh có cách chơi để người thứ luôn thắng

(Đề thi HSG lớp 7-TP Vĩnh Yên năm học 2012-2013)

Hướng dẫn giải

Ta tô đen - trắng bàn cờ hình vẽ Khi số đen nhiều số ô trắng Như số bọ dừa ô đen nhiều số bọ dừa ô trắng Do bọ dừa di chuyển sang bên cạnh(ngang dọc), sau di chuyển ô đen chứa bọ dừa ô trắng Mà số bọ dừa ô đen nhiều số bọ dừa ô

trắng nên sau bọ dừa bị có đen bị bỏ trống

Vậy: Có thể khẳng định sau di chuyển có bàn cờ khơng có bọ dừa

Ví dụ 2: Trong bàn cờ kích thước 5x5 có bọ dừa Vào thời điểm tất bọ dừa bị sang bên cạnh (ngang dọc) Có thể khẳng định sau bọ dừa di chuyển ln có bàn cờ khơng có bọ dừa khơng?

(Đề thi HSG lớp 8-TP Vĩnh Yên năm học 2012-2013)

Hướng dẫn giải

Để đảm bảo thắng người thứ phải có chiến lược chơi sau: + Đầu tiên anh chiếm vị trí trung tâm, tức đặt đồng xu trắng

Sao cho tâm đồng xu trùng với tâm hình chữ nhật (Vị trí A)

+ Giả sử người chơi thứ đặt đồng xu đen lên bàn (Tâm đồng xu B) + Khi điểm đối xứng với B qua tâm A D chắn trống Người thứ đặt đồng xu trắng

sao cho tâm đồng xu trùng D + Luật chơi tiếp tục Nghĩa sau người thứ hai đặt đồng xu người thứ chọn vị trí đối xứng qua tâm A để đặt đồng xu (lưu ý vị trí đối xứng ln chưa có đồng xu đặt trước đó)

(140)

Vì vậy, người thứ hai cịn người thứ bước Vì người thứ khơng thua

Do mặt bàn có diện tích hữu hạn, nên thực theo chiến thuật người trước chắn đảm bảo chiến thắng thuộc

Ví dụ 3: Có 2008 gà nhốt vào 1004 chuồng, chuồng có con.Sau ngày người ta lại thay đổi vị trí gà cho khơng có hai gà nhốt chung chuồng trước đólại nằm chuồng lần nữa.Hỏitối đa có ngày làm vậy?

(Đề khảo sát HSG Huyện Vĩnh Tường năm học 2008-2009)

Hướng dẫn giải

Vẽ đa giác 2007 cạnh nội tiếp đường tròn Ký hiệu tâm 2008, Còn đỉnh 1, 2, 3, ….2007 Ký hiệu đoạn thẳng nối i với j i – j với i j, =1, 2008

Xét bán kính – 2008 Do tính chất đa giác nên ta thấy 1003 dây cung sau vng góc với bán kính ấy: – 2007; – 2006; – 2005; ; 1004 – 1005

Xét bán kính – 2008 Tương tự ta có 1003 dây cung sau vng góc với bán kính ấy: 1- 3; 2007 – 4; 2006 –

Xét bán kính – 2008 Tương tự ta có 1003 dây cung sau vng góc với bán kính ấy: – 4; – 5; 2007 – 6;

Cứ làm cuối xét bán kính2007 – 2008, ta có 1003 dây cung sauđây vng góc với bán kính ấy: 2006 – 1; 2005 – 2; 2004 – 3;

Dựa vào nhận xét cách đánh số gà từ – 2008,ta cách xếp gà theo yêu cầu toán sau:

+ Ngày thứ xếp vào chuồng đôi gà sau:1 – 2008; 2- 2007; ; 1004 – 1005 + Ngày thứ hai xếp vào chuồng đôi gà sau: 2- 2007;1-3;2007 – 4; ; 1006 – 1005 + Ngày thứ ba xếp vào chuồng đôi gà sau: – 2008; 2- 4;1-5; ; 1007 – 10056 Tương tự

+ Ngày thứ 2007 xếp vào chuồng đôi gà sau: 2007 – 2008; 2006- 1; ; 1004 – 1003 Mặt khác khơng có qua 2007 ngày gà chuồng với 2007 gà cịn lại

Vậy tối đa có 2007 ngày để xếp gà theo yêu cầu đề

(141)

Ví dụ 4: Xét 20 số nguyên dương 1, 2, 3,…, 20 Hãy tìm số ngun dương k nhỏ

nhất có tính chất: Với cách lấy ksố phân biệt từ 20 số trên, lấy hai số phân biệt a b cho a b+ số nguyên tố

(Đề tuyển sinh THPT chuyên Vĩnh Phúc 2013-2014) Hướng dẫn giải

Xét tập hợp {2, 4, 6,8,10,12,14,16,18, 20}, ta thấy tổng hai phần tử tập hợp

đều khơng phải số nguyên tố

Do k ≥11, ta chứng minh k=11 số nhỏ thỏa mãn yêu cầu toán

Thật vậy, ta chia tập hợp A={1, 2, 3, , 20} thành 10 cặp số sau:

( ) (1, , 3,16 , 4,19 , 5, , 7,10 , 8, , 11, 20 , 12,17 , 13,18 , 14,15) ( ) ( ) ( ) ( ) ( ) ( ) ( ) ( )

Tổng hai số cặp số số nguyên tố

Khi tập A có 11 phần tử tồn hai phần tử thuộc vào

trong 10 cặp số Suy A ln có hai phần tử phân biệt có tổnglà số

nguyên tố

Ví dụ 5: Hỏi có haykhơng 16 số tự nhiên, số có ba chữ số tạo thành từ ba chữ số

a, b, cthỏa mãn hai số chúng khơng có số dư chia cho 16?

(Đề tuyển sinh THPT chuyên Vĩnh Phúc 2013-2014) Hướng dẫn giải

Trả lời: Không tồn 16 số Thậtvậy, giả sử trái lại, tìm 16 số thỏa mãn Khi đó, ta có 16 số dư phân biệt chia cho 16: 0,1,2,3,4,5,6,7,8,9,10,11,12,13,14,15; có số chẵn, số lẻ

Do đó, ba chữ số a, b, ckhác tính chẵn lẻ, giả sử hai chữ số chẵn a, bvà chữ số lẻ c. Có số lẻ tạo thành từ chữ số này: aac abc acc bac bbc bcc cac cbc ccc, , , , , , , ,

Gọi x x1, 2,…,x9 số có hai chữ số thu từ số cách bỏ chữ số c(ở

hàng đơn vị) Khi x ci ≡/ x cj (mod16)⇔16 khơng ước x cix cj tức xixj

không chia hết cho

Nhưng số x x1, 2,…,x9 có ba số lẻ ac bc cc, , nên số số x x1, 2,…,x9

ln có hai số có số dư chia cho 8, mâu thuẫn

Tương tự, trường hợp ba số a, b, ccó hai số lẻ, số chẵn khơng xảy Ví dụ 6:

Cho 2011 điểm nằm mặt phẳng, khơng có điểm thẳng hàng tam giác có đỉnh 2011 điểm cho có diện tích nhỏ Chứng minh đặt 2011 điểm tam giác có diện tích

(Đề khảo sát HSG Huyện Vĩnh Tường năm học 2011-2012)

Hướng dẫn giải

(142)

Giả sử tam giác ABC tam giác có đỉnh 2011 diểm cho có diện tích lớn => SABC < Qua A, B, C kẻ đường thẳng song song với cạnh tam giác ABC ta tam giác MNP có SMNP<4

Ta chứng minh ko có điểm 2011 điểm cho nằm tam giác MNP Thật vậy: Giả sử có điểm D nằm ngồi tam giác MNP

Khi SDBC> S ABC trái với cách chọn tam giác ABC có diện tích lớn

Suy 2011 điểm cho khơng nằm ngồi tam giác MNP có diện tích nhỏ Vì 2011 điểm nằm tam giác KEF đồng dạng với tam giác MNP(h.vẽ) có diện tích

Ví dụ 7:

Mỗi điểm mặt phẳng tô ba màu Đỏ, Xanh, Vàng

Chứng minh tồn hai điểm A B, tô màu mà độ dài AB=1.

(Đề thi HSG Tỉnh Vĩnh Phúc năm học 2010 2011) Hướng dẫn giải

- Giả sử trái lại, với cách tô, không tồn hai điểm màu mà có khoảng cách Xét hai điểm M N MN, : = tồn điểm P Q,

sao cho tam giác MPQ NPQ, tam giácđều có

độ dài cạnh

Khi đó, hai điểm có khoảng cách tơ hai màu khác nhau, nên M N, phải tô

một màu,

chẳng hạn tô P: Đỏ, Q: Vàng M, N: phải tơ màu Xanh, (Hình vẽ)

- Từ đó, điểm Mđược tơ màu Xanh, điểm nằm đường trịn tâm M,bán kính tơ màu Xanh Nhưng đường trịn ln có hai điểm mà

khoảng cách chúng Mâu thuẫn với giả thiết phản chứng Từ suy điều phải chứng minh

Ví dụ 8:

Trong bảng hình vng gồm 10 10× vng (10 hàng, 10 cột), người ta viết vào ô

vuông số tự nhiên từ đến 100 theo cách sau: hàng thứ nhất, từ trái sang phải, viết số từ đến 10; hàng thứ hai, từ trái sang phải, viết số từ 11 đến 20; hết hàng thứ 10 Sau cắt bảng hình vng thành hình chữ nhật nhỏ kích thức 2× 1× Tính tích số hai số hình chữ nhật nhỏ cộng 50

tích lại Cần phải cắt hình vng để tổng tìm nhỏ ? Hãy tính giá trị nhỏ

(Đề KS HSG Huyện Vĩnh Tường năm học 2011-2012) Hướng dẫn giải

C A

B

M N

P

D

K E

F

(143)

Cắt hình vng thành hình chữ nhật cỡ 2× 1× tất 50 hình Giả sử

trong hình thứ kcó số a bk, k akbk =1 akbk =10

Ta có ( )

2 2 2 k k k k k k a b a b

a b⋅ = + − − suy ( ) ( )

50 50 50

2

2

1 1

1

2

k k k k k k

k k k

a b a b a b

= = =

⋅ = + − −

∑ ∑ ∑

Trong 50 ( 2) ( 2)

1 1 100 101 201

1 100 169175

2 k k k 2

a b

=

⋅ ⋅

+ = + + + = ⋅ =

∑ 

và số ( )2

k k

ab hoặc 100

Do đó, để tổng thu nhỏ nhất, ( )2

100, 1, 2, , 500

k k

ab = ∀ =k

Vì vậy, cần cắt hình vng thành hình chữ nhật với kích thước 1× Và giá trị

nhỏ tổng 169175 25 100 166675− × =

Ví dụ 9: Cho đa giác lồi A A1 2A100 Tại đỉnh Ak (k=1, 2, ,100), người ta ghi số

thực ak cho giá trị tuyệt đối hiệu hai số hai đỉnh kề

Tìm giá trị lớn giá trị tuyệt đối hiệu hai số ghi cặp đỉnh đa giác cho, biết số ghi đỉnh cho đôi khác

(Đề tuyển sinh THPT chuyên Vĩnh Phúc 2011-2012) Hướng dẫn giải

Xét đa giác lồi A A1 2A100 hình vẽ Khi

1

k k

aa + = akak+1 =3 (k =1, 2, , 99)

Khơng tính tổng qt, coi a1 nhỏ nhất, an

lớn (dễ thấy n≥2) Đặt max i j i j

d a a

= −

1

n

d =aa Ta chứng minh d =149

Nằm A A1, n, theo chiều kim đồng hồ có n−2 đỉnh có 100−n đỉnh, theo chiều

ngược kim đồng hồ Hơn giá trị tuyệt đối hiệu hai số kề khơng vượt q Do đód = a1−ana1−a2 + a2−a3 + + an−1−an ≤3(n−1) tương tự ta có

( )

3 100

d≤ − +n Suy (3( 1)) (3(100 1)) 300 150

2

n n

d≤ − + − + = =

150

d = hiệu hai số ghi hai đỉnh kề hay ta có

1 3, 1, 2, , 99

i i

aa+ = i= 1 1 2( )

2

1, , 98

i i i i

i i i i

i i

a a a a

a a a a i

a a + + + + + + + − = −  ⇒ − = − ⇒ = =  ( ) ( )

1 100 2 99 100 99 100 99 99.3

a a a a a a a a a a a a a a

⇒ − = − + − + + − = − ⇒ − = − ⇒ =

Điều không xảy suy d =150 không thỏa mãn

Ta xây dựng trường hợp cho d =149 sau:

1 0, 2, k k

a = a = a =a − +

với k=2, 3,…, 52;a53 =a52 −2,ak =ak−1−3,k =54, 55,…,100

Khi hiệu lớn a53− =a1 149

Các số a a2, 3,…,a53 có dạng 3+ t, số a54,a55,…,a100 có dạng 147 3− k

(144)

Rõ ràng không tồn k t, cho 3+ =t 147 3− k⇔3(k+ =l) 145 (k t, ∈)

Suy điều phải chứng minh

Ví dụ 10: Một số tự nhiên dương gọi số “Đẹp”, hợp số khơng chia hết cho 2, 3, 5, Hỏi có tất số tự nhiên “Đẹp” nhỏ 2011

Hướng dẫn giải

Số “Đẹp” hợp số, không chia hết cho 2,3,5,7, nên phải tích củacác số ngun tố lớn 10

Do 133=2197> 2010 , nên số cần tìm có dạng a ba b c⋅ ⋅ với a,b, clà số nguyên tố

11≤ ≤ ≤a b c

Xét số “Đẹp” dạng a b⋅ với 11≤ ≤a b, a,b số nguyên tố :

Với a=11 11≤ ≤b 181, có 38 số

Với a=13 13≤ ≤b 151, có 31 số

Với a=17 17≤ ≤b 113, có 24 số

Với a=139 19≤ ≤b 103, có 20 số

Vớia=23 23≤ ≤b 83, có 15 số

Với a=29 29≤ ≤b 71, có 12 số

Với a=31 31≤ ≤b 61, có số

Với a=37 37≤ ≤b 53, có số

Với a=41 41≤ ≤b 47, có số

Với a=43 b=43, có số

Xét số “Đẹp” dạng a b c⋅ ⋅ với 11≤ ≤ ≤ ≤a b c 13, a,b,c số nguyên tố

Có số có dạng là: 11.11.11=1331; 11.11.13=1573 ; 11.13.13=1859 Suy số số “Đẹp” cần tìm S = 38+31+24+20+15+12+8+5+3+1+3 = 160 số Ví dụ 11:

Trong hộp có 2014 viên sỏi Có hai người tham gia trị chơi, người phải bốc 11 viên sỏi nhiều 20 viên sỏi Người bốc viên sỏi cuối thua Hãy tìm thuật chơi để đảm bảo người bốc người thắng

Hướng dẫn giải

Để đảm bảo thắng cuộc, nước cuối người bốc sỏi phải để lại hộp 11 viên sỏi Ở nước trước phải để lại hộp: 11 (20 11)+ + =42 viên sỏi

Suy người bốc sỏi phải đảm bảo hộp lúc 11 31+ k viên sỏi

Ta có (2014 11) : 31 64− = dư 19 Như người bốc sỏi đầutiên lần thứ

phải bốc 19 viên

Tiếp theo, đối phương bốc kviên sỏi (k=1, 2, , 20) người bốc sỏi phải

bốc 31−k viên sỏi, cuối để lại 11 viên sỏi cho đối phương

Ví dụ 12: Có điền hay khơng 100 số gồm 10 số -2, 10 số -1, 30 số 0, 40 số 1, 10 số vào ô bảng 10 10× (mỗi điền số gọi số hàng i tính từ lên cột j tính từ trái sang phải aij) cho thỏa mãn hai điều kiện:

i) Tổng số hàng, cột m; ii) Tổng số aijtrong bảng thỏa mãn i j  5m

Hướng dẫn giải

Không điền

(145)

Thật vậy, giả sử trái lại, điền số thỏa mãn Khi m= ⋅[10.( 2) 10.( 1) 30.0 40.1 10.2− + − + + + ]=3

10 số lẻ

Ta chia ô bảng thành loại: - Loại gồm ô hàng lẻ, cột lẻ - Loại gồm ô hàng lẻ, cột chẵn - Loại gồm ô hàng chẵn, cột lẻ - Loại gồm ô hàng chẵn, cột chẵn

Kí hiệu Sk tổng tương ứng tất ơloại k Khi S1+ S2là tổng sốtrên tất cảcác hàng lẻ, nên S1+ S2= 5m S2+ S4là tổng sốtrên tất cảcác cột chẵn, nên S2+ S4= 5m Loại loại gồm ô mà i - j chẵn, S1+ S4= 5m

Suy 2(S1+ S2+ S4) = 15m (1) Do m lẻ, nên VP(1) lẻ

Mà VT(1) chẵn: vơ lí Do điều giả sử sai

Vậy điền số thỏa mãn Ví dụ 13:

Có 40 học sinh lớp đứng thành vòng tròn quay mặt vào tâmđường tròn để tham gia trò chơi đếm số sau: Mỗi học sinh đếm dãy số tuần hoàn

1,2,1,2,1,2….lần lượt theo chiều kim đồng hồ học sinh A(lớp trưởng) Nếu học sinh đếm số phải rời khỏi vòng tròn Việc đếm tiếp tục học sinh - Học sinh coi thắng B học sinh giỏi Tốn, B tìm vị trí đứng để người thắng Hỏi B đứng vị trí theo chiều kim đồng hồ kể từ A?

Hướng dẫn giải

Xét trường hợp lớp có 32 học sinh lớp trưởng vị trí thứ ln thắng Như vậy, để B người thắng B phải đứng vị trí thứ sau loại (40 – 32) = người

Hay B phải đứng vị trí x + = 17 theo chiều kim đồng hồ kể từ vị trí củalớp trưởng A

(GV yêu cầu HS tổng qt hóa tốn với n HS lớp Nếu lớp có 2kHS B có hội thắng ko?)

C BÀI TẬP ÁP DỤNG

Câu Cho p số nguyên tố lớn chứng minh tồn số có dạng 111 11 mà chia hết cho p

Câu 2.Trong hình vng đơn vị (cạnh 1)có 101 điểm Chứng minh có điểm chọn phủ hình trịn bán kính .

Câu 3.Trong hình chữ nhật 3x4 đặt sáu điểm chứng minh số ln tìm hai điểm có khoảng cách chúng không lớn

1

5

(146)

Câu 4.Trong tam giác đều, cạnh có độ dài 1, đặt năm điểm Chứng minh tồn hai điểm mà khoảng cách chúng nhỏ

Câu 5.Trong hình trịn đường kính có 10 điểm Chứng minh tồn hai điểm mà khoảng cách chúng bé

Câu 6.Tìm hình vng có kích thước bé nhất, để hình vng xếp năm hình trịn có bán kính 1, cho khơng có hai hình trịn chúng có điểm chung

Câu 7.Cho 2014 đường thẳng có tính chất: chia hình vng thành hai tứ giác có tỉ số diện tích Chứng minh có 504 đường thẳng 2014 đường thẳng đồng quy

Câu 8.Cho bảng kích thước 2n x 2n vng Người ta đánh dấuvào 3n ô bảng Chứng minh chọn n hàng n cột bảng cho ô đánh dấu nằm n hàng n cột

Câu 9.Cho 1000 điểm M1, M2 …M1000trên mặt phẳng Vẽ mơt đường trịn bán kính tùy ý Chứng minh tồn điểm S đường tròn cho SM 1+ SM2+ … +SM1000≥ 1000

Câu 10.Cho điểm mặt phẳng tô hai màu xanh, đỏ Chứng minh tồn tam giác mà ba đỉnh trọng tâm màu

Câu 11.Một hình lập phương có cạnh 15 chứa 11000 điểm Chứng minh có hình cầu bán kính chứa sáu điểm số 11000 điểm cho

Câu 12.Trong hình vng cạnh 12 chứa 2014 điểm Chứng minh rằngluôn tồn tam giác cạnh 11 phủ kín 504 điểm 2014 điểm cho

Câu 13.Cho ( ), i = 1, 2, 3, 4, 5, 6, 7, 8,9 tập hợp gồm điểm khác có tọa độ nguyên không gian

Chứng minh trung điểm đường nối cặp điểm có tọa độ nguyên

Câu 14.Trong hình vng có cạnh chứa số đường tròn Tổng tất chu vi chúng 10 Chứng minh tồn đường thẳng cắt đường trịn đường trịn đó?

1

2

, , i i i

x x x

(147)

Câu 15.Cho hình vng 13 đường thẳng, đường thẳng chia hình vng thành hai tứgiác có tỉ sốdiện tích : 3.Chứng minh số13 đường thẳng cho, có đường thẳng qua điểm

Câu 16.Trong bát hình vng cạnh 18 cm có 128 hạt vừng Chứng minh tồn hai hạt vừng có khoảng cách tới nhỏhơn 20 cm

Câu 17.Trong hình chữ nhật 3x4 đặt điểm Chứng minh số ln tìm hai điểm có khoảng cách chúng không lớn

Câu 18.Bên tam giác ABC cạnh đặt điểm Chứng minh tồn điểm có khoảng cách nhỏ 0,5

Câu 19.Trong hình trịn đường kính có 10 điểm Chứng minh tồn hai điểm mà khoảng cách chúng bé

Câu 20.Trên mặt phẳng cho 25 điểm Biết ba điểm số luôn tồn hai điểm cách nhỏ Chứng minh tồn hình trịn bán kính chứa khơng 13 điểm cho

Câu 21.Tìm hình vng có kích thước bé nhất, để hình vng xếp năm hình trịn bán kính 1, cho khơng có hai hình trịn chúng có điểm chung

Câu 22.Chứng minh hình trịn bán kính 1, khơng thể chọn điểm mà khoảng cách hai điểm tùy ý chúng lớn

Câu 23. Cho hình trịn có bán kính n, n số ngun dương Trong hình trịn có 4n đoạn thẳng có độ dài bẳng Cho trước đường thẳng d Chứng minh tồn đường thẳng d’ song song với d, vng góc với d cho d’ cắt hai đoạn thẳng cho

Câu 24.Cho 1000 điểm M M1, 2, ,M1000trên mặt phẳng Vẽ đường trịn bán kính

tùy ý Chứng minh tồn điểm S đường trònsao cho

1 1000 1000

SM +SM + +SM

(148)

Câu 25.Cho chín đường trịn có tính chất mối đường thẳng chia hình vng thành hai tứ giác có tỉ số diện tích 2

3 Chứng minh có ba đường thẳng số qua điểm

Câu 26.Cho bảng có kích thước 2n×2n ô vuông Người ta đánh dấu vào 3n ô

của bảng Chứng minh chọn n hàng n cột bảng cho ô đánh dấu nằm n hàng n cột

Câu 27.Trong mặt phẳng cho tập hợn A có n điểm ( n≥ 2) Một số cặp điểm nối với đoạn thẳng Chứng minh tập hợp A cho, có hai điểm đươc nối với số lượng điểm khác thuộc A

Chương VI

PHẦN NGUYÊN PHẦN LẺ VÀ ỨNG DỤNG

C. KiÕn thøc cÇn nhí

PHẦN NGUN

1 Định nghĩa

Phần nguyên có sốthực a số ngun lớn khơng vượt q a, kí hiệu [ ]a Ta có

[ ]a ≤ <a [ ]a +1

Phần lẻcủa sốthực a hiệu a với phần ngun nó, kí hiệu { }a

Ta có { }a = −a [ ]a , 0≤{ }a ≤1

Ví dụ.

[ ]

[ ]

[ ]

{ }

{ } ( )

{ }

5, 5;

5, 6;

2012 2012;

5, 5, 0, 3;

5, 5, 0, 7;

2012 2012 2012

= − = −

=

= − = − = − − − =

= − =

(149)

2 Tính chất

1) a∈ ⇔ [ ]a =a { }a =0

2) n∈ n≤ < + ∈ ⇔a n  [ ]a =n

3) { }a ={ }[ ]a =0

4) Nếu n∈ [n+a]= +n [ ] [a ; n+a]={ }a

5) Nếu [n+a]=n n∈ 0≤ ≤a

6) a≥ ⇒b [ ] [ ]ab

7) [ ] [ ] [a + ba b+ ]

Tổng quát [ ] [ ]a1 + a2 + + [ ] [ana1+a2+ + an],

8) [ ] [ ] [aba b− ]

9) { } { } {a + ba b+ } { } { } {; aba b− }

10) Nếu [ ] [ ]a = b a b− <1

11) [ ] [ ]2

a +a+ = a

 

12) Nếu n∈* [ ] [ ] [ ]na n a ; a a

n n

    ≥    =    

13) Nếu a sốnguyên [ ] [ ]− = −a a ;

Nếu a khơng sốngun [ ] [ ]− = −a a −1;

Chứng minh

Các tính chất 1) đến 5) chứng minh dễ dàng dựa vào định nghĩa phần nguyên

6) Vì ab nên tồn số c≥0 cho a= +b c Do a=[ ]b +{ }b +c, suy

[ ] [ ]a = b +{ }b +c Mà { }b +c≥0 nên [ ] [ ]ab

7) Viết a=[ ]a +{ }a b, =[ ]b +{ }b Khi

[ ] { } [ ] { } [ ] [ ] { } { }

[a b+ =]  a + a + b + b = a + b + a + b 

Mà { } { }a + b ≥0 nên

[a b+ ≥] [ ] [ ]a + b

8) Áp dụng tính chất ta có

[a b+ +] [ ] [ba b b− + =] [ ]a nên [ ] [ ] [aba b− ]

9) { } { }ab = −a [ ]a + −b [ ]b =(a b+ −) ([ ] [ ]a + b )≥ + − + =a b [a b] {a b+ } Vậy { } { } {aba b− }

10) [ ] [ ]a = b suy a−{ }a = −b { }b Khơnggiảm tính tổng quát, giả sử ab

Nếu a=b a b− =0;

Nếu a>b từ a b− ={ } { } {aba b− }

(150)

Suy a b− = − ≤a b {a b− }<1

Vậy ln có 0≤ − ≤a b

11) Đặt { }a =d 0≤ ≤d

• Nếu

d

≤ < [ ] [ ] [ ];

2 2

a a d a d a

 +  = + + = + + =

     

     

[ ]2a =2([ ]a +d)=2[ ] [ ] [ ]a + 2d =2 a Từđó suy điều phải chứng minh • Nếu 1

2≤ <d [ ] [ ] [ ]

1 1

1;

2 2

a a d a d a

 +  = + + = + + = +

     

     

[ ]2a =2([ ]a +d)=2[ ] [ ] [ ]a + 2d =2 a +1 Suy điều phải chứng minh

12) Ta có [ ]na =n a([ ]+{ }a )=n a[ ]+ n a{ }, mà n a{ }≥0 nên [ ] [ ]nan a

[ ] { } [ ]

a a a

a

n n n n

   

  = +  =   

     

13) Nếu a sốnguyên [ ]− = − = −a a [ ]a

Nếu a khơng ngun 0<{ }a <1, nên − < −1 { }a <0, suy −{ }a = −1

Ta có [ ]− = −a  ([ ]a +{ }a ) = − [ ]a + − { }a = −[ ]a +1 B. CÁC VÍ DỤ MINH HỌA

Dạng 1: Chứng minh số dạng toán số học

Bài toán Cho a>0 số n nguyên dương Chứng minh số số nguyên dương

là bội sốcủa n không vượt a a

n

     

Hướng dẫn giải

Ta viết a=nq r+ , q sốtựnhiên, 0≤ <r n

Rõ ràng bội sốcủa n không vượt a n, , ,n qn tổng cộng có q số

Mặt khác a q

n

  =  

  Từđó suy kết luận tốn

Bài tốn Số 2012! có tận số0?

Hướng dẫn giải

Vì 10=2.5 nên để biết 2012! có tận bao nhêu chữ số 0, ta cần phải tính

sốmũ phân tích 2012! thừa sốnguyên tố

Theo Ví dụ 1, Sốmũ phân tích 2012!ra thừa sốnguyên tốbằng

2

2012 2012 2012 2012

402 80 16 501

5 5

  +  +  + = + + + =

       

        (Do

5

2012<5 )

(151)

Do mũ phân tích 2012!ra thừa sốnguyên tốnhiều 501

Vậy 2012! Có tận 501chữ số

Nhận xét. Nếu

5k ≤ <n 5k+ sốchữ số0 tận vềbên phải số n!bằng

2

5 5k

n n n

   + + + 

     

     

Bài tốn Tìm sốtựnhiên k lớn cho (2011!)2012 chia hết cho 2012 k

Hướng dẫn giải

Ta có

2012=2 503

Sốmũ cao 503 có 2011! Là

2011 503

  =

 

  (do

2

2011 503< ) Vậy 2011! chia hết cho

503 không chia hết cho 5034, hiển nhiên 2011! chia hết

cho

4 Do 2011! chia hết cho 20123 không chia hết cho 2012

Muốn ( )2012

2011! chia hết cho 2012k k≤3.2012=6036.

Vậy maxk =6036

Bài tốn Tìm sốtựnhiên n cho

2010 2011 2012

n n n

  =  = 

     

      (1)

Hướng dẫn giải

Viết n=2010k+r(0≤ ≤r 2009, ,k r có sốtựnhiên) Thay vào (1) ta có

2010 2011 2012

2010 2011 2012

k+r k+ −r k k+ −r k

  =  = 

     

     

2

0

2011 2012 2011 2012

r k r k r k r k

k k  −  k  −   −   − 

⇔ = + = + ⇔  = =

       

Suy 0≤ −r 2k nên 2k≤ ≤r 2009, 0≤ ≤k 1004 Vậy n=2010k+r(0≤ ≤k 1004; 2k≤ ≤r 2009)

Do có 105 giá trị k (từ đến 1004) Với k r nhận giá trịtừ 2k đến

2009 Vậy sô nghiệm tựnhiên n (1)

( )

1004

2010 1011030

k

k

=

− =

Bài tốn Tìm tất cảcác sốnguyên tố x cho

(152)

2

1  x 1

  + + + −

      sốnguyên tố

Hướng dẫn giải

Nhận xét

( )2

2

1 1 ,

n nnn n

  = + = = + − = ∈

 

      

Đặt 2 ( )2 ( )

1 1 2

n

S = n  + n + + +  n+ − = n+ n= n +n

     

Do ( )

2

1

4

1

6 x

x x x

y=    + + + x − = +S S + +S − = − −

Nên ( )

6y=x 4x −3x−1 , suy 6y x , mà x y, sốnguyên tốsuy x∈{2;3;y}

Nếu x=2 y=3 (thỏa mãn); x=3 y=13 (thỏa mãn); x= y y= −1

4

y= (loại)

Vậy tốn có hai nghiệm x=2 x=3

Giải phương trình có chứa dấu phần ngun a) Dạng f x( )=a a( ∈)

Phương pháp: f x( )=a a( ∈)⇔ ≤a f x( )< +a Ví dụ Giải phương trình [ ]2 [ ]

3

xx + =

Ví dụ Giải phương trình 2 2

5 26

x x

 +  −  + = −

    (gợi ý: x2+7  = x2+5+2) b) Dạng f x( )=g x( )

Phương pháp: Đặt g x( )=t (t nguyên), biểu diễn f x( )=h t( ) đưa phương trình

( ) ( )

h t = ⇔ ≤t t h t < +t

 

  hay 0≤h t( )− <t

Tìm t, sau từ g x( )=t tìm x

Bài tốn Giải phương trình 5

5 x x − −   =    

Hướng dẫn giải

Đặt 5 ( )

7

x

t t

− = ∈

 3; 21

5 25

t x t

x= + − = −

Ta có 21 21

25 25

t t

t t t

− −

  = ⇔ ≤ < +

 

 

20

25 21 25 25

46 46

t t tt

⇔ ≤ − ≤ + ⇔ < ≤

Do t nguyên nên t=0 Suy x=1

Vậy phương trình có nghiệm x=1

(153)

Bài tốn Giải phương trình [ ]

9

xx + =

Hướng dẫn giải

Biến đổi phương trình vềdạng [ ]

x x = +

Đặt ( *)

x

t t

+ = ∈

x= 9t−8 (do x>0) Ta có

2

9

1

7 13

9

2

7

7 13

t t t t t

t t

t t t

t t

t

 − = ⇔ ≤ − < +

  ≤ ≤   −   − + ≤   ≤ ⇔ ⇔ − + ≥   +  ≥  

Do t sốtựnhiên nên t∈{1; 6; 7;8 } Do x∈{1; 46; 55;8 } Vật tập nghiệm phương trình {1; 46; 55;8 }

Bài tốn Giải phương trình

3

xx+ x

  + =

   

   

Hướng dẫn giải

Áp dụng tính chất 11) [ ] [ ]2 ,

a +a+ = a

  ta có

2 2 1

3 3

xx+ xxx

  +  =  + +  = 

         

         

Nên phương trình cho trởthành

4

3

xx

  =

 

 

Đặt ( )

3

x

t t

= ∈ 4;

5

t x t

x= + − = + Suy

{ }

4

0 2; 1; 0;1;

5

t t

t t t t

+ +

  = ⇔ ≤ − < ⇔ − < ≤ ⇔ ∈ − −

 

 

(do t nguyên), tương ứng tìm 7; ; ; ; 5 5

x∈ − 

 

c) Dạng f x( )  = g x( )

Phương pháp:Đặt f x( )  = g x( )=t suy f x( )−g x( ) <1, dẫn đến a< <x b

(154)

Với a< <x b suy ( )

( )

1

2

,

a f x b

a f x b

< < 

< <

 từđó tìm t

Ứng với giá trịcủa t nguyên, giải hệ ( )

( )

f x t

g x t

 =    =     

đểtìm x

Tập hợp giá trị x tìm từ hệtrên sẽlà nghiệm phương trình

Bài tốn Giải phương trình 1

3

xx+

  = 

   

   

Hướng dẫn giải

Đặt 1 ( )

3 x x t t − +   = = ∈    

     Theo tính chất 10) ta có

2 1

1 1 11

3

x x x

x

− + −

− < ⇔ − < < ⇔ − < < Khi

1

1 0 5

0

2

2

1

3 x x x x   +  +

 < < ≤ ≤   

 ⇒  

 − 

−   − < < − ≤ ≤

 

 

   

Suy t∈{0;1; 2;3; 4;5}

Với t=0

2

1

0

2

2 1

0

1

3 2

1 1 x x x x x x x

 ≤ < 

 ≤ <

− +  

  = = ⇔ ⇔ ⇔ ≤ <

 

    +

     ≤ < − ≤ <

 

Với t =1thì

2

7

1

2

2 1 3

1 2

1 3 2 x x x x x x x

 ≤ < 

 ≤ <

− +  

  = = ⇔ ⇔ ⇔ ≤ <

 

    +

     ≤ <  ≤ <

 

Với t=2

2

7

2

5

2 1

2

1

3 2

3 x x x x x x x

 ≤ < 

 ≤ <

− +  

  = = ⇔ ⇔ ⇔ ≤ <

 

    +

     ≤ <  ≤ <

 

Với t=3

2

11

3

5

2 1 11

3

1

3 2

5 x x x x x x x

 ≤ < 

 ≤ <

− +  

  = = ⇔ ⇔ ⇔ ≤ <

 

    +

     ≤ <  ≤ <

 

Với t=4

2

13

4

8

2 1

4

1 x x x x x x x

 ≤ < 

 ≤ <

− +  

  = = ⇔ ⇔ ⇔ ≤ <

 

    +

     ≤ <  ≤ <

 

(155)

Với t=5

2

19

5

8

2 1 3 19

5

1

3 2

9 11 x x x x x x x

 ≤ < 

 ≤ <

− +  

  = = ⇔ ⇔ ⇔ ≤ <

 

    +

     ≤ <  ≤ <

 

Vậy tập nghiệm phương trình [0, 5;1)∪[2;3)∪[3, 5;5, 5] [∪ 7;8)∪[9;9, )

d) Dạng Phương trình chứa nhiều dấu phần nguyên

Phương pháp:Sửdụng tính chất phần nguyên, phân tích đa thức thành nhân tử, đặt ẩn phụ(nếu cần) đểdưa vềcác dạng 1, 2,

Bài toán Giải phương trình [ ] [ ] [ ]x + 2x + 3x + + [2009x]=4036082

Hướng dẫn giải

Nhận xét

[ ]x ≤ < +x [x 1] suy k x[ ]≤kx<k x[ ]+k nên k x[ ] [ ] [ ]≤ kxk x + −k 1(kZ+)

Do thay k =1, 2, , 2009 cộng theo vếta có

[ ] [ ] [ ] [ ] [ ]

[ ] [ ]

2019045 2009 2019045 2017036

2019045 4036082 2019045 2017036

x x x x x

x x

≤ + + + ≤ +

≤ ≤ +

Lại có 4036082=2019045 2017037.+ Do phương trình vơ nghiệm

Bài tốn Giải phương trình 2

x x x −      − = −        

Hướng dẫn giải

Áp dụng tính chất 13) ta có

2 2 2 , 1, x x x x x   − ∈    − =    − − ∉      

• Nếu

x sốnguyên phương trình cho trởthành

2 1

0

3

x x

x

− −

  = ⇔ ≤ < ⇔ ≤ <

 

 

x sốnguyên nên x∈{1; 2; } • Nếu

x khơng sốngun phương trình cho trởthành

2 1

1 1

3

x x

x

− −

  = − ⇔ ≤ + < ⇔ − ≤ <

 

 

x không nguyên nên phải loại 1, ( 1; 0) 0;1

x= − x= ⇒ ∈ −x ∪  

 

Vậy tập nghiệm phương trình ( 1; 0) 0;1 {1; 2; }

 

− ∪ ∪

 

3.BẤT PHƯƠNG TRÌNH CĨ CHỨA DẤU PHẦN NGUN

(156)

Khi giải bất phương trình có chứa dấu phần nguyên, ta thường đặt biểu thức ( )

f x =t

 

  (t nguyên) để chuyển giải bất phương trình khơng cịn chứa dấu

phần nguyên, vận dụng định nghĩa tính chất phần ngun để tìm nghiệm bất phương trình

Bài tốn Giải bất phương trình [x+ >2]

Hướng dẫn giải

Cách 1. Nhận xét [ ]a >b (b nguyên) a≥ +b

Ta có [x+ >2] x+ ≥2 Do x≥4

Cách 2.Đặt [x+ =2] t (t sốngun) có t>5 Do t∈{6; 7;8; }

Từ [x+ =2] t suy t≤ + < +x t suy t− ≤ < −2 x t 1,t∈{6; 7;8; }

Vậy x≥4 Bất phương trình có vơ sốnghiệm x≥4

Bài tốn Giải bất phương trình [ ]2 [ ]

2 x −9 x+ +1 16<0

Hướng dẫn giải

Áp dụng tính chất 4) ta có [x+ =1] [ ]x +1 Biến đổi bất phương trình thành

[ ]2 [ ]

2 x −9 x + <7

Đặt [ ]x =t (t số nguyên) có 2t2− + <9t suy 1< <t 3, mà t nguyên nên

{ }2;3

t

Với t=2 [ ]x =2 suy 2≤ <x

Với t=3 [ ]x =3 suy 3≤ <x

Vậy tập nghiệm bất phương trình [2; )

Bài tốn Giải bất phương trình [ ] [ ]2x > x

Hướng dẫn giải

Cách Đặt [ ]x =t (t số nguyên) t≤ < +x t suy 2t≤2x< +2t Do

[ ]2x =2t 2t+1

• Với [ ]2x =2t 0≤{ }x <0, 2t> ⇔ >t t 0, mà t nguyên nên t

sốnguyên dương Dẫn đến x≥1

• Với [ ]2x = +2t 0, 5≤{ }x <1và 2t+ > ⇔ > −1 t t 1, mà t nguyên nên t sốnguyên dương Dẫn đến x≥0

Kết hợp với 0, 5≤{ }x <1dẫn đến x≥0,

(157)

Cách 2. Nhận xét [ ] [ ]a > b a>b [ ] [ ]ab

Ta có [ ] [ ]2x > x ⇔2x>x [ ] [ ]2xx ⇔ >x [ ] [ ]2xx

Trước hết ta tìm x cho [ ] [ ]2x = x

Đặt [ ] [ ]2x = x =t (t nguyên) ta có

2x− < ⇔x x <1 suy 0< <x nên [ ]x =0

Với t=0 [ ] [ ]x = 2x =0 suy 0≤2x<1 nên 0≤ <x 0, Vậy nghiệm bất phương trình x≥0,

C BÀI TẬP ÁP DỤNG Bài 1: Tìm [ ]x biết: x -

3

1 〈- 2 〈 x Bài 2: Tìm [ ]x biết : x 〈- 〈x + 0,5

Bài 3: Tìm [ ]x biết: x =

2

1 +

1 +

1 + +

) (

1

+

n

n

Bài 4: Tìm [ ]x biết: x = +

2

+

3

+

4

+ +

6

10

Bài 5: Tìm [ ]x biết:

x = 2+ 2+ + 2+ , (với n dấu căn) Bài 6: : Tìm [ ]x biết:

x = 4+ 4+ + 4+ x = 6+ 6+ + 6+

x = 2008+ 2008+ + 2008+ 2008

x = 6+ 6+ + + 3

6

6+ + +

x = +

2

+

3

+ + n+1 +1

n n

Bài 7: Tìm [ ]x biết:

x= 4n2 + 16n2 +8n+3 , (với n dấu căn)

Bài 8: Tính tổng sau: S = [ ]1 + [ ]2 + [ ]3 + +[ n2 −1]

Bài 9: Tính tổng sau:

S = [ ]1 + [ ]2 + [ ]3 + +[ ]24 Bài 10: Tìm số nguyên tố: x,y thoả mãn: [ ]1 + [ ]2 + [ ]3 + +[ x2 −1]= y

Bài 11: Chứng minh rằng:

[ ]1 + [ ]2 +[ ]3 + .+ [ 20092 −1]

Chia hết cho: 1004 2009

(158)

Bài 12: Tính tổng sau:

S = [ 1.2.3.4]+ [ 2.3.4.5]+ [ 3.4.5.6]+ + [ n(n+1)(n+2)(n+3)]

Bài 13: Tính tổng sau: S =  + 

m b a , +     + m b a , +     + m b a ,

2 + +

    − + m b a m 1), ( Bài 14: Chứng minh rằng, với số nguyên n ta có: [n+x]= n + [ ]x

Bài 15: Chứng minh rằng, với x,y ta có: [ ]x + [ ]y ≤ [ü+y] ≤ [ ]x + [ ]y +

Bài 16: Cho n số nguyên dương, chứng minh:   n + +  n = n

Bài 17: Cho n số tự nhiên, chứng minh: [ 4n+1]= [ 4n+2]

Bài 18: Cho n số tự nhiên, chứng minh: [ n + n+1]= [ 4n+2]

Bài 19: Cho n số tự nhiên, chứng minh [ 63n+ 63n+1]= [ 252n+2]

Bài 20: Chứng minh với số tự nhiên n, ta có:

a, [ 9n+ 9n+1]= [ 9n+2] b, [ 7n+ 7n+6]= [ 36n+2] Bài 21: Chứng minh rằng: [ ]x +

    +

x = [ ]2x , (với x số thực bất kỳ)

Bài 22: Chứng minh rằng: [ ]x +  + 

n

x +  +  n

x + +  + −  n n

x = [ ]nx

Bài 23: Tính tổng: S =  + 

2 n +     + 2 n +     + n +     +

n +

Bài24: Tính tổng sau: S =  + 

12 n +     + 24 12 n +     + 48 24

n +

Bài 25: Chứng minh rằng: m[ ]x ≤ [ ]mx ≤ m[ ]x + m - (với giá trị m nguyên

dương)

Bài 26: Chứng minh : Không tồn x thoả mãn: [ ]x + [ ]2x + [ ]3x + + [100x] = 313096

Bài 27: Chứng minh rằng, không tồn x thoả mãn: [ ]x + [ ]2x + [ ]4x + [ ]8x + [ ]16x + [ ]32x = 12345

Bài 28: Chứng minh , không tồn x thoả mãn: [ ]x + [ ]2x + [ ]4x + +[ ]nx =

2 ) (n+

n k - 1, (với: n 〉1; k Z

∈ ) Bài 29: Giải phương trình:[x+0,7] = -

Bài 30:Giải phương trình: [x+1]+ [x+2]+ [x+3]=

Bài 31: Giải phương trình 4[ ]x =3x

(159)

Bài 32: Giải phương trình:  + 

8

5 x =

5 15x

Bài 33: Giải phương trình:  − 

3 2x +

    +

4x =

3 5x

Bài 34: Giải phương trình: [ ]x { }x = x -

Bài 35: Giải phương trình: x - 3 

2

x = 2

Bài 36: Giải phương trình: [x−1] =

    +1

2

x

Bài 37: Giải phương trình: x4= 2x2 + [ ]

x

Bài 38: Giải phương trình: x3 - [ ]

x =

Bài 39: Giải phương trình: [−x2 +3x]=

    + 2 x

Bài 40: Chứng minh: [ n]

)

( + số lẻ, (với n ∈N ) Bài 41: Tìm hai chữ số tận của: [ 2000]

) 21 29

( +

Bài 42: Tìm chữ số đứng trước sau dấu phẩy biểu diễn thập phân ( + 3)1992

Bài 43: Chứnh minh rằng: { n}

) 26

( + Có n chữ số giống sau dấu phẩy Bài 44: Với p số nguyên tố lớn

Chứng minh: [ p]

)

( + - 2p+1 chia hết cho p Bài 45: Tìm số mũ dạng PTTC của: [ n]

) ( + Bài 46: Cho dãy số: x0; x1; x2; x3; .xn;

Bài 47: Chứng minh rằng: Số (5+ 26)101 viết hệ thập phân có 100 chữ số

đứng sau dấu phẩy

Bài 48: Chứng minh rằng: tồn số tự nhiên n : (2+ 2)n- [ n]

) 2

( + Lớn hơn: 0,999999

Bài 49: Tìm số nguyên lớn khơng vượt q: (4+ 15)7

Bài 50: Tìm tất số nguyên tố biểu diễn dạng: 

     2 n ;       n ;

Bài 51: Trong biểu diễn thập phân số sau: a 100! có tận chữ số ? b ((3!)!)! có tận chữ số ? Bài 52: Chứng minh rằng:

a )! !.( )! ( + n n

n Là số tự nhiên

b )! !.( ! )! ( )! ( m n m n m n

+ Là số tự nhiên

c Với n nguyên dương, Hỏi n! có chia hết cho 2n không ?

(160)

d Số: C500

1000 Có chia hết cho không ?

Bài 53: Cho A =

) (

) (

n n

Với n số tự nguyên dương Chứng minh rằng: Trong dãy số: A; 2A; 22A; 23A;

Đén vị trí trở số hạng số nguyên

HƯỚNG DẪN GIẢI

Chương I

QUAN HỆ CHIA HẾT TRONG TẬP HỢP SỐ

Câu Ta có:

( ) ( )( ) ( )( ) ( )

( )( )( )( ) ( )( )

2

5 2

a a a a a a a a a a a a a a a a 5a a a

 

− = − = − + = + −  − + 

 

= + − − + + + −

Do tích sốngun liên tiếp chia hết cho sốngun liên tiếp ln có ba sốnguyên liên tiếp mà tích chúng chia hết cho ( )6,5 =1

Suy a a a a a 30( + )( − )( − )( + ) 5a a a 30.( + )( − )

Vậy a a 305−  Câu a) Ta có:

( ) ( )

( )( ) ( )

3

3

A n 3n 3n 2n n n n n n n

= + + + + + = + + + =

= + + + +

Khi đó: n 3( + ) ; n n n 2( + )( + )là tích sốnguyên dương liên tiếp nên chia hết cho A 3⇒ 

b) Ta có:

a 13k 2,b 13n 3= + = +

( ) (2 )2 ( )

2 2

a +b = 13k 2+ + 13n 3+ = 13 13k 4k 13n 4n 13= + + + + 

Câu Ta có: A=n n 73( 2− )2−36n

 

 

(161)

( ) ( ) ( )( )

( )( ) ( ) ( ) ( ) ( )

( )( )( )( ) ( )( )( )( )( )( )

2 3

3 2

2

n n n n n n n 7n n 7n

n n n 6n n n 6n n n n n n n n n n n n n n n n n n n n n

   

=  − −   − + = − − − +

   

= − − − − − + =  − − +   − − − 

= + − − − + − = + + − − − +

Do Alà tích 7sốngun liên tiếp ⇒A n ∀ ∈

Câu Ta có: n 2k,= với k sốnguyên; n 28n3 − =( )2k 3−28 2k( )=8k 56k3−

( ) ( )

( ) ( )( )

2

2

8k k 8k k

8k k 48k 8k k k 48k

= − = − −

= − − = − + −

( )( )

k k k 1− + tích ba sốnguyên liên tiếp nên chia hết

( )( )

8k k k 48k− + − chia hết cho 48 Câu Ta có: n n n n n 13− = ( − )( + )

Vì n 1; n; n 1− + ba sốtựnhiên liên tiếp nên có ba sốđó chia hết cho Do đón lẻnên n có dạng n 2k k N= + ( ∈ )

Ta có: n n n n n 13− = ( − )( + =) (2k 2k 2k 2+ ) ( + )=4.k k 2k 1( + )( + )

Vì k (k + 1) sốtựnhiên liên tiếp suy ra: k k 2( + ) ⇒4k k 2k 8( + )( + ) ⇒(n n 83− ) Vì hai sốnguyên tốcùng nên kết hợp với ( ) ( )1 ; suy

(n n 243− ) (dpcm)

Câu Ta có:

n 17n n n 18n n n n 18n3+ = 3− + = ( − )( + +)

Vì n n n 1( − )( + )là tích ba số nguyên liên tiếp nên chia hết cho 3, ( )2,3 1= nên chia hết cho

18n 6 , suy điều phải chứng minh Câu Ta có:

( ) (3 )3

3

Q n= + n 1+ + n 2+

( ) ( )

( )

3 3

3

n n 3n 3n n 6n 12n

3 n 3n 5n

= + + + + + + + +

= + + +

Đặt C n 3n 5n n n 2n 2n 3n 3= 3+ 2+ + = 3+ 2+ + + +

( ) ( ) ( )

( )( ) ( )

2

n n 2n n n n n n n

= + + + + +

= + + + +

Ta thấy n n n 2( + )( + )chia hết cho 3( tích sốtựnhiên liên tiếp)

(162)

Và n 3( + ) ⇒Cchia hết cho Nên Q 3C= chia hết cho

Câu Ta có: 20092008+20112010 =(20092008+ +1) (20112010−1) Vì 20092008+ =1 (2009 2009+ )( 2007+ 2010) (1)

( )( )

2010 2009

2011 − =1 2011 2011− + 2010(2)

Từ(1) (2) ta có dpcm Câu

a) Ta có: 85+211 =( )23 5+211=215+211 =2 111 ( 4+ =) 2 1711 chia hết cho 17

b) Ta có: 1919 +6919 =(19 69 19+ )( 18−19 ,69 6917 + + 18)=88 19( 18 −19 ,69 6917 + + 18) chia hết cho 44

Câu 10

Nếu n 3k k= ( ∈) A 9k= 2+3k 2+ khơng chia hết cho 3. Nếu n 3k k= + ( ∈) A 9k= +9k 4+ không chia hết cho 3. Nếu n 3k k= + ( ∈) A 9k 15k 8= 2+ + không chia hết cho 3. Do Akhơng chia hết cho với sốngun n

Vậy Akhông chia hết cho 15 với sốnguyên n Câu 11 Ta có : n 6n 11n4+ 3+ 2+30n 24−

=(n4+6n 11n3+ 2+6n)+(24n 24− )=n n 6n 11n 24 n 1( 3+ 2+ + )+ ( − )

=n n n( 3+ 2) (+ 5n 5n2+ )+(6n 6+ )+24 n n n n 5n 24 n 1( − =) ( + )( 2+ + +) ( − )

 

= n n n n 24 n 1( + )( + )( + +) ( − )

Vì n; n + 1; n + 2; n + 3; bốn sốtựnhiên liên tiếp nên tích chúng chia hết cho Mặt khác sốtựnhiên liên tiếp tồn sốchẵn liên tiếp nên có sốchia hết cho 2, sốchia hết cho Vậy n n( +1)(n+2)(n+3) chia hết 2.3.4 = 24 24 (n - 1)

chia hết cho 24 nên

6 11 30 24

n + n + n + n− chia hết cho 24

Câu 12 Ta có: 2

2a +3ab+2b 7

( ) 7

2 7 ) (

2 a2 +b2 − ab + ab ⇒ ab + ab

Do ( ) ( )2 ( )

7ab7 a b, ∈Z ⇒2 a b− 7do 2, =1 Từđó ta có 2

( )( )

− = − + 

a b a b a b

Vậy 2

(ab ) 7

(163)

Câu 13 n sốngun khơng chia hết n=3k+1 n=3k+2(kZ)

- Xét n=3k+1 ta có: 32n =32 3( k+1) =3 36k =( )33 2k.9=27 92k ≡9 mod 13( ) 3 ( )3

3n =3k+ =3 3k = k.3=27 3k ≡3

Suy ra: ( )

3 n 3n 13 mod 13

P= + + ≡ + + = ≡

- Xét n=3k+2 ta có: 2 3( 2) ( )3 2 ( )

3 n =3 k+ =3 3k = k.81=27 81 81 mod 13k ≡ ≡

3 ( )3

3n =3k+ =3 3k = k.9=27 9k ≡9

Suy ra: ( )

3 n 3n 13 mod 13

P= + + ≡ + + = ≡

Vậy , với n sốnguyên không chia hết cho P=32n+ +3n chia hết cho 13

Câu 14 Xétx sốnguyên dương, ta thấy

( )( )( ) ( )

5

x x x x x x 1− = − + +  (vì chứa tích ba sốngun liên tiếp) Với x 5q q= ( ∈+) x5−x 5

Với x 5q q= ± ( ∈+) x5−x 5 Với x 5q q= ± ( ∈+) x5−x 5 Suy x x 25−  ( ) mà ( )5,6 =1 3( ) Từ ( ) ( ) ( )1 , , suy x5−x 30

Xét hiệu ( ) ( ) ( ) ( )

1 2 3 2019 2019

Q P− = a a− + a a− + a a− + + a −a Vì x5−x 30 nên Q P 30− 

Mà theo P 30 nên Q 30

Câu 15. Vì a sốtựnhiên chẵnnên a = 2k (k ∈N) Do M = 8k3 +4k2 +2k k= +k2 +k

24 12

= 2k3+3k2+k= k k 2k 1( + )( + )

6

Ta có : k k+1 2( ) ⇒k k+1 2k+1 2( )( )

Ta cần chứng minh k k+1 2k+1 3( )( )

+ Nếu k = 3n (với n ∈N) k k+1 2k+1 3( )( ) + Nếu k = 3n + (với n ∈N) 2k+1 3

(164)

+ Nếu k = 3n + (với n ∈N) k 3+ 

Như ∀ k ∈ N ta có k k 2k 1( + )( + )luôn chia hết cho cho Mà (2, 3) = ⇒

( )( )

k k+1 2k+1 Vậy A có giá trịnguyên

Câu 16. Với n = ta có A(0) = 19  19

Giả sửA chia hết cho 19 với n = k nghĩa là: A k( )=7.52k+12.6 19k

Ta phải chứng minh A chia hết cho 19 với n = k + nghĩa phải chứng minh:

( ) k 1( ) k A k 7.5+ = + +12.6 + 19

Ta có: A k 7.5( + =) k 1( )+ +12.6k 1+ 19

( )

2k n

2k 2k n

2k

7.5 12.6

7.5 7.5 19 12.6 8.A k 7.5 19 19

+

= + +

= + 

Vậy theo nguyên lý quy nạp A = 7.52n+12.6nchia hết cho 19 với sốtựnhiên n Câu 17 Ta có:

( ) ( )

+ + + + = + +

= − + = + −

n n 2n n n 2n

n n n n n

5 26.5 25.5 26.5 8.8 59 8.64 59.5 64

n

59.5 59 8 64( n −5n)(64 5− )=59 Vậy 5n 2+ +26.5 8n+ 2n 1+ 59

Câu 18 Dễthấy a a a a a 13− = ( − )( + )là tích sốtựnhiên liên tiếp nên chia hết cho 3 Xét hiệu:

( ) ( ) ( )

( ) ( ) ( )

3 3

1 2016 2016 2016

3 3

1 2 2016 2016

A a a a a a a a a a a a a a a a

− + + + = + + + − + + +

= − + − + + −

Các hiệu chia hết cho , A chia hết cho

Câu 19 a) Gọi sốphải tìm avà b, ta có a b+ chia hết cho Ta có: a3 +b3 =(a b a ab b+ )( 2− + 2)=(a b a b+ ) ( + )2−3ab

 

 

Vì a b+ chia hết (a b+ )2−3abchia hết cho Do vậy, (a b a b+ ) ( + )2−3ab

 chia hết cho

b) n n 15+ ( 3+ ⇔) (n5+n2−n n 12 + ) ( 3+ )

(165)

( ) ( ) ( )

( )( ) ( )( )

( )

2 3

2

2

n n n n

n n n n n n n n

n n n n

⇔ + − − +

⇔ − + + − +

⇔ − − +

⇒ − − +

  

( ) ( )

2 2

2

n n n n n n

H 1 n n

n

a

1 n

y − − + ⇒ − + − − + ⇒ − +

 

Xét hai trường hợp:

2 n

) n n 1 n n

n

 =

+ − + = ⇔ − = ⇔ 

= 

2

) n n 1 n n 0,

+ − + = − ⇔ − + = khơng có giá trịcủa n thỏa mãn Câu 20

Dễthấy xy Không tính tổng quát, giả sửx > y

Từ(3y + 1)  x ( *)

3y p x p N

⇒ + = ∈

Vì x > y nên3x > 3y + = p.x ⇒ p < Vậy p∈{ }1;

• Với p = 1: ⇒x = 3y + 1⇒3x + = 9y +  y⇒ 4y Mà y > nên y∈{ }2;

+ Với y = x = + Với y = x = 13

• Với p = 2: ⇒2x = 3y + 1⇒6x = 9y + 3⇒2(3x + 1) = 9y + Vì 3x + 1y nên 9y + 5y suy 5y , mà y > nên y = 5,

suy x =

Tương tựvới y > x ta giá trịtương ứng

Vậy cặp (x; y) cần tìm là: (7;2);(2;7);(8;5);(5;8);(4;13);(13;4); Câu 21 Ta có: F = n3+ 4n2 – 20n – 48 = (n – 4)(n + 2)(n + 6) Thửvới n= 1; 2; Fđều khơng chia hết cho 125

Thửvới n= F= chia hết cho 125

Vậy sốnguyên dương bé cần tìm là: n= Câu 22.Theo đềbài có: a b+ =k a b k N( − ) ( ∈ *)

( ) ( )

a k b ka b a k mb

⇔ + = − ⇔ + =

Với ka2− =b m m N( ∈ *)⇒m b ka+ = ( )2 Từ(1) (2) có (m – 1)(b - 1)= mb – b – m +

(166)

= a + k – ka2+ = (a + 1)(k + – ka) (3) Vì m > theo (1) nên (m – 1)(b – 1) ≥ Từ(3) => k + – ka ≥ => k + ≥ ka => ≥ k(a – 1) k(a 1) a

k(a 1) a 2,k

 − =  = ⇒ − = ⇒ = =

 

* Nếu a = từ(3) => (m – 1)(b – 1) = => b = b = => (a; b) = (1; 2) (1; 3)

* Nếu a = 2, k = => (m -1)(b – 1) = Khi m = từ(1) => (a; b) = (2; 3) Khi b = => (a; b) = (2; 1)

Thửlại ta có đáp số(a,b) = (1,2),(1,3), (2,3),(2,1) Câu 23

- Xét phép chia xy cho3 Nếu xy không chia hết cho

2

2 2

x 1(mod 3) y 1(mod 3)

x 1(mod 3) y 1(mod 3)

z x y 2(mod 3)

 ≡ ±  ≡ ± 

 ≡

 ⇒ 

≡ 

⇒ = + ≡

(Vơ lí)

Vậy xy chia hết cho (1) - Xét phép chia xy cho Nếu xy không chia hết cho TH1: x 1(mod 4)

y 1(mod 4)

 ≡ ±  ≡ ± 

2

x 1(mod 4) y 1(mod 4)

 ≡  ⇒ 



2 2

z x y 2(mod 4)

⇒ = + ≡ (vơ lí )

TH2: Trong hai sốx,y sốchia dư 2, sốchia dư -1 Khơng tính tổng qt giả sử

2

x 1(mod 4) y 2(mod 4)

x 1(mod8) y 4(mod8)

 ≡ ±  ≡ 

 ≡  ⇒ 

≡ 

2 2

z x y 5(mod8)

⇒ = + ≡ ( vơ lí)

(167)

- Vậy xy chia hết cho (2)

- Từ(1) (2): Vậy xy chia hết cho 12 Câu 24

Ta có

1 1

1.2.3

2

 

=  + + + + 

 

B n

n ( )* sốtựnhiên Thật vậy:

Với n=1 B= ∈1  suy ( )* Với n=2 B= ∈3  suy ( )*

Giả sử ( )* n=k, nghĩa 1.2.3 1

2   =  + + + + ∈    B k k

Cần chứng minh ( )* n= +k 1, nghĩa

( ) 1 ( )

1.2.3

2

  = +  + + + + ∈ +    B k k

Ta có: 1.2.3 ( 1) 1 ( ) 1.2.3 1 ( 1) 1.2.3

2 3

   

= +  + + + + + =  + + + +  + +

 

 

B k k k

k k

1 1

1.2.3

2 1.2.3   + + + + ∈       + ∈ ⇒ ∈   ∈       k k B k

Vậy 1.2.3 1

2

 + + + + 

 

 

n

n sốtựnhiên

Suy ra, với n=2k 1.2.3 1

2

 + + + + 

 

 

k

k

1 1

1.2.3

2

 + + + + 

 

 

k

k số

tựnhiên

Suy 1 ( 1)( 2)

1 2

 + + +  + +

 + + 

k k kk k k sốtựnhiên

Áp dụng chứng minh ta có: 1.2 1009 1

2 1009

 + + + 

 

 

1 1

1010.1011 2018

1010 1011 2018

 + + + 

 

  sốtựnhiên

Ta có 1011 1010.1011 1342 2018 2019 1342 673  ⇒      1

1.2 1009 1010.1011 1342 2018 2019

2 1009

 

⇒  + + + 

  

(168)

Và 3 1.2.3 673 1009 2019 673 673

⇒ 

1 1

1.2 1009 .1010.1011 2018 : 2019

1010 1011 2018

 

⇒  + + + 

 

Vậy sốtựnhiên 1.2.3 2017.2018 1 1

2 2017 2018

 

=  + + + + + 

 

A chia hết cho 2019

Câu 25 Ta có:

( )( )( )( )

( )( )

P(x) x x x x 2010 x 10x 16 x 10x 24 2010

= + + + + +

= + + + + +

Đặt t x 10x 21,= 2+ + biểu thức P(x)được viết lại:

( )( )

P(x)= −t t 2010 t+ + = − +2t 1995

Do chia t 2t 19952− + cho t ta có sốdư 1995 Câu 26 Ta có: g(x) x= 2+ − =x 2 (x x 2− )( + )

Vì f(x) ax= 3+bx 10x 42+ − chia hết cho đa thức g(x) x= 2+ −x 2 Nên tồn đa thức q(x) cho f(x) g x q(x)= ( )

( ) ( )

3

ax bx 10x x x q(x)

⇒ + + − = + −

Với x a b 0= ⇒ + + = ⇒ = − −b a (1) Với x= − ⇒2 2a b 0− + = (2)

Thay (1) vào (2), ta có: a 2; b 4= =

Câu 27 Chia cho thương dư Để chia hết cho chia hết cho

chia hết cho chia hết cho

chia hết cho chia hết cho mà

Thửlại ta thấy thỏa mãn Vậy với chia hết cho Câu 28 Giả sử f x( )=ax4 +bx3+cx dx e3+ + Do f 0( )=enên e 7

Mặt khác: f(1) a b c d e a c

f( 1) a b c d e b d

 = + + + +  +

 − = − + − +  +

 

 

 

f(x) x2 +2 x 3− x 2+

f(x) x2+2 x 2+ x2+2

(x x 2)( )

⇒ + − x2 +2

2

x

⇒ − x2+2

2

x

⇒ + − x2+2

6

⇒ x2+2 x2+ ≥ ⇒2 2 x2+ ∈2 { }3;6 ⇒ ∈ ± ±x { 1; 2} x 1; x= = −2

x 1; x= = −2 f(x) x2+2

(169)

f(2) 16a 8b 4c 2d e 4a c

f( 1) 16a 8b 4c 2d e 4b d

 = + + + +  +

 − = − + − +  +

 

 

 

a c 3a a 4a c c c

 +  

⇒ ⇒ ⇒

+

  

  

  

b d 3b b 4b d d d

 +  

⇒ ⇒

 +  

  

  

  

Vậy hệ sốcủa f(x) chia hết cho Câu 29

Ta có: (x x x x 2033 + )( + )( + )( + )+ = =(x 12x 27 x 12x 35 20332+ + )( 2+ + )+ Đặt x 12x 30 t,2+ + = ta có: (x x x x 2033+ )( + )( + )( + +) = −(t t 2033)( + +)

( )

2

t 2t 15 2033 t t 2018

= + − + = + +

Vậy ta có (x x x x 2033+ )( + )( + )( + )+ =(x 12x 30 x 12x 32 20182+ + )( + + )+ Vậy sốdư phép chia (x x x x 2033+ )( + )( + )( + +) cho x 12x 302+ + là 2018. Câu 30 Giả sử f x( )chia cho x 42− được thương −5xvà cịn dư ax b.+ Khi

( ) ( )

f(x)= x −4 5x ax b− + +

Theo đềCâu, ta có:

( ) ( )

f 26 2a b 26 a 2a b 10 b 18 f 10

 =  + =  =

 ⇔ ⇔

 − + =  =

− =  



Do f x( )=(x2−4 5x 4x 18) ( )− + +

Vậy đa thức f x( )cần tìm f(x)=(x 5x 4x 182− ) ( )− + + Câu 31 Ta có: P(0) = d

P(1) = a + b + c + d  => a + b + c  (1) P(-1) = -a + b – c + d  => -a + b – c  (2)

Từ(1) (2) suy 2b  => b  (2,5) = 1, suy a + c  P(2) = 8a + 4b + 2c + d  => 8a + 2c  => a  => c

Câu 32

Ta có 20 ( )( 16 12 )

1 1

p − = pp +p + p + p +

Do p sốnguyên tốlớn nên p sốlẻ

2

1

p

⇒ +

1

p − sốchẵn

4

1

p

⇒ − chia hếtcho

20

1

p

⇒ − chia hết cho

p sốnguyên tốlớn ⇒ p sốkhông chia hết cho

Lập luận ta

1

p − chia hết cho

(170)

Lập luận ta 16 12

1

p + p +p +p + chia hết cho

Suy 20

1

p − chia hết cho 25

Mà (4; 25)=1 nên p20−1 (đpcm)

Câu 33.Từgiảthiết  + +  = + + ⇔  + + =

   

2

2 2

1 1 1 2 1 0 a b c a b c ab bc ca

Vì a, b, c ≠0 nên a + b + c =

( ) ( )

⇒ + = − ⇒ + = −

⇒ + + + = − ⇒ + + =

3

3 3

3 3

a b c

a b c

a b 3ab(a b) c a b c 3abc

Vậy a3+b c 33+ 3 với a, b, c ∈Z Câu 34 Ta có :

N= k4+ 2k3 – 16k2 – 2k +15 = (k4- k2)+(2k3 – 2k)- (15k2 – 15) = (k2-1)(k2+ 2k – 15) = (k-1)(k+1)(k-3)(k+5)

Ta thấy với k sốnguyên lẻthì N tích thừa số ( nhân tử) chẵn Do N chắn chia hết cho 16 Vậy k phải sốnguyên lẻ

Câu 35

Vì sốthứnhất chia cho dư nên có dạng 5a 1+ , sốthứhai chia cho dư nên có dạng 5b 2+ (a,b∈)

Ta có tổng bình phương hai sốđó là:

( ) (2 )2 2 2 ( 2 2 )

5a 1+ + 5b 1+ =25a 10a 25b 10b 5a 5b 2a 2b 5+ + + + + = + + + + 

Vậy tổng bình phương hai sốchia hết cho Câu 36 Ta có:

chia hết cho chia hết cho 13 Do nên chia hết cho 91

Câu 37 Ta có:

A = + + 32+ 33 + + 311

= ( + + 32+ 33) + (34+ 35+36+ 37)+ (38+ 39+ 310+ 311)

( ) ( )

n n n n n n n n n

A 5 3= + − +2 =25 +5 18 12− −

( n n) ( n n)

A= 25 18− − 12 −5 A

( n n) ( n n)

A= 25 12− − 18 −5 A

(13,7)=1 A

(171)

= ( + + 32+ 33) + 34 (1 + + 32+ 33) + 38(1 + + 32+ 33) = 40 + 34 40 + 38 40

= 40 (1 + 34+ 38)  40 Vậy A  40

Câu 38 Gọi dư phép chia cho Ta có:

Theo bàira : nên ta có: nên

Vậy đa thức cần tìm Câu 39 Ta có:

Ta chứng minh

Thật , từđẳng thức có chữ sốtận

Đặt ta có:

Nếu tận

Suy

Từ suy Câu 40.

Theo giảthiết p−5 chia hết đặt p=8k+5 (klà sốtựnhiên)

Ta có ( )2 ( )2 ( 2) 4

k k k k k k

ax + by + ax by a + x + b + y + p

 −  − ⇒ −

 

  

( 4k 4k 2) 8k 4k 2( 8k 8k 4)

a + b + x + b + x + y + p

⇒ + − + 

Mà 4k 4k ( )2 2k ( )2 2k 2

a + +b + = a + + b + a +b = p b< px8k+4+y8k+4p ( )*

Nếu hai số x y, có sốchia hết cho pthì từ(*) ta suy sốthứhai chia hết

cho p

Nếu cảhai không chia hết cho p, theo định lý Fec-ma ta có

( ) ( )

8 8

1 mod mod

k k k k

x + ≡ y + ≡ px + +y + ≡ p mâu thuẫn với (*) Vậy hai sốx, y chia hết cho p

( )

f x x2 −1 ax+b

( ) ( )( )( )

2 3 1

f x = xxx − +ax+b

(2) 5

f = 2a+ =b 5; (3)f =7 3a+ =b 7 2; 1

a b

⇒ = =

( ) ( )( )( )

2 3 1 2 1

f x = xxx − + x+

2n =10a+ ⇒b b2⇒ab2 (1) 3 (2)

ab

2n =10a+ ⇒b 2n b

( )

4 , ,0 3

n= k +r k r∈ ≤ ≤r 2n =16 2k r 0

r = 2n −2r =2 16r( k −1 10) ⇒2n 2r

( )

2r 10 2n 2r 2 16r k 1 3 3 3

b= ⇒ a= − = −  ⇒a ⇒ab

( )1 ( )2 ab6

(172)

Câu 41.Do a3+b3+c3 chẵn nên số , ,

a b c có sốchẵn Từđó suy

tích abc chia hết cho ( )1

Giả sửtrong ba số a b c, , khơng có sốnào chia hết cho Ta thấy rằng, với x ngun

khơng chia hết cho x≡ ± ± ±1, 2, mod 7( ), suy x3≡ ±1 mod 7( ) Do ( )

1 mod

a ≡ ± , b3≡ ±1 mod 7( ), c3≡ ±1 mod 7( )

Suy 3 ( )

3, 1, 1, mod

a +b +c ≡ − − , tức a3+b3+c3 không chia hết cho 7, mâu thuẫn Vậy

trong ba số a b c, , phải có sốchia hết cho Từđó suy tích abc chia hết cho ( )2

Từ ( )1 ( )2 với ý ( )2;7 =1, ta có abc chia hết cho 14 Câu 42.

a) n=3k suy 2n+ =1 8k + ≡ −1 ( )1k +1 mod 9( ) Suy k lẻ, k= +2t Suy n=3 2( t+ = +1) 6t

Nếu n=3k+1 ta có 2n+ =1 3.8k + ≡ −1 ( )1 mod 9k + ( ) suy 2n+1 không chia hết cho Vậy với n= +6t 2, với t sốtựnhiên sốcần tìm

b) Cách 1: Ta có 2km −1 2 m−1 Từ 22n=(2n+1 2)( n−1) Đặt 2n=km+q(0≤ <q m)

Khi 22n− =1 2km q+ −2q+2q− =1 2q(2km − +1) 2q−1 chia hết cho 2m−1, suy

2q−1 chia hết cho m mà 0≤2q− <1 2m−1, suy q=0

Do 2n =km

Trường hợp 1: Nếu m lẻ, suy k chẵn, k =2 'k , suy n=k m' , 2n+ =1 2k m' +1=2k m' − +1 chia hết cho 2m−1, suy chia hết cho 2m−1 vô lý

Trường hợp 2: Nếu m chẵn m=2 'm nên n=km', suy 2km'+1 chia hết cho 2m−1, mà 2m−1 chia hết cho 2m'−1 nên 2km'+1 chia hết cho 2m'−1, suy 2 chia hết cho 2m'−1 vơ lý

'

m >

Cách 2: Ta có 2n m− (2m−1 2) m−1, suy 2n−2n m− 2m−1, mà 2n+1 2 m−1 suy 2n m− +1 chia hết cho 2m −1

Lý luận tương tựta có 2n km− +1 chia hết cho 2m−1 Giả sử n=km+q, 0≤ <q m

(173)

Chọn k trên, ta có 2q+1 chia hết cho 2m−1 Mà q<m nên 2q+ =1 2m−1, giải 1,

q= m= (vô lý)

Câu 43.

4

9.3 8.2 2019 9.81 8.16 2019

= nn+ = nn+ M

Ta có:

81 1(mod 4) 81 1(mod 4) 9.81 1(mod 4)

8.16 0(mod 4)

1 2019 2020 0(mod 4)

≡ ⇒ ≡ ⇒ ≡ ≡ ≡

⇒ ≡ − + ≡ ≡

n n

n

M

hay M4 (1)

Lại có:

81 1(mod 5) 81 1(mod 5) 9.81 4(mod 5)

16 1(mod 5) 16 1(mod 5) 8.16 3(mod 5)

4 2019 2020 0(mod 5)

≡ ⇒ ≡ ⇒ ≡ ≡ ≡ ⇒ ≡ ⇒ ≡ ≡ ⇒ ≡ − + ≡ ≡

n n

n n

M

hay M5 (2)

Từ(1) và(2) ⇒M BCNN (4,5) hay M20 (đpcm)

Câu 44.

Đặt n=6q+r r, ∈{0,1, 2,3, 4,5} Khi n3+2019 chia hết cho r3+3 chia hết cho

Nếu r chẵn r3+3 lẻ, r3+3 khơng chia hết cho Suy r∈{1,3,5 }

Với

1

r= ⇒r + = không chia hết cho Với

3 30

r= ⇒r + =  Với

5 128

r= ⇒r + = không chia hết cho Suy n=6q+3.Mà 0≤ ≤n 2019⇒ ≤ ≤0 q 336

Vậy có tất cả337 sốtựnhiên n thỏa mãn đềbài

Câu 45. Ta có:

( ) ( )

( ) ( ) ( )( )

2 2

2

2 2

2

x xy y xy y x x xy y x

x xy xy y x y x y x y

− − + − − = − − − = + − + − + = + − −

Lại có: 2

2 ,

xxyy xyyxchia hết cho

(x y)(x 2y 1)

⇒ + − − chia hết cho

(174)

TH1: Nếu x+ychia hết cho5 y≡ −x(mod 5)

( )

2 2

0 x 2xy y x 2x x x 3x (mod 5)

⇒ ≡ − − ≡ + + = + , xchia hết cho chia

+)Nếu xchia hết cho ycũng vậy, toán chứng minh +)Nếu xchia cho dư y chia dư 2,

2

2x + y +2x+ ≡y 2.9+ +4 2.3=30≡0(mod 5)

Ta có điều phải chứng minh

TH2: Nếu x−2y−1chia hết cho x≡2y+1 mod 5( )

( )2 ( )

2

0 x 2xy y 2y (y y 1) y y mod

⇒ ≡ − − ≡ + − + − = + Do ychia dư xcũng chia dư nên:

( )

2

2x + y +2x+ =y 2.16 16+ +2.4+ =4 60≡0 mod

Vậy ta có điều phải chứng minh

Câu 46 Giả sử a b c; ; sốnguyên không âm thỏa mãn đềbài, ta có:

( ) (2 ) (2 )2 2 2 2

6 3

ab + bc + ca = abca +b +cabbcca= abc (1)

Phân tích 3 ( )( 2 )

3

a +b + −c abc= a+ +b c a +b +cabbcca (2)

Từ(1) (2) 3 ( )

3 3

a +b + −c abc= abc a+ +b c hay a3+b3 +c3 =3abc a( + + +b c 1) Do 3

1

a +b + +c chia hết cho a+ + +b c 1 nên ta chia hết cho a+ + +b c 1 Suy a = = =b c 0.

Thửlại: a= = =b c 0 thỏa mãn Vậy có số (a b c; ; ) (= 0;0;0) thỏa mãn đề

Câu 47. Ta có: n n n n n n n

a −b = −(a b)(a − +a − b a+ − b + + ab − +b − )

n n

a b m(a b) (a, b, n, m )

⇒ − = − ∈ (*)

Vì n sốtựnhiên chẵn nên n = 2k (k∈) ⇒ A = n n n k k k

20 −3 +16 − =1 400 −9 +256 −1 Áp dụng (*), có: k k k k

A=(400 −1 ) (256+ −9 )=399x+247y 19 21x 19.13y (x, y= ⋅ + ∈)

⇒A 19 với sốtựnhiên n chẵn (1)

có: k k k k

A=(400 −9 ) (256+ −1 )=391p 255q+ =17 23p 17 15q (p, q⋅ + ⋅ ∈)

⇒A 17 với sốtựnhiên n chẵn (2)

mà 17 19 hai sốnguyên tốcùng nên từ(1) (2) suy ra:A 17 19 ⋅ với số

tựnhiên n chẵn Vậy n n n

20 −3 +16 −1 323 với sốtựnhiên n chẵn

Câu 48. Ta có ngay: ( )

1

2

n n S = + + + + =n +

(175)

Ta sẽchứng minh S2019chia hết cho n

2

n+

Giả sử nlẻthì

2

n+

nguyên Sửdụng khai triển Newton ta có:

( )( )

2 2 2

( )

k k k k k

a + +b + = a+b aab+ +ba+b Do vậy:

( ) ( ) ( ( ) ) ( ) ( )

( ) ( ( ) ) ( ( ) ) (( ) )

2019

2019 2019 2019 2019 2019 2019 2019 2019 2019 2019 2019

2019 2019 2019 2019 2019 2019 2019

2 2 1

2 1 2 1

n n n n n

n n n n n n

+ + + = + + + − + + + +

+ + + = + − + + − + + − + +

Do (n n; + =1) 1nên ( 2009 209 2009)

2 +2 + +n chia hết cho n n( +1)

Do S2019S1

Vậy ta có điều phải chứng minh Câu 49 a) Chứng minh rằng…

Ta có: 9! 1.2.3.4.5.6.7.8.9= sốchẵn ⇒ x32⇒x2⇒ =x 2m (m∈)

3 3 3

8m 2y 4z 9! 4m y 2z 1.3.4.5.6.7.8.9

⇒ + + = ⇔ + + = sốchẵn

( )

3

3 3

2 2

4 1.3.4.5.6.7.8.9

y y y n n

m n z

⇒ ⇒ ⇒ = ∈ ⇒ + + =

  

3 3

2m 4n z 1.2.3.5.6.7.8.9

⇔ + + = sốchẵn

( )

3

2 2

z z z p p

⇒  ⇒  ⇒ = ∈

3 3 3

2 1.2.3.5.6.7.8.9

2 1.3.5.6.7.8.9

m n p

m n p

⇒ + + = ⇔ + + =

Chứng minh hoàn toàn tương tựta có ( )

2

2 ; ;

2

m x m

n m n p y n

p z p

=    ∈ ⇒ =     =         

Vậy ta có điều phải chứng minh

b) Chứng minh không tồn tại…

Theo ý a) ta có thểđặt x=4 ;a y=4 ;b z =4c (a b c; ; ∈)

3 3

3

9! 1.2.3.4.5.6.7.8.9

2 1.3.5.6.7.9

4

a b c

⇒ + + = = = sốchẵn

( )

2 2

a a u u

⇒  ⇒ = ∈

3 3 3

8u 2b 4c 1.3.5.6.7.9 4u b 2c 1.3.3.5.7.9 1.5.7.3

⇒ + + = ⇔ + + = = Lại có:

( ) ( )

( )

( )

4 4

3 3

1.5.7.3 1.5.7.3

0; (mod 9)

; ; 9

x x Z

a b c u b c

 ⇒   ≡ ± ∈  ⇒ ⇒ + +    

Nhưng

1.5.7.3 không thểchia hết cho 93nên ta có điều vơ lý

(176)

Vậy ta có điều phải chứng minh Câu 50.

Ta có nhận xét sau: Nếu p sốnguyên tốlớn

1(mod 24) (1)

p ≡ Lại có: − ≡1 23 mod 24( ) (2)

Cộng vếtheo vếcủa ( ) ( )1 ; 2 ta : p2− ≡1 24(mod 24)≡0 mod 24( )

Vậy

1

p − chia hết cho 24 với p sốnguyên tốlớn Câu 51.

( ) ( )

3

1

n − = nn + +np

(p−1)n⇒ − ≥ ⇒ ≥ +p 1 n p n 1

p≥ + ⇒n 1 (n−1)không chia hết cho p

Do đó: ( )( ) ( )

1 1

nn + +npn + +np Đặt : p− =1 kn, k≥ ⇒ =1 p kn+1 (*)

( ) ( ) ( ) ( ) ( ) ( ) ( ) ( ) ( ) 2 2

1 1

1

1 1

1

1

1

n n kn kn n n

kn n n k n

k n n n kn kn

k n k kn

k k n k

k n k kn

⇒ + + + ⇒ + ≤ + + ⇔ ≤ + ⇔ ≤ +

+ + − + + ⇒  − +  +

≥ ⇒ − + > ⇒ − + ≥ +    ( ) 2

1 1

2 1

k n

k n p kn n n

n p n n n

⇒ ≥ +

⇒ = + ⇒ = + = + + ⇒ + = + + = +

Vậy n+plà sốchính phương

Câu 52.+) Chứng minh đẳng thức

( )( )

( )

1 2

1 2 3 2 1 2 2

n n n n n n

n n n n n n n n n n n n n

a b a a b a b a b ab b

a a b a b a b a b ab a b a b a b ab b

a b − − − − − − − − − − − − − − − − + + + + + + = + + + + + + − + + + + + = −

+) Vì nlà sốchẵn, đặt n=2 ,k k∈ta có:

2 2

20n+16n− − =3n 20 k +16 k −3 k − =1 400k +256k −9k −1

Đểchứng minh (20n+16n− −3n 323) , ta cần chứng minh (20n+16n−3n−1)chia hết cho

19 17 Ta có:

(177)

( )( )

( )

( )

( )

1 2 1 2

1 2 1

400 400 400 400 400 400.1

399 400 400 400 400.1

19.21 400 400 400 400.1 19

256 256 256 256

k k k k k k k

k k k k k

k k k k k

k k k k

− − − − − − − − − − − − − − − − − − = − + + + + + = + + + + + = + + + + + − = − + + +  ( ) ( ) ( ) ( )

1 2

1 2

9 256

247 256 256 256

13.19 256 256 256 19

400 256 19

k k

k k k k

k k k k

k k k k

− − − − − − − − − − + = + + + + = + + + + ⇒ − + −  

Tương tựta có:

( )( )

( )

( )( )

( )

1 2 1 2

1 2

1 2

400 400 400 400 400

17.23 400 400 400 17

256 256 256 256 256.1

17.15 256 256 256.1 17

400 256

k k k k k k

k k k k

k k k k k

k k k k

k k k

− − − − − − − − − − − − − − − − − = − + + + + = + + + + − = − + + + + = + + + + ⇒ − + −   17 

Như ta có:

( )

( ) ( ) ( )

( )

20 16 400 256 19

20 16 19.17

20 16 400 256 17

20 16 323

n n n k k k

n n n n n n k k k

n n n

 + − − = + − −  ⇒ + − −  + − − = + − −  ⇒ + − −    

Như ta có điều cần chứng minh

Câu 53.Ta có : 5 ( )

1 2018; 2018 1; 2018

N = +a a + +a M =a +a + +a a a a ∈+ Xét hiệu

( ) ( ) ( )

5 5

1 2018 2018

5 5

1 2 2018 2018

M N a a a a a a

a a a a a a

− = + + + − − − − = − + − + + −

Ta có ( ) ( )( ) ( )( )( )

1 1 1

a − =a a a − =a a + a − =a a + aa+

30=2.3.5với 2,3,5đều sốnguyên tố

Ta có: ( )( )( )

1 1

a a + aa+ có tích sốtựnhiên liên tiếp a−1; ;a a+1nên

( 1)( 1)

a aa+ sẽchia hết cho

Nếu achia cho dư 0,1 a a, −1,a+1sẽchia hết cho

Nếu achi dư a+1sẽchia hết cho Vậy ( )( )( )

1 1

a a + aa+ sẽchia hết cho cả2;3;5 nên sẽchia hết cho 30

Do MNchia hết cho 30 M chia hết cho 30

Ta có diều phải chứng minh

(178)

Câu 54.Ta có:

( )

( ) ( ) ( )

2018 2019 2020 2016 2017 2018 2016 2017 2018

2015 2016 2017

1 1

( 1)( 1) ( 1)( 1) ( 1)( 1)

a b c a b c

a a b b c c

a a a a b b b c c c c

+ + − + + = − + − + −

= − + + − + + − +

Ta có tích số tựnhiên liên tiếp chia hết cho 6, có sốchẵn số chia hêt cho Do vậy:

( 1)( 1); ( 1)( 1); ( 1)( 1)

a aa+ b bb+ c cc+ chia hết

( )

2018 2019 2020 2016 2017 2018

6

a b c a b c

 + + − + + 

 

Vậy ta có điều phải chứng minh Câu 55.

+) n= 2k(knguyên dương): M= 2k.42k+ 32k= 2k.16k+ 9k Ta có: 16kvà 9kcùng dư với 2kchia

7

Mcùng dư với (2k.2k+ 2k) = 2k.(2k+ 1) chia 7⇒(2k+ 1) chia hết cho 7⇒kchia7 dư 3,

hay k= 7q+ ⇒n= 14q+ (q ∈N)

+) n= 2k+ (knguyên dương): M= (2k+ 1).42k+ 1+ 32k+1= 4(2k+1).16k+ 3.9kMcùng dư với (k + 4).2k+ 3.2k= (k+ 7).2kchia

kchia hết cho 7⇒k= 7p (p∈N)

Vậy n= 14q+ n= 14p+ 1, với pqlà sốtựnhiên Câu 56.

Chứng minh với sốtựnhiên n thì 𝑛3−9𝑛+ 27khơng chia hết cho 81.

Giả sửtồn sốtựnhiên nđể𝑛3−9𝑛+ 27⋮81,

suy 𝑛3−9𝑛+ 27⋮3hay 𝑛 ⋮3

=> n=3k 𝑛3−9𝑛+ 27 = 27(𝑘3− 𝑘+ 1)

mà 𝑛3−9𝑛+ 27⋮81nên 𝑘3− 𝑘+ 1⋮3

Nhưng 𝑘3− 𝑘+ = (𝑘 −1).𝑘 (𝑘+ 1) + 1không chia hết cho với mọi k

Vậy với sốtựnhiên n thì 𝑛3−9𝑛+ 27khơng chia hết cho 81.

Câu 57

Đặt ( )2

4

A= m+nmn

(179)

( ) ( ) ( ) ( ) ( ) ( ) ( ) ( ) ( ) ( ) ( ) ( ) ( ) ( ) ( ) ( )

2 2

2 2 2 2

4 16 16

15

3

225 15 225

5 25

3 15 15

15 225

15

5 15

A m n mn m n m n m n

m n m n

m n m n

A m n

m n m n

m n m n m

m n mn

n

m n m n

  ⇒ = + − = + − + − −  = + + −   − −   = ⇒ ⇒ ⇒ − −   −    +  −    ⇒ + ⇒ ⇒ ⇒ ⇒ + +                   

Vậy ( )2

4 m+nmnchia hết cho 225 mncũng chia hết cho 225

Câu 58 Ta có

( 1)

1

2

n n S = + + + =n +

Để chứng minh toán ta cần chứng tỏ 2S2019n n( +1)

Ta có nhận xét sau: Với a b, nguyên dương (a2019+b2019)(a b+ )

Thật vậy:

2019 2019 2018 2017 2017 2018

( )( ) ( )

a +b = a b a+ −a b+ −ab +ba b+

Xét hai trường hợp:

+) Nếu n lẻ: Từ nhận xét ta có 2S2019 =2n2019+2 1 2019+ −(n 1)2019+2 2 2019+ −(n 2)2019+ 2019 2019 1 2 n n n  −   +   +   +          

2019 2019 2019 2019

2019

2S =2(1 +n )+2 2 + −(n 1) +

2019 2019 2019 2019

1 1

( 1)

2 2

n n n n

n  −   +    +   +   +   +   +   +   +                

Mặt khác n n+1nguyên tố nên 2S2019 n n( +1)

+) Nếu n chẵn: Ta có

2019 2019 2019 2019 2019

2019

2S =2n +2 1 + −(n 1) +2 2 + −(n 2) +

2019 2019 2019 2019

2

2 2

n n n n

n  −   +        +   +   +   +                  

2019 2019 2019 2019

2019

2S =2(1 +n )+2 2 + −(n 1) +

2019 2019 2019 2019

2

2 ( 1)

2 2

n n n n

n  −   +      +   +   +  +   +   +                

Suy 2S2019 n n( +1)

Vậy S2019 S1

Câu 59.

Ta có: p+(p+2)=2(p+1)

Vì p lẻnên (p+1) 2 =>2(p+1) 4 (1)

(180)

Vì p, (p+1), (p+2) sốtựnhiên liên tiếp nên có sốchia hếtcho 3, mà p (p+2) nguyên tốnên (p+1) 3 (2)

Từ(1) (2) suy [p+(p+2) 12] (đpcm) Câu 60.

Ta có với sốnguyên m m2chia cho dư , 4. + Nếu n2chia cho dư 2 *

5 5 5;

n = k+ =>n + = k+  kN

Nên n2+4 không sốnguyên tố

Nếu n2chia cho dư 2 *

5 16 20 5;

n = k+ =>n + = k+  kN

Nên n2+16 không sốnguyên tố Vậy n2  hay n 5

Câu 61. Ta có

( )

5 5

S =n n + n + nn

( )( ) ( )

1

nn n n n

=  − + + − 

( )( )

1

n n n n

= − + +

( 1)( 1)( 2)( 3)

n n n n n

= − + + +

(n 1) (n n 1)(n 2)(n 3)

= − + + +

Ta có S tích số nguyên tự nhiên liên tiếp chia hết cho 5! nên chia hết cho 120

Câu 62.

Với sốnguyên dương a, b ta có:

2015 2015 2014 2013 2013 2014 2015 2015

( )( ) ( )

a +b = a b a+ +a b+ +ab +ba +ba b+

+ Xét trường hợp n sốlẻ Áp dụng khẳng định ta có:

2015 2015

2015 2015

2015 2015

2 ( 1)

2 ( 2)

1 2 n n n n n n n  + −     + −     −   +   +                 Suy 2015 2015

2015 2015 2015 2015 2015 1

2 ( 1) 2 ( 2)

2

n n

A=n +  + −n +  + −n + +  −  + +   n

   

 

 

Tương tự

2015 2015 2015 2015

2015 2015 2015 2015 1

2(1 ) 2 ( 1) ( 1)

2 2

n n n n

A= +n +  + −n + +  −  + +    +  +  + +   n+

       

   

   

(181)

Mặt khác n n + nguyên tốcùng nên A ⋮n(n + 1) Tương tựvới trường hợp n chẵn ta có A ⋮n(n + 1) Câu 63. Ta có:

4

3

2 16 15 ( 2 1) (16 16)

( 1)( 1) 16( 1)

Q a a a a a a a a

a a a

= + − − + = + − − − −

= − + − −

Với alẻ, a=2k+1, kZ

Khi đó: 3

(a−1)(a+1) =2 (2k k+2) =16 (k k+1) 16.

16(a −1) 16 nên Qchia hết cho 16

Với achẵn, a=2 ,k kZ

Khi đó: 3

(a−1)(a+1) =(2k−1)(2k+1) số lẻ nên khơng chia hết cho 16 Do Qkhơng

chia hết cho 16

Vậy alà số nguyên lẻ Câu 64

+) Vì số nguyên phải số chẵn số lẻ Do theo nguyên lý Đirichlet số nguyên ln chọn số có tính chẵn lẻ

+) Áp dụng ta có số phương ln chọn hai số có tính chẵn lẻ Gọi số phương chọn

a b2 Khi ta có

2

( )( )

ab = a b a b− +

+) Vì

a b2 tính chẵn lẻ nên a,b tính chẵn lẻ Do a b− số chẵn a b− số chẵn 2

( )( )

ab = a b a b− +  , (đpcm)

Chú ý:

Ta giải tốn cách vận dụng tính chất sau số phương: “Một số phương chia cho có số dư hặc 1” Khi lập luận cách làmtrên ta thu điều phải chứng minh Tuy nhiên làm thi vận dụng tính chất học sinh phải chứng minh lại.

Bình luận: Với cách làm ngắn gọn, đầy đủ song số học sinh cảm thấy trừu tượng ( ngun lí Đirichlet học sinh ơn tập khơng nằm chương trình SGK mà sách tham khảo) tốn trình bày sau:

Trong ba số nguyên tùy ý tồn hai số chẵn lẻ Gọi hai số phương chọn

a b2( a, b nguyên)

+ TH1: a, b chẵn: suy 2 2 2

1 2

(2 ) (2 ) 4( )

ab = kk = kk chia hết cho ; ,k k1 2∈Z

+ TH2: a, b lẻ: suy 2 2 2

1 1 2

(2 1) (2 1) 4( )

ab = k + − k + = k + −k kk chia hết cho

1

; ,k kZ

Vậy ba số phương tùy ý tồn hai số mà hiệu chúng chia hết cho Câu 65.

Cách 1: 1 1= + ⇔bc a(b c) (1)= +

a b c

TH1: Nếu a số nguyên chẵn, suy a(b c) 2+  , theo (1)Suy ra: b.c 2 Vậy abc chia hết cho

TH2: Nếu a số nguyên lẻ Với b c hai số lẻ thì: b c 2+  ⇒a(b c) 2+ 

(182)

a b c . khơng chia hết cho (vì a, b, c lẻ) Suy mâu thuẫn

Vậy hai số, b, c tồn số chẵn

+ Với b chẵn, mà a lẻ nên c chẵn (vì b.c chẵn nên a(b+c) chẵn suy c chẵn, a lẻ) Suy abc chia hết cho

+ Với c chẵn, tương tự abc chia hết cho

Cách 2: 1 1= + ⇔ bc a(b c) = + ⇔abc = a (b + c) (2)2

a b c

Ta thấy a, b, c số lẻ vây abc số lẻ, b+c số chẵn Vậy số tồn số chẵn

Nếu a chẵn a2 chia hết cho 4, từ (2) suy abc chia hết cho Nếu b chẵn, a lẻ nên b + c chẵn (vì abc chẵn) suy c chẵn

Vậy abc chia hết cho

Tương tự cho trường hợp c chẵn Câu 66.

Ta có

2 n 4n 16

A= + + =(22n − +1) (4n− +1) 18 Đặt 2

2 n =2 k (k∈*) suy 22n − =1 22k − =1 4k −1 3 Do với nnguyên dương ta có: 22n 1 3; 4 1 3; 18 3

n

−  −  

2

2 n 4n 16 3

A

⇒ = + + 

Câu 67.

Ta có A=4n +17 =(4n − +1) 18

Với nnguyên dương ta có: 4n−1 3; 18 3  4n 17 3

A

⇒ = + 

Câu 68.

Giả sử 2 ( *)

2n+ =1 m , 3n+ =1 k m k, ∈N

m

⇒ sốlẻ ⇒mlà sốlẻ

( )( )

2

2n m m m

⇒ = − = − +  , Suy ra: n chẵn, k lẻ

Vì k sốlẻnên k−1,k+1là hai sốchẵn liên tiếp (3, 8) = nên

Từ 2 ( )( )

3n+ =1 k ⇒3n=k − =1 k−1 k+1 8 ⇒n8 (1)

Khi chia sốchính phương cho sốdư chỉcó thểlà 0; 1; Ta xét trường hợp:

Nếu n chia cho dư 2n + chia cho dư ( vơ lí ) Nếu n chia cho dư 3n + chia cho dư ( vơ lí ) Nếu n chia cho dư 2n + chia cho dư ( vơ lí ) Nếu n chia cho dư 3n + chia cho dư ( vơ lí ) Vậy n5 (2)

Vì (5, 8) = nên từ(1) (2) suy n chia hết cho 40

(183)

Câu 69.

Chứng minh với sốnguyên n chẵn thì: n3+20n 96+ chia hết cho 48

Ta có n chẵn ⇒ =n ,k kZ Suy

( )3 ( ) ( ) ( )

3 20 96 2 40 96 8 5 96 8 6 96 8 48 48.2

n + n+ = k + k+ = k + k + =  k − +k k+ = k − +k k+

Do k−1; ;k k+1 sốnguyên liên tiếp nên (k−1 ) (k k+1) chia hết cho

( ) ( ) ( )

3

1 48,

k k k k k k k k Z

⇒ − = − +  ⇒ −  ∀ ∈

Vậy với sốnguyên n chẵn n3+20n+96 chia hết cho 48

Câu 70.

Ta cóp a b= 3− =(a b a− )( 2+ab b+ 2).

a, blà sốnguyên dương nên, ta có a2+ab b+ >1.

Do pnguyên tốnên a b− = ⇒ = + ⇒ =1 a b 1 p 3b2+3b+1

( 2 ) ( )2

4p 4b 4b 1 2b 1

⇒ = + + + = + + (đpcm) Câu 71.

Cách 1:1 1 bc a b c( ) (1)

a = + ⇔b c = +

TH1: Nếu a sốnguyên chẵn, suy a b c( + ) 2 , theo (1)Suy ra: b.c 2

Vậy abc chia hết cho

TH2: Nếu a sốnguyên lẻ Với b c hai sốcũng lẻthì: b c+ 2⇒a b c( + ) 2

a b c không chia hết cho (vì a, b, c lẻ) Suy mâu thuẫn

Vậy hai số, b, c tồn sốchẵn

+ Với b chẵn, mà a lẻnên c chẵn (vì b.c chẵn nên a(b+c) chẵn suy c chẵn, a lẻ) Suy abc chia hết cho

+ Với c chẵn, tương tựabc chia hết cho Cách 2:

( )

2

1 1

( ) (2)

bc a b c abc a b c a = + ⇔b c = + ⇔ = +

Ta thấy a, b, c khơng thểđều sốlẻvì vây abc sốlẻ, cịn b+c sốchẵn Vậy sốtồn sốchẵn

(184)

Nếu a chẵn a2 chia hết cho 4, từ(2) suy abc chia hết cho

Nếu b chẵn, a lẻnên b + c chẵn (vì abc chẵn) suy c chẵn Vậy abc chia hết cho Tương tựcho trường hợp c chẵn

Câu 72. 1.Ta có:

2016 504 504

1 ( ) 1 ( 1). ( 1)( 1)( 1). (1) ( )

p − = p − = pA= pp+ p + A AN

Vì P số nguyên tố lớn nên p số lẻ, suy p – 1, p +1 hai số chẵn liên tiếp

(p 1)(p 1) 4 (2)

⇒ − + 

Vì p – 1, p, p+1 ba số tự nhiên liên tiếp nên (p−1) (p p+1) 3 Nhưng p không chia hết

cho nên (p−1)(p+1) 3 (3)

Vì p khơng chia hết p có dạng5k ±1; 5k ±2 - Nếu p=5k±1thì p2 =25k2 ±10k + =1 5n+1

- Nếu p=5k ±2thì p2 = 25k2 ±20k+ = −4 5l 1

Cả hai trường hợp cho ta

1 5 (4)

p − = q (( , ,n l qN)

Vì 3, 4, số nguyên tố đôi nên từ (1), (2), (3), (4) suy 2016

1

p − chia hết cho 4.3.5 tức chia hết cho 60

2 Vì vai trị x, y, z bình đẳng nhau, khác đơi nên ta giả sửx< <y z

Khi , gọi t thương phép chia 3 2 :

x + y + z x y z Suy :

3 3

3 3 2 2 2 2

2 2 (1)

x y x y

x y z tx y z z tx y tx y tx y x y

z x y

+ +

+ + = ⇔ = − > − = − −

- Nếu 2

0

tx y − − <x y (*) t 12 12 t

xy x y

< + < ⇒ = Thay t = vào (*), ta 2

0 ( 1)( 1)

x y − − < ⇒x y xy− − < ⇒x y xy− <

1

x

⇒ =

2

0 ( 1)

y y y y

⇒ − < ⇔ − < ( vô lý) Vậy 2

0 (2)

tx y − − ≥x y

- Từ (1), (2) suy : 2 2

( ) (3)

ztx y − −x y

- Mặt khác 3 2

x +y +z =tx y z nên x3+ y z3 ⇒ x3+y3≥z2 (4)

- Từ (3) (4) suy :

(185)

3 2

3 4 2 2 3 2 4

3 2 3

3

( )

2 ( )

2 ( )

2 ( )

1 1

2 (5)

x y tx y x y

x y t x y tx y x y x xy y

x y tx y x y t x y

x y tx y x y

txy

tx y txy

x y tx ty

+ ≥ − −

⇔ + ≥ − + + + + ⇒ + + + >

+ + + ⇔ <

 

⇔ <  + + +  

- Nếu x≥2 1 13 13 1 13 13

2 2

y txy

t t x y t x t y

   

≥ ⇒ ≥ >  + + + >  + + +    

Điều mâu thuẫn với (5) Vậy x = Khi (5) trở thành :

3

2 1

2 (6)

ty

y t ty < + + +

- Nếu y≥4 2 13 2 13

4

ty

t t y t ty

≥ > + + + ≥ + + + Điều mâu thuẫn với (6)

Vậy y∈{ }2;3 (Vì y > x = 1)

+ Nếu y =

3

9

1; 2;

1;

x y z

x y z

x y z

x y  + =  ⇔ = = = ≤ ≤   = =  

+ Nếu y =

3

28

1;

x y z

x y z

x y  + =  ≤ ≤   = =   ( Loại)

- Thử lại ta thấy (x, y, z) = (1, 2, 3) hốn vị thỏa mãn Vậy thương phép chia 3 2

:

x + y + z x y z t =

Câu 73. Cách 1:

2 2 2 2

24a + =1 b ⇔25a + =1 a +ba +b ≡1(mod 5) (1)

Ta có: 0, 1, 2(mod 5)

0, 1, 2(mod 5)

a b ≡ ± ±   ≡ ± ±  2

0,1, 4(mod 5) 0,1, 4(mod 5)

a b  ≡  ⇒  ≡

 (2)

Từ (1) (2) suy ra: 22 0(mod 5)

1(mod 5) a b  ≡   ≡



2 1(mod 5) 0(mod 5) a b  ≡   ≡ 

Suy chỉmột sốa b chia hết cho

(186)

Cách 2:

2 2 2 2

24a + =1 b ⇔25a + =1 a +ba +b =5.k+1 (1)

{ }

( )

5 , 0;1; 2;3;

n Z n l r l Z r

∀ ∈ ⇒ = + ∈ ∈ 2 ( { })

1 1

5 , 0;1;4

n l r l Z r

⇒ = + ∈ ∈ (2) Từ (1) (2) suy ra:

2 5 a k b k  = +   =



2 2 5 a k b k  =   = + 

Suy chỉmột sốa b chia hết cho Cách 3:

2 2

24a + =1 b ⇔24ab = −1 không chia hết a b không đồng thời chia hết

cho

+ Giả sửa b không chia hết cho

Theo định lý Fermat ta có 2 2

1(mod 5)

( )( ) 0(mod 5)

1(mod 5)

a

a b a b

b  ≡  ⇒ + − ≡  ≡ 

Nếu 2

0(mod 5)

a +b ≡ 25a2+ =1 a2 +b2 ≡0(mod 5)( vơ lí)

Suy 2

0(mod 5)

ab ≡ 2

23a b a 0(mod 5)

⇒ + = − ≡ (*) Vì a khơng chia hết a≡ ± ±1, 2(mod 5)

Với 2

1(mod 5) 1(mod 5) 23 1 1(mod 5)

a ≡ ± ⇒a ≡ ⇒ a + ≡ − ( trái với (*))

Với 2

2(mod 5) 4(mod 5) 23 3(mod 5)

a≡ ± ⇒a ≡ ⇒ a + ≡ ( trái với (*))

Vậy điều giả sửlà sai Từđó suy điều cần chứng minh Câu 74.

Do q sốnguyên tốlớn nên q có dạng 3k 1+ 3k 2+ với k N∈ *

+ Nếu q 3k 1= + , p q 2= + nên p 3k 3= + chia hết cho 3, trường hợp loại p sốnguyên tố

+ Nếu q 3k 2= + , p q 2= + nên p 3k 4= + Do p sốnguyên tốnên k phải số tựnhiên lẻ Khi ta p q k 12+ = ( + ) Vậy sốdư chia p q+ cho 12

Câu 75.

Từgiảthiết a3+b3 =2 c 8d( 3− 3) ta có

( ) ( ) ( ) ( ) ( )

( ) ( ) ( ) ( )

3 3 3

3 3 3

a b 3ab a b c d 3cd c d 3ab a b 3cd c d 3c 15d a b c d 3ab a b 3cd c d 3c 15d

+ + + + + + + = + + + + −

⇔ + + + = + + + + −

Dễ thấy 3ab a b 3cd c d 3c 15d( + )+ ( + )+ 3− chia hết ta (a b+ ) (3+ +c d)3 chia hết cho

Mặt khác ta lại có (a b+ ) (3+ +c d) (3 = a b c d+ + + )3−3 a b c d a b c d( + )( + )( + + + )

(187)

Mà a b c d a b c d( + )( + )( + + + ) chia hết suy (a b c d+ + + )3 chia hết cho Do a b c d+ + + chia hết cho

Chú ý: Bản chất tốn tốn bản: Nếu x y3+ chia hết cho x y+ chia hết cho

Câu 76 Ta có:

Với số nguyên chia hết cho

Vậy chia hết cho 18

Câu 77

Ta có:

Mà nên mà sốnguyên tốnên

nên

Từ mà sốnguyên tố

và mà nên

Vậy chia hết cho Câu 78

Vì nên Xét hiệu:

chia hết cho Do chia hết cho (đpcm) Câu 79

Ta có: 1947 = 3.11.59

Đặt

*

Suy ra:

     

3

15 1

3n = 3 3 6n

A  n n  n n   nn n  

 1  1

,

n nn n  n

 1  1

3

A  nn n  n 

( 2)

+ + 

a ab b ⇒4(a2+ab b+ 2)9

( )2 2

2

 

⇒ a b− + b  ( )1

2

3b 3 (2a b− )23 (2a b− )3

(2a b− )3 (2a b− )29 ( )2

( )1 ( )2 ⇒3b29⇒b23 ⇒b3

(2a b− )3 b3 ⇒ 2a3 ( )2;3 =1 a3

a b

2018

2019 3 (a1+a2+ + an)3

( 3 3) ( ) ( ) ( ) ( ) ( ) ( ) ( )

1 n n 1 1 2 n n n

a +a + +aa +a + +a = aa a + + aa a + + + aa a +

3 a13+a23+a33+ + an3

46n 296.13n

A= +

46 1(mod 3)

13 1(mod 3)

n n n n

 ≡ = 

≡ = 

1 296 297 (mod 3) (1)

A≡ + ≡ ≡ ⇒ A

(188)

*

Suy ra:

*

Vì n sốtựnhiên lẻ

Mà 3; 11; 59 đôi nguyên tốcùng nên từ(1), (2), (3)

Cách 2:

Ta có: 1947 = 33 59 Đặt

Lại có:

(vì n lẻ)

Mà nên

Câu 80

Ta có:

+ tích hai sốnguyên liên tiếp nên chia hết cho 2.

+ Xét tac có:

- Nếu n chia hết cho 3thì chia hết cho 3

- Nếu nchia 32 thì chia hết cho 3 nên sẽchia hết cho 3

- Nếu nchia 31thì chia hết cho 3nên sẽchia hết cho 3

Vậy trường hợp sẽchia hết cho3.

46 1(mod11)

13 1(mod11)

n n n n

 ≡ = 

≡ = 

2n 296.2n 297.2n 11.27.2n (mod11) 11 (2)

A≡ + ≡ ≡ ≡ ⇒ A

46n ≡ −( 13)n ≡ −13 (mod13)n

13 296.13 295.13 5.59.13 0(mod 59)

59 (3)

n n n n

A A

⇒ ≡ − + ≡ ≡ ≡ ⇒ 

(3.11.59) 1947

A A

⇒  ⇒ 

( )

46n 296.13n 46n 13n 297.13n 46n 13n 297.13n

A= + = − + = − +

(46 13 ) 33.9.13 33( 9.13 ) 33

n n

A= − A + = A + 

46n 296.13n 46n ( 13 )n 295.13n 46n ( 13 )n 295.13n

A= + = − − + = − − + +

[46 ( 13) ] 59.5.13

n

A= − − A + +

( )

59 A 5.13n 59

= + 

(33;59)=1 A(33.59) 1947=

3 2

2n +3n + =n n n(2 +3n+ =1) n n( +1)(2n+1)

( 1)

n n+

3 2

2n +3n + =n n n(2 +3n+ =1) n n( +1)(2n+1)

3

2n +3n +n

1

n+ 2n3+3n2+n

2n+1 2n3+3n2+n

3

2n +3n +n

(189)

Ta có nên chia hết cho với sốnguyên Câu 81

Ta có chia hết cho Mặt khác

chia hết cho Do chia hết cho

Câu 82

Ta xét 2014 số khác có dạng 20142014…2014 = an, có n 2014 n ∈N* Trong 2014 số có hai số chia cho 2013 có số dư

Giả sử số , aj (j > i) Khi aj – ai 2013 hay:

j sơ 2014 i sơ 2014 j í sơ 2014 4i sơ

20142014 2014 20142014 2014 20142014 2014 0000 0000 2013 −

− = 

    Số có dạng 20142014…2014 .104i  2013

Vì UCLN(10, 2013) = nên UCLN(10n, 2013) = với n ∈N* Vậy: có số dạng 20142014…2014 chia hết cho 2013 Câu 83

Từ giả thiết suy a≡1 (mod 3), a=3k+1(k);b≡2 (mod 3), b=3q+2 (q)

Suy A=4a+ + + ≡ + + +9b a b 1 (mod 3) hay A≡4 (mod 3) (1)

Lại có:

4a =4 k+ =4.64k ≡4 (mod 7)

3

9b =9q+ ≡2 q+ (mod 7) ⇒ 9b ≡4.8q ≡4 (mod 7)

Từ giả thiết ta suy a≡1 (mod 7),b≡1 (mod 7)

Dẫn đến A=4a+ + + ≡ + + +9b a b 4 1 (mod 7) hay A≡10 (mod 7)

Từ (1) suy A≡10 (mod 3); mà nguyên tố nên A≡10 (mod 21)

Vậy A chia cho 21 dư 10

Câu 84

Ta có : x5 – x = x( x4 – 1)= x(x2 – 1)(x2+ 1)= x(x2 – 1)

(x 4)

 − + 

 

=(x – 2)(x – 1)x(x + 1)(x + 2) + 5(x – 1)(x + 1)x Ta có : (x – 2)(x -1) x(x + 1)(x + 2) chia hết ch

mà (5,6) = nên (x – 2)(x -1) x(x + 1)(x + 2)30

lại có (x-1)x(x+1) Chia hết cho mà (2,3)=1 nên 5(x – 1)x(x+1) 30

Do x5 – 1 30

Suy A = (a2020+ b2020+c2020) - (a2016+ b2016+ c2016) A = a2015(a5 – a) + b2015(b5 – b) + c2015(c5 – c) 30 Vậy A30

( )2;3 =1 2n3+3n2 +n n

( ) ( )

3

2018 2016

a b+ = −c c⇔ + + = −a b c c c c+ − c

( 3 3) ( ) ( ) ( ) ( ) ( ) ( ) ( )

1 1 1

a + +b c − + + =a b c aa a+ + −b b b+ + −c c c+

3 3

A=a + +b c

(190)

Câu 85.

Trước tiên, ta chứng minh x 

Đặt y5= a, a ∈N*, ta có 2x2 – = y15⇔2x2= a3+ ⇔2x2= (a + 1)(a2 - a + 1) (1) Gọi ƯCLN(a + 1; a2 – a + 1) = d (d ∈N*), ta có: a + d, a2 – a +  d

Suy ra(a2 – a + 1) – (a + 1)(a – 2) =  d⇒d = d = * Nếu d = từ(1), ta có:

2

a 2

a a 1 x

+ = 

− + =

2

a 1 x a a 2

 + = 

− + =

 (loại a ∉N*)

2

a 2

a a 1 x

+ = 

− + =

a 1 a 1

x 1

x 1

= =

 

⇔ ⇒ =

= 

 (loại phải có x > 1)

* d = từ(1) ta có: 2x2 Vì ƯCLN(2; 9) = 1nên x2 ⇒x  (*) Chứng minh x 

Đặt y3= b, b ∈N*, ta có: 2x2 – = b5⇔2x2= b5+

⇔2x2= (b + 1)(b4 – b3+ b2 – b + 1) (2)

Gọi ƯCLN(b + 1; b4 – b3+ b2 – b + 1) = k (k ∈N*) Ta có: b +  k; b4 – b3+ b2 – b +  k

⇒(b4 – b3+ b2 – b + 1) – (b + 1)(b3 – 2b2+ 3b – 4) = 5 k Suy k = k =

* Nếu k = từ(2) có

4

b 1 x

b b b b 2

 + = 

− + − + =

 (loại b ∉N*) Hoặc: 2

b 1 2

b b b b 1 x

+ = 

 − + − + =

⇒ b 1 x 1 =   =

 (loại phải có x > 1)

* Nếu k = từ(2) suy 2x2 25 Vì ƯCLN(2; 25) = nên x2  25 ⇒x 5 (**) Từ(*) (**) suy x  BCNN(3; 5) hay x  15 (đpcm)

Câu 86

Giả sử A số tự nhiên tạo thành cách viết 100 số theo hàng ngang Khi số tự nhiên A có 21 chữ số 20 chữ số từ đến

(191)

Do tổng chữ số A 21.1 + 20 + + + + + + + = 901( ) không chia hết cho

Mà 2016 chia hết cho A khơng thể chia hết cho 2016 Câu 87

Giả sử tồn số k∈{0;1; 2; 3} để tồn số tự nhiên n cho (n2−k 4) Khi ta xét

trường hợp sau

• Trường hợp 1: Nếu n 4q= với q số tự nhiên Khi n k 16q2 − = −k

Do để (n2−k 4) thì k 4 nên suy k 0=

• Trường hợp 2: Nếu n 4q 1= ± với q số tự nhiên Khi n k 16q2− = 2±8q k+ −

Do để (n2−k 4) thì 1 k 4−  nên suy k 1=

• Trường hợp 3: Nếu n 4q 2= + với q số tự nhiên Khi n k 16q 16q k2− = + + −

Do để (n2−k 4) thì k 4 nên suy k 0=

Vậy với k 0= k 1= ln tồn số tự nhiên n để (n2−k 4)

Câu 88.

Ta có: (n n 2n+ )( + ) ( ) ( )= 2n ! =(1.3.5 2n 2.4.6 2n( − ))( )

n! n!

= ( − ) n = ( − ) n

n!

1.3.5 2n 1.3.5 2n n!

Do đó: (n n 2n+ )( + ) ( ) chia hết cho 2n

Chương II

CÁC BÀI TỐN VỀ SỐ CHÍNH PHƯƠNG

Bài 1:

Ta có: 2 ( )( )

1

+ = + + + = + +

a a ab bc ca a b a c

Tương tự: ( )( ) ( )( )

1 ;

+ = + + + = + +

b a b b c c b c c a

Do đó: ( 2 )( 2 )( 2 ) ( )( )( )

1 1

+ + + = + + + 

a b c a b b c c a

(192)

Vậy toán chứng minh Bài 2:

Đặt n(2n – 1) = 26q2 (1)

Do VP chẵnvà (2n – 1) lẻ nên n chẵn hay n = 2k Do đó: (1) suy k(4k – 1) = 13q2 (2)

Nhận thấy (k, 4k – 1) = nên:

( )1 k u2 2v k 13u22

4k 13v 4k v

 =  =   ⇒   − =  − =   

Xét trường hợp ta có:

( ) ( )

2

2 2 2

2

k u

4k 13v 12v v v v mod vo ly

4k 13v  =

 ⇒ = + = + + ⇒ + ⇒ ≡ 

− =

 

Xét trường hợp ta có:

( )

2

2

k 13u 4k v vo ly

4k v  =

 ⇒ = + 

− = 

Vậy không tồn n thỏa mãn yêu cầu đầu Bài 3:

Ta có A = 2( )

1

n +n +n =n n + +n

Với n = A = (thỏa mãn)

Với n≠0 A số phương

1

n + +n số phương

Khi 2 ( )

1

n + + =n k k∈ ( ) ( )2

4 n n 4k 2n 4k

⇒ + + = ⇒ + − = − (2n 2k)(2n 2k)

⇒ + − + + = −

Vì 2n+ +1 2k≥2n+ −1 ,k ∀ ∈n ,k∈ nên

2

2

2

2

n k n k n k n k  + − = −   + + =    + − = −  + + =  

2

1

2

n k n n k + − = −  ⇒ = −  + + =

 (thỏa mãn)

2

0

2

n k n n k + − = −  ⇒ =  + + =

 (loại)

Vậy n=0;n= −1 Bài 4:

Ta có A = (x + y)(x + 2y)(x + 3y)(x + 4y) +

y

= ( 2 2

5 4 )( 5 6 )

x + xy+ y x + xy+ y +y

Đặt 2

5 5 ( )

x + xy+ y =t tZ

(193)

A = ( 2 4 2 2

)( ) ( 5 5 )

ty t+y + y = −t y + y = =t x + xy+ y

Vì x, y, z ∈ Z nên 2 2

, , 5

xZ xyZ yZx + xy+ yZ

Vậy A số phương Bài 5:

a) Ta có:

 

 

 

2

2

2 2

2 2

2

224 99 9100

224.10 99 9.10 10

224.10 10 10 10

224.10 10 10 10

225.10 90.10

15.10                             n n

n n n

n n n n

n n n n

n n

n

A

Vậy A sốchính phương

b) Ta có :

      2 11 155 11 155

11 1.10 5.11 1

10 10

.10

9

10 10 5.10

9

10 4.10

9 10                              n n n n n n n n n n

n n n

n n

n

B

Do B sốchính phương

Bài 6:

Giả sử: n2;n1; ;n n1;n2 với 2 n  sốtựnhiên liên tiếp

Ta có:   2 2 2   2 2  2 

2 1

         

n n n n n n

n khơng thểcó chữ sốtận nên n2255n22 khơng số

chính phương

Vậy tổng bình phương sốtựnhiên liên tiếp khơng phải sốchính phương

(194)

Bài 7:

Ta có      

1

44 488 89 44 488 44 10n 11 1

n nn n n n

= + = + +

10 10

4 .10

9

n n

n

− −

= + +

2

4.10 4.10 8.10 4.10 4.10

9

nn+ n− + n+ n+

= =

2.10

3 n

 +  =  

 

Ta thấy 2.10n+ =1 200 01

( có n−1 chữ số ) có tổng chữ số chia hết chia hết cho

Suy 2.10

n

 +  ∈

 

   hay số có dạng 44 488 89 số phương

Bài 8:

p tích nsốngun tốđầu tiên Nên p2 p không chia hết cho ( )1

a) Giả sử p+1 sốchính phương Đặt p+ =1 m m2( ∈)

p chẵn nên p+1 lẻ ⇒m2 lẻ ⇒m lẻ

Đặt m=2k+1(k∈) Ta có: m2 =4k2+4k+1

2

1 4

p k k

⇒ + = + +

( )

4 4

p k k k k

⇒ = + = +  mâu thuẫn với ( )1

p

⇒ + sốchính phương

b) p= ⋅ ⋅ ⋅⋅⋅2 sốchia hết cho ⇒ −p có dạng 3k+2 Khơng có sốchính phương có dạng 3k+2

Nên p−1 khơng sốchính phương

Vậy p tích n sốngun tốđầu tiên p−1 p+1 khơng sốchính phương Bài 9:

Giả sử 2010 + n2là số phương 2010 + n2= m2(m∈N )

(195)

Từ suy m2 - n2 = 2010 ⇔(m + n) (m – n) = 2010 Như số m n phải có số chẵn (1)

Mặt khác m + n + m – n = 2m ⇒ số m + n m – n tính chẵn lẻ (2) Từ (1) (2) ⇒ m + n m – n số chẵn

⇒ (m + n) (m – n)  2006 không chia hết cho

⇒ Điều giả sử sai

Vậy không tồn số tự nhiên n để 2010 + n2là số phương. Bài 10:

Gọi sốtựnhiên liên tiếp n−2,n−1, ,n n+1,n+2(n∈,n≥2 )

Ta có: ( ) (2 )2 2 ( ) (2 )2 ( 2 )

2 1

n− + n− +n + n+ + n+ = n +

n khơng thểtận 3hoặc

Do

2

n + không thểchia hết cho

Suy ra: ( )

5 n +2 khơng sốchính phương

Hãy nói cáchkhác: A khơng sốchính phương

Bài 11:

Vì n + 2n + số phương nên đặt n + = k2, 2n + = m2(k, m ∈N) Ta có m số lẻ ⇒ m = 2a + ⇒ m2= 4a(a + 1) +

Mà ( 1)

2 ) (

1

2

+ =

+ =

= m a a a a n

⇒ n chẵn ⇒ n + lẻ ⇒ k lẻ ⇒ đặt k = 2b + (với b∈N) ⇒ k2= 4b(b+1) +

⇒ n = 4b(b+1) ⇒ n  (1) Ta có: k2+ m2= 3n + ≡ 2 (mod3)

Mặt khác k2chia cho dư 1, m2chia cho dư Nên để k2+ m2≡ (mod3) k2 ≡ (mod3)

m2 ≡ (mod3)

⇒ m2 – k2  hay (2n + 1) – (n + 1)  ⇒ n  (2) Mà (8; 3) = (3)

Từ (1), (2), (3) ⇒ n  24 Bài 12:

Gọi số phương phải tìm là: aabb = n2với a, b ∈ N, ≤ a ≤ 9; ≤ b ≤

Ta có: n2= aabb = 11 a0b = 11.(100a + b) = 11.(99a + a + b) (1)

(196)

Nhận xét thấy aabb  11 ⇒ a + b  11

Mà ≤ a ≤ 9; ≤ b ≤ nên ≤ a + b ≤ 18 ⇒ a + b= 11

Thay a + b = 11 vào (1) n2= 112(9a + 1) 9a + số phương Bằng phép thử với a = 1; 2;…; ta thấy có a = thoả mãn ⇒ b =

Số cần tìm là: 7744 Bài 13:

Gọi số lẻ liên tiếp 2n - 1; 2n + 1; 2n + (n ∈N) Ta có: A =(2n – 1)2+ (2n + 1)2+ (2n +3)2= 12n2+ 12n + 11

Theo đề ta đặt 12n2+ 12n + 11 = aaaa = 1111 a với a lẻ ≤ a ≤ 9

⇒ 12n(n + 1) = 11(101a – 1)

⇒ 101a –  3⇒ 2a – 

Vì ≤ a ≤ nên ≤ 2a – ≤17 2a – lẻ nên 2a – ∈{3;9;15} ⇒ a∈{2;5;8}

Vì a lẻ ⇒ a =5 ⇒ n = 21 số cần tìm là: 41; 43; 45 Bài 14:

Ta có ( )

2

444 444 000 444 444 10n 888

n n n n n n

A= =   +  =  − +

=

2

4.111 1.999 4.111 1.9.111 6.111

n n n n n

B B   B

+ = + =  +

 

     =

2 2

3

.888

4 n B 4B B

   

+ = +

     

Khi

2 2

3 3

2 4 .2

4 4

A+ B+ = B + +B B+ = B + B + = B+ 

     

=

2 2

1

3

.888 3.222 2 666 68

4 n n n

     

+ = + =

     

       

Ta có điều phải chứng minh Bài 15:

a 2N− =1 2.1.3.5.7 2007 1−

Có 2N 3 ⇒2N−1 khơng chia hết cho 2N− =1 3k+2(k∈) Suy 2N−1 khơng sốchính phương

b 2N =2.1.3.5.7 2007

N lẻnên N khơng chia hết cho 2N2

(197)

Nhưng 2N không chia hết cho

2N chẵn nên 2N không sốchính phương

c. 2N+ =1 2.1.3.5.7 2007 1+

2N+1lẻnên 2N+1 không chia hết cho

2N không chia hết 2N+1không chia cho dư

Do đó: 2N+1khơng sốchính phương

Bài 16:

Kí hiệu pnlà số nguyên tố thứ n Giả sử tồn số tự nhiên m

( )

2 * ; ,

− = = ∈

m m

S a S b a b N

S1 =2;S2 =10;S4 =17⇒ >m 4

Ta có: 2 ( )( )

1 .

= − = − = − +

m m

p S S b a a b a bpmlà số nguyên tố b + a >

Nên  + =− =1

m

b a

b a p Suy ra: ( )

2 1

2 1 2 1 1

2

+

 

= − = − ⇒ =  

 

m

m m m

p

p b S S

Do m > 4 nên ( )

2

1

1 1

1 2 9 8

2 2

+ +

   

≤ + + + + + + − − =  − < 

   

m m

m m m

p p

S p p

mâu thuẫn với (1)

Nên dãy số S1, S2,…… không tồn hai số hạng liên tiếp số phương Bài 17:

Do p số nguyên tố nên ước số nguyên dương p4là: 1; p; p2; p3; p4 Đặt S = 1+ p + p2+ p3 + p4

Giả sử S = n2 ( ) ( )

4n 4p 4p 4p 4p n

⇒ = + + + + ∈

Ta có: 4 3 2 ( )2 4 2 3 2

4p +4p +p < 2n <4p +p + +4 4p +8p +4p

( 2 )2 ( )2 ( 2 )2

2p p 2n 2p p

⇔ + < < + + 2 ( 2 )2 ( )

4n 2p p

⇔ = + + Từ (1) (2) suy

p −2p 3− = ⇔ =0 p

Thử lại với p = thỏa mãn Vậy số nguyên tố cần tìm là: p = Bài 18:

Đặt n 14n 256 k k N2− − = 2( ∈ ) (⇔ n 7− )2−k2 =305 ⇔(n k n k 7− − )( + − )=305 1.305 61.5= =

Xét trường hợp: n + k - > n – k –

(198)

Trường hợp 1: n – k – 7= n + k – = 305 => n = 160(nhận) Trường hợp 2: n – k – = - 305 n + k – = -1 => n = -146 (loại) Trường hợp 3: n – k – = n + k – = 61 => n = 40(nhận) Trường hợp 4: n – k – = -61 n + k – = -5 => n = -26 (loại) Vậy n = 40, k = 28 n = 160 , k = 152

Bài 19:

Ta có: 1 1 ab bc ca a b c abc+ + = ⇒ + + =

2

1 a ab bc ca a a(a b) c(a b) (a b)(a c)

⇒ + = + + + = + + + = + +

2

1 b ab bc ca b b(a b) c(a b) (a b)(b c)

⇒ + = + + + = + + + = + +

2

1 c ab bc ca c b(a c) c(a c) (a c)(b c)

⇒ + = + + + = + + + = + +

( )( )( ) ( ) ( ) ( ) ( )( )( )

⇒ + + + = + + + = + + + 2

1 a b c a b b c a c a b b c c a Vì a, b, c

các sốnguyên ⇒ +(a b)(b c)(c a) Z+ + ∈

2 2

(1 a )(1 b )(1 c )

⇒ + + + sốchính phương Bài 20:

- ĐểA sốchính phương A = n2+n+6 = a2(a

N

∈ )

- Ta có: n2+n+6 = a2

( ) ( )

( ) ( )

2

2

4n 4n 24 4a 2a 2n 23

2a 2n 2a 2n 23

⇔ + + =

⇔ − + =

⇔ + + − − =

- Vì a,n sốtựnhiên nên (2a +2n +1) sốtựnhiên 2a +2n +1 > 2a – 2n -1 Do

2a 2n 23 4a 24 a

2a 2n 1 4n 20 n

 + + =  =  =

⇔ ⇔

 − − =  =  =

  

- Vậy n = Bài 21:

Ta có

+ dlà sốnguyên tốvà abcd sốchính phương nên d =5

+ ( )2

2

10000 100 ;

abcd< = ⇒abcd = x với x∈{1; 2;3; 4; ;9}

+ Vì abcd chia hết cho ( )x5 29⇒x5 3 ⇒ + ∈x {6;9;12}⇒ ∈x {1; 4; 7} Kiểm tra lại ta hai số: 2015 5625

Bài 22:

Gọi M = + 22+ 23+ + 298 ⇒ S = + M

(199)

M = 2M – M = (22+ 23+ + 298+ 299) – (2 + 22+ 23+ + 298) M = 299 –

⇒ S = 299= (24)24.23= 8.1624 Vì 1624có chữ sốtận

⇒ Scó chữ sốtận Nên S khơng sốchính phương Bài 23:

4x +14x +9x−6 sốchính phương, nên ta có 4x3+14x2+9x−6=k2với k∈N

Ta có

14

x + x + x− =…=(x+2 4)( x2+6x−3)nên ta có (x+2 4)( x2+6x−3)=k2

Đặt ( )

2,

x+ x + x− =d với d∈N*

Ta có x+2d⇒(x+2 4)( x−2)d⇒4x+6x−4d

Ta lại có ( ) ( )

4x +6x−3d ⇒ 4x +6x− −3 4x +6x−4 =1d ⇒ =d Vậy ( )

2,

x+ x + x− =

mà ( )( )

2

x+ x + x− =k2 nên ta có

x+2

4x +6x−3 sốchính phương⇒ + =x a v2 4x2+6x− =3 b2 với a,b∈N*

Vì x > nên ta có 2 2 2 ( )2 2 ( )2

4x <b <4x +12x+ ⇔9 2x <b < 2x+3

Vì b lẻnên 2 ( )2 2 2

2 4

b = x+ ⇔ x + x− = x + x+ ⇔ =x

Với x = ta có

4x +14x +9x−6=100=102là sốchính phương Bài 24:

Giả sử: 2

17

n + =k ( kN) k >n ( )( ) 17 17

k n

k n k n n

k n

− = 

− + = ⇔ + = ⇒ =

Vậy với n=8 thỏa mãn yêu cầu toán Bài 25:

Đặt A=2n + +3n 4n Nếu n=1 A=9 (thỏa mãn)

Xét n>1 hay n≥2 2n +4n chia hết cho

Ta có 3n chia dư với n chẵn −1 với n lẻ Mà sốchính phương chia

dư 1nên A phải chia dư 1nên 3n phải chia dư Suy n chẵn

Với n chẵn: 2n chia dư 1, 4n chia dư 1, 3n chia hết cho

Do A chia dư (vơ lí, sốchính phương chia có sốdư 1)

Vậy n=1 Bài 26:

Giả sử 2

2014 ( )

+ = ∈

n k k N

⇔2014=k2−n2 ⇔2014=(k+n k)( −n) (1)

(200)

Suy (k + n) (k – n) = 2k sốchẵn nên (k+ n) (k – n) tính chẵn lẻ Do 2014 sốchẵn nên (k + n) (k – n) sốchẵn

Khi từ(1) suy ta lại có (điều vơ lí)

Vậy khơng có sốngun n để sốchính phương Bài 27:

Ta có: ( )( )

3 2

xx + + =x xx − −x

* Xét x− = ⇒ =2 x 2: thỏa mãn yêu cầu toán

* Xét

1

x − − =x : Loại

* Xét

2

x− =x − −x ta có: x=1

* TH x≠2; x≠1 Với x nguyên ta chứng minh (x−1;x2− − =x 1)

Nên

3

xx + +x số phương x−2 x2 − −x số

phương Để

1

x − −x sốchính phương x2− − =x y2 với y∈

Tìm x=2(loại x≠2) x= −1 Thửlại x= −1 ta có x3−3x2+ +x có giá

trịbằng −1khơng phải sốchính phương nên ⇒ = −x (loại)

Vậy x=2 x=1 x3−3x2+ +x sốchính phương

Bài 28:

Nếu mệnh đềb) A + 51 có chữ sốtận A – 38 có chữ sốtận nên hai sốnày không sốchính phương Vậy mệnh đềb) sai mệnh đềa)và c)

Giả sử 2

51 ; 38 ( , ; )

A+ =m A− =n m nN m>n

2

89

m n

⇒ − = hay (m – n)(m + n) = 89

Vì 89 sốnguyên tốnên m + n = 89 m – n = => m = 45 n = 44 nên A = 1974

Bài 29:

Giả sửtồn số hữu tỉ n sốnguyên dương m để n2+ +n 503=m2

Vì: n số hữu tỉnên tồn a b, ∈Z b, ≠0 cho n a b

= ( )a b; =1

Ta có:

2

2 2 2 2

503 a a 503 503

n n m m a ab b m b

b b

 

+ + = ⇒  + + = ⇔ + + =

 

( )

2 2

503

a b a b m b a b

⇔ = − + − ⇒ 

Mà ( )a b; =1nên b=1hay b= ∈a Z

Do đó: 2 2 2 2 2 ( )2

503 4 2012 4 2011

n + +n =mn + n+ = mmn+ =

4 ) )( (k+n kn

4 2014

2014

2 +

n

Ngày đăng: 24/02/2021, 04:55